• Shuffle
    Toggle On
    Toggle Off
  • Alphabetize
    Toggle On
    Toggle Off
  • Front First
    Toggle On
    Toggle Off
  • Both Sides
    Toggle On
    Toggle Off
  • Read
    Toggle On
    Toggle Off
Reading...
Front

Card Range To Study

through

image

Play button

image

Play button

image

Progress

1/504

Click to flip

Use LEFT and RIGHT arrow keys to navigate between flashcards;

Use UP and DOWN arrow keys to flip the card;

H to show hint;

A reads text to speech;

504 Cards in this Set

  • Front
  • Back
a pt in involved in a car accident is fully conscious, and his voice is normal
does he need to be intubated?
(1) trauma, abcs

NOPE! doesn't need to be intubated now, if voice is normal his airway is NOT in immediate risk
a patient with multiple stab wounds arrives in the ER fully conscious, and he has a normal voice, but he also has an expanding hematoma in the neck

whats the next step in management?
(2) trauma - abcs

intubate STAT by orotracheal intubation with rapid sequence anesthetic intubation & pulse oximetry

the airway may be fine now, but its going to be compromised soon. intubation is indicated now before an emergency situation develops
a patient with multiple stab wounds arrives in the ER fully conscious, and he has a normal voice, but he also has subcutaneous air (emphysema) in the tissues in the neck & upper chest

whats the next step in management?
(3) trauma - abcs

intubate STAT by fiberoptic bronchoscopy (best intubation option with subcutaneous emphysema)

the airway may be fine now, but its going to be compromised soon. intubation is indicated now before an emergency situation develops
a patient involved in a severe car accident has multiple injuries and is unconscious. He is breathing spontaneously, but his breathing sounds gurgled and noisy

whats the next step in management?
(4) trauma - abcs

orotracheal intubation - but special note that since pt is already unconscious, doesn't need rapid anesthesia to intubate (duh)
an unconscious pt is brought in by the paramedics with spontaneous but noisy & labored breathing. They relate that at the accident site the pt was conscious, but was complaining of neck pain and was able to move his lower extremities. He lost consciousness during the ambulance ride, and effort to secure a nasotacheal airway was unsuccessful

whats the next step in management?
(5) trauma - abcs

key: find an answer that provides AIRWAY w/o moving the neck
key: DONT choose to do imaging before airway is secured!

The perennial dilemma of airway management is what to do when there may be a cervical spine injury, but an airway is needed fast. The standard approach is that the airway cannot wait for neck x-rays or elaborate neck traction to be done. Orotracheal intubation can still be performed with manual in-line cervical immobilization (i.e., intubate without whipping the neck around), or better yet over a flexible bronchoscope. Some authors prefer nasotracheal intubation in this setting if facial injuries do not preclude it.
A patient involved in a severe automobile crash is fully awake and alert, but he has extensive facial fractures and is bleeding briskly into the airway, and his voice is masked by gurgling sounds.
(7) trauma - abcs

Securing an airway is mandatory, but the orotracheal route will not be suitable.

**Cricothyroidotomy is probably the best choice under these circumstances.

**Percutaneous transtracheal ventilation is another option (an intravenous catheter is placed into the trachea with high-pressure oxygen delivery).

The old "emergency tracheostomy" can be a horror show and is no longer favored.
an unconscious trauma pt has been rapidly intubated in the ER> he has spontaneous breathing and bilateral breath sounds, and his oxygen saturation by pulse oximetry is above 95

whats the next step in management?
(8) trauma - abcs

As far as breathing is concerned, he is moving air (physical examination) and getting oxygen into his blood (oximetry). Deterioration could occur later, but right now we are ready to move to the "C" in the ABCs. The three conditions that might produce inadequate breathing are plain pneu- mothorax, tension pneumothorax, or flail chest with underlying pulmonary contusion. We will review those with other types of chest trauma.
A 22-year-old gang member arrives in the E.R. with multiple guns shot wounds to the abdomen. He is diaphoretic, pale, cold, shivering, anxious, asking for a blanket and a drink of water. His blood pressure is 60 over 40. His pulse rate is 150, barely perceptible.
Dx?
Management? (3)
Tx?
(9a) trauma - abCs - Circulation/Shock

Dx: Hypovolemic shock

Management: Big bore IV lines, Foley catheter and I.V. antibiotics.

Tx: Ideally Exploratory Lap immediately for control of bleeding, and then fluid and blood administration.
During a bank robbery an innocent bystander is shot repeatedly in the abdomen. When the emergency medical technicians (EMTs) arrive, they find him to be in shock. A fully staffed trauma center is 2 miles away from the site of the shooting.

whats most imp for the paramedics to do onsite?
(10) trauma - abCs - Circulation/Shock

scoop & run- dont waste time!

An ambulance can travel 2 miles in 2 minutes- maybe 3. The point of the vignette is that elaborate attempts to start an IV at the site and begin to infuse Ringer lactate would waste precious time that would be best spent moving the patient to a place where the urgently needed laparotomy can be done ("scoop and run").
A 19-year-old male is shot in the right groin during a drug deal gone bad. He staggers to the hospital on his own, and arrives in the ER with a blood pressure of 90 over 70 and a pulse rate of 105. He is squirting bright red blood from the groin wound.

whats the next steps?
(11) trauma - abC - circulation/ shock

direct pressure to control bleeding first

The point of this vignette is that control of the bleeding by direct local pressure is the first order of business before volume restoration is started. And a gloved finger or a sterile pressure dress- ing is the way to do it- not blind clamping or a tourniquet.
A car accident victim has arrived in the ER, and the initial survey indicates that he is unconscious, with spontaneous but noisy breathing and a blood pressure of 80 over 60 with a pulse rate of 95. His head and neck veins are not obviously distended. While the anesthesia team is intubating him, another team is placing a central line for central venous pressure(CVP) measurement, and others are examining his chest and abdomen.
Dx?
Management?
(12) trauma - abC - circulation/ shock

Hypovolemic shock - likely bleeding from unknown site + trouble breathing

The emphasis on control of bleeding first and fluid replacement later cannot be implemented if we do not know yet where the bleeding is coming from, and whether it might stop sponta- neously or not. In a case like this, two large (16-gauge) peripheral lines should be started, and Ringer lactate should be poured in. At one time central venous lines were deemed essential for fluid resuscitation, but short, wide catheters in peripheral veins work better, and placing them does not interfere with other ongoing therapeutic and diagnostic maneuvers. Percutaneous femoral vein catheter is an acceptable alternative when peripheral IVs are hard to start. Saphenous vein cut-downs, which were very popular in the 1950s, have also made a comeback as a suitable route.
A 4-year-old child has been shot in the arm in a drive-by shooting. The site of bleeding has been controlled by local pressure, but he is hypotensive and tachycardic. Two attempts at starting peripheral IVs have been unsuccessful.
Dx?
Management?
(13) trauma - abC - circulation/ shock

In this age group, the access of last resort is intraosseous cannulation in the proximal tibia. The initial bolus of Ringer lactate would be 20 ml/kg of body weight.
During a wilderness trek, a 22-year-old man is attacked by a bear and bitten repeatedly in the arms and legs. His trek companion manages to kill the bear and to stop the bleeding by applying direct pressure, but when paramedics arrive 1 hour later, they find the patient to be in a state of shock. Transportation to the nearest hospital will take at least 2 hours.
Dx?
Management?
(14) trauma - abC - circulation/ shock

All the training that paramedics took to enable them to infuse IV fluids has not been wasted. In the urban setting we now prefer rapid transportation to the hospital ("scoop and run"), but in this case prompt and vigorous fluid resuscitation is in order. The preferred fluid is Ringer lactate (without sugar), infusing at least a couple of liters in the first 20 or 30 minutes.
A 22-year-old gang member arrives in the ER with multiple gunshot wounds to the chest and abdomen. He is diaphoretic, pale, cold, shivering, anxious, and asking for a blanket and a drink of water. His blood pressure is 60 over 40. His pulse rate is 150, barely perceptible.

Dx?
Management?
(15) trauma - abC - circulation/ shock

What is it? Hypovolemic shock is still the best bet, but the inclusion of chest wounds raises the possibility of pericardial tamponade or tension pneumothorax. As a rule, if significant findings are not included in the vignette, they are not present. Thus, as given, this is still a vignette of hypovolemic shock, but you may be offered in the answers the option of looking for the missing clinical signs: distended neck veins (or a high measured CVP) would be common to both tam- ponade and tension pneumothorax; and respiratory distress, tracheal deviation, and absent breath sounds on a hemithorax that is hyperresonant to percussion would specifically identify tension pneumothorax.
22-year-old gang member arrives in the E.R. with multiple guns shot wounds to the chest and abdomen. He is diaphoretic, cold, shivering, anxious, asking for a blanket and a drink of water. His blood pressure is 60 over 40. His pule rate is 150, barely perceptible. He has big distended veins in his neck and forehead. He is breathing OK, has bilateral breath sounds and no tracheal deviation.
Dx?
Diagnostic test?
Tx?
(16) trauma - abCs - Circulation/Shock
Dx: Pericardial tamponade

Diagnostic test: No X-Rays needed, this is a clinical diagnosis!
Do Pericardial window. (****in the meantime, can give IVF) or pericardiocentesis, tap, tube to evacuate blood from pericardial space

Tx: If positive, follow with Thoracotomy, and then Exploratory Lap.
A 22-year-old gang member arrives in the E.R. with a single gunshot wound to the precordial area. He is diaphoretic, cold, shivering, anxious, asking for a blanket and a drink of water. His blood pressure is 60 over 40. His pule rate is 150, barely perceptible. He has big distended veins in his neck and forehead. He is breathing OK, has bilateral breath sounds and no tracheal deviation.
Dx?
Management?
(17) trauma - abCs - Circulation/Shock

Dx: Pericardial Tamponade

Management: Exploratory Lap

(when the location of the wound strongly suggests pericardial tamponade, emergency thoracotomy might be done right away without prior pericardial window)
A 22-year-old gang member arrives in the E.R. with multiple gun shot wounds to the chest and abdomen. He has labored breathing is cyanotic, diaphoretic, cold and shivering. His blood pressure is 60 over 40. His pulse rate is 150, barely perceptible. He is in respiratory distress, has big distended veins in his neck and forehead, his trachea is deviated to the left, and the right side of his chest is tympantic, with no breath sounds.
Dx?
Management? (2 steps)
Tx?
(18) trauma - abCs - Circulation/Shock

Dx: Tension pneumothorax

Management:
1. Immediate big bore IV catheter placed into the right pleural space (2nd intercostal midclavicular)
2. followed by Chest Tube to the right side, Immediately!

(Watch out for trap that offers chest X-Ray as an option. This is a clinical diagnosis, and patient needs that chest tube now. He will die if sent to X-Ray.)
A 22-year-old man is involved in a high-speed, head-on automobile collision. He arrives in the ER in coma, with fixed, dilated puprls. He has multiple obvious fractures in both upper extremities and in the right lower leg. His blood pressure is 70 over 50, with a barely perceptible pulse rate of 140.

where is he bleeding?
(19) Trauma - abC- circulation/ shock

NOT bleeding in the head! recognize he is in SHOCK & there isn't enough room in the brain for a intracranial bleed to cause shock, therefore he much be bleeding somewhere else

We have pointed out that shock in the trauma setting is caused by bleeding (the most common source), pericardial tamponade, or tension pneumothorax. This case fits right in, but the presence of obvious head injury might lead you into a trap: the question will offer you several kinds of intracranial bleeding (acute epidural hematoma, acute subdural hematoma, intracerebral bleed- ing, subarachnoid hemorrhage, etc.) as the answer. They are not. Intracranial bleeding can indeed kill you, but not by blood loss. There isn't enough room in the head to accommodate the amount of blood needed to go into shock (roughly a liter and a half in the average size adult). Thus, you need to look for another source (we will elaborate in the section on abdominal trauma).
A 72-year-old man who lives alone calls 911 saying that he has severe chest pain. He cannot give a coherent history when picked up by the EMTs, and on arrival at the ER he is cold and diaphoretic and his blood pressure is 80 over 65. He has an irregular, feeble pulse at a rate of 130. His neck and forehead veins are distended, and he is short of breath.

dx/ management?
(20) nonTrauma - abC- circulation/ shock

What is it? old man, chest pain, straight- forward cardiogenic shock from massive myocardial infarction (MI).
Management. Verify high CVP, electrocardiogram (ECG), enzymes, coronary care unit, etc. Do not drown him with enthusiastic fluid "resuscitation," but use thrombolytic therapy if offered.
A 17-year-old girl is stung many times by a swarm of bees. On arrival to the ER she has a blood pressure of 75 over 20 and her pulse rate is 150, but she looks warm and flushed rather than pale and cold. CVP is low.

dx/ management?
(21) nonTrauma - abC- circulation/ shock

Dx: Vasomotor shock (massive vasodilation, loss of vascular tone).

Management. Vasoconstrictors. Volume replacement would not hurt.
Twenty minutes after receiving a penicillin injection, a man breaks into hives and develops wheezing. On arrival at the ER his blood pressure is 75 over 20 and his pulse rate is 150, but he looks warm and flushed rather than pale and cold. CVP is low.

dx/ management?
(22) nonTrauma - abC- circulation/ shock

Dx: Vasomotor shock (massive vasodilation, loss of vascular tone).

Management. Vasoconstrictors. Volume replacement would not hurt.
In preparation for an inguinal hernia repair, a patient has a spinal anesthetic placed. His level of sensory block is much higher than anticipated, and shortly thereafter his blood pressure becomes 75 over 20, but he looks warm and flushed rather than pale and cold. CVP is low.

dx/ management?
(23) nonTrauma - abC- circulation/ shock

Dx: Vasomotor shock (massive vasodilation, loss of vascular tone).

Management. Vasoconstrictors. Volume replacement would not hurt.
A 17 year old girl is stung by a swarm of bees…or a man of whatever age breaks out with hives after a penicillin infection…or a patient undergoing surgery under spinal anesthetic…eventually develop BP of 75 over 25, pulse rate of 150, but they look warm and flushed rather than pale and cold. CVP is low.
Dx?
Management? (2)
(21-23) nontrauma shock - ABCs - circulation

Dx: Vasomotor shock
(massive vasodilation, loss of vascular tone)

Management: Vasoconstrictors and Volume replacement as needed
An 18-year-old man arrives in the ER with an ax firmly implanted into his head. Although it is clear from the size of the ax blade and the penetration that he has sustained an intracranial wound, he is awake and alert and hemodynamically stable.

management?
(1) Trauma - review from head to toe (head)

The management of penetrating wounds is fairly straightforward. There will be exceptions, but as a rule the damage done to the internal organs (in this case the brain) will need to be repaired surgically. This man will go to the OR, and it will be there, under anesthesia and with full con- trol, that the ax will be removed. An important detail when the weapon is embedded in the patient and part of it is sticking out is not to remove it in the ER or at the scene of the accident.
In the course of a mugging, a man is hit over the head with a blunt instrument. He has a scalp laceration, and skull x-rays show an underlyinglinear skull fracture. He is neurologically intact and gives no history of having lost consciousness.

management?
(2) Trauma - review from head to toe (head)

The rule in skull fractures is that if they are closed (no overlying wound) and asymptomatic, they are left alone. If they are open (like this one), the laceration has to be cleaned and closed, but if not comminuted or depressed, it can be done in the ER.
In the course of a mugging, a man is hit over the head with a blunt instrument. He has a scalp laceration, and the skull x-rays show an underlying comminuted, depressed skull fracture. He is neurologically intact and gives no history of having lost consciousness.
management?
(3) Trauma - review from head to toe (head)

This one goes to the OR for cleaning and repair, and possible craniotomy.
A pedestrian Is hit by a car. When brought to the ER he has minor bruises and lacerations but is otherwise quite well, with a completely normal neurologic exam. However, the ambulance crew reports that he was unconscious at the slte, and although he woke up during the ambulance ride and is now completely lucld, he does not remember how the accident happened.

management?
(4) Trauma - review from head to toe (head)

Anyone who has been hit over the head and has become unconscious gets a computed tomography (CT) scan, looking for intracranial hematomas. If the CT scan and the neurologic exam are normall, he can go home - provided his family is willing to wake him up frequently over the next 24 hours to make sure he is not going into coma.
A car hits a pedestrian. He arrives in the ER in coma. He has…(raccoon eyes… or clear fluid dripping from the nose…or clear fluid dripping from the ear…or ecchymosis behind the ear)…
Dx?
Diagnostic Test?
Tx?
(5-8) Trauma - review from head to toe (head)

Dx: Base of the skull fracture.

Diagnostic Test: CT scan and cervical spine X-Rays.

Tx: needs neurosurgical consult and antibiotics
A 14-year-old boy is hit over the right side of the head with a baseball bat. He loses consciousness for a few minutes, but recovers promptly and continues to play. One hour later he is found unconscious in the locker room. His right pupil is fixed and dilated.
Dx?
How is it diagnosed?
Tx?
(9) Trauma - review from head to toe (head)

Dx: Acute epidural hematoma (probably right side)

Diagnostic Test: CT scan

Treatment: Emergency surgical decompression (craniotomy)

Good prognosis if treated, fatal within hours if it is not.
A 32-year-old male is involved in a head-on, high-speed automobile collision. He is unconscious at the site, regains consciousness briefly during the ambulance ride and arrives at the E.R. in deep coma, with a fixed, dilated right pupil.
Dx?
Diagnostic Test?
Tx?
(10) Trauma - review from head to toe (head)

Dx: Acute Subdural hematoma

Diagnostic Test: CT scan
(Also need to check cervical spine!)

Treatment: Emergency craniotomy

poor prognosis because of brain injury
A man involved in a high-speed, head-on automobile collision is in a coma.He has never had any lateralizing signs, and CT scan shows a small crescent- shaped hematoma, but there is no deviation of the midline structures.

Dx?
Diagnostic Test?
Tx?
(11) Trauma - review from head to toe (head)

Another subdural hematoma, but without lateralizing signs and evidence of displacement of the midline structures, surgery has little to offer. Management will probably be directed at con- trolling ICP, as detailed in the next vignette.
A patient involved in a head-on, high-speed automobile collision arrives in the ER in deep coma, with bilateral fixed dilated pupils. CT scan of the head shows diffuse blurring of the gray-white mass interface and multiple small punctate hemorrhages. There is no single large hematoma or displacement of the midline structures

Dx?
Diagnostic Test?
Tx?
(12) Trauma - review from head to toe (head)

The CT findings are classic for diffuse axonal injury. Prognosis is terrible, and surgery cannot help. Therapy will be directed at preventing further injury from increased ICP. Probably ICP monitoring will be in order. First-line measures to lower ICP include head elevation, hyper- ventilation, and avoidance of fluid overload. Mannitol and furosemide are next in line. Do not overdo the treatment. Lowering ICP is not the ultimate goal; preserving brain perfusion is. Thus, diuretics that lead to systemic hypotension, or measures that produce excessive cerebral vasoconstriction may be counterproductive. Lowering oxygen demand may also help. Sedation has been used for that purpose, and hypothermia is currently advocated for the same reason.
A 77-year-old man becomes “senile” over a period of three or four weeks. He used to be active and managed all of his financial affairs. Now he stares at the wall, barely talks and sleeps most of the day. His daughter recalls that he fell from a horse about a week before the mental changes began.
Dx?
Diagnostic Test?
Tx?
(13) Trauma - review from head to toe (head)

Dx: Chronic subdural hematoma. [DONT CONFUSE WITH ALZHEIMERS!!]

Diagnostic Test: CT scan

Treatment: Surgical decompression (craniotomy)

Spectacular improvement expected
A 45-year-old man is involved in a high-speed automobile collision. He arrives at the ER in coma with fixed, dilated pupils. He has multiple other injuries, including fractures of the extremities. His blood pressure is 70 over 50, with a feeble pulse at a rate of 130. What klnd of intracranial bleeding IS responsible for the low blood pressure (BP) and high pulse rate?

dx/management/ tx?
(14) trauma - review from head to toe (head)

I trust you remember this very same vignette from the review of shock. Shock does not result from intracranial bleeding (not enough room in the head for sufficient blood loss to cause shock). Look for answer of significant blood loss to the outside (could be scalp laceration), or inside (abdomen, pelvic fractures).
A man has been shot in the neck and his blood pressure is rapidly deteriorating.

next steps?
(15) trauma - review from head to toe (neck)

RUSH TO THE OR
A 42-year-old man is shot once with a 22-caliber revolver. The entrance wound is in the anterior left side of the neck, at the level of the thyroid cartilage. X-rays show that the bullet is embedded in the right scalene muscle. He is spitting and coughing blood and has an expanding hematoma under the entrance wound. His blood pressure responded promptly to fluid administration, and he has remained stable.

next steps?
(16) trauma - review from head to toe (neck)

A clear-cut case of a penetrating wound in the middle of the neck (zone 2) that has alarming symptoms and therefore follows the rule (rather than the exception) for all penetrating injuries: immediate surgical exploration is required. This is true even though he is stable. The middle of the neck is packed with structures that should not have holes in them.
A young man is shot in the upper part of the neck. Evaluation of the entrance and exit wounds indicates that the trajectory is all above the level of the angle of the mandible. A steady trickle of blood flows from both wounds, and does not seem to respond to local pressure. The patient is drunk and combative but seems to be otherwise stable.

next steps?
(17) trauma - review from head to toe (neck)

Now we are getting into the exceptions. In this very high level of the neck there is no trachea or esophagus to worry about, but only pharynx- injuries of which are inconsequential. Vascular injuries are the only potential problem, but getting to them surgically is not easy. Thus angiog- raphy is a better choice, both for diagnosis and potentially for embolization.
A young man suffers a gunshot wound to the base of his neck. The entrance and exit wounds are above the clavicles but below the cricoid cartilage. He is hemodynamically stable.

next steps?
(18) trauma - review from head to toe (neck)

Zone 1 - also crammed with vital structures that should be promptly repaired if they are injured. But precise preoperative diagnosis would help plan the incision and surgical approach.

If the patient is stable, the standard workup includes:
angiography
soluble contrast esophagogram (followed by barium if negative)
esophagoscopy
bronchoscopy

This should be done even if the patient is asymptomatic.
In the course of a bar fight, a young man is stabbed once in the neck. The entrance wound is in front of the sternomastoid muscle on the right, at the
level of the thyroid cartilage. The patient is completely asymptomatic, and his vital signs are completely normal.

next steps?
(19) trauma - review from head to toe (neck)

In stab wounds to the upper and middle zones of the neck, completely asymptomatic patients can be safely observed for 12 hours, without expensive workup or surgical exploration.
A patient who was the unbelted right front-seat passenger in a car flies through the windshield when the car crashes into a telephone pole at 30 miles an hour. He arrives in the ER strapped to a headboard and with sandbags on both sides of the neck. He has multiple facial lacerations but is otherwise stable. Examination of the neck reveals persistent pain and tenderness to palpation over the posterior midline of the neck. Neurologic examination is normal.

next steps?
(20) trauma - review from head to toe (neck)

Need to r/o C-spine injury:

(1) check for neuro deficits (more about those later)
(2) if pain over neck, get imaging! - AP & Lateral C-spine films (including T1), as well as odontoid views
(3) If xrays negative & you are still suspicious - do a CT scan.
An 18-year-old street fighter gets stabbed in the back, just to the right side of the midline. He has paralysis and loss of proprioception distal to the injury on the right side, and loss of pain perception distal to the injury on the left side.

dx?
(21) trauma - review from head to toe (spinal cord)

this is a classic spinal cord hemisection, better known as Brown Sequard syndrome.
A patient involved in a car accident sustains a burst fracture of the vertebral bodies. He develops loss of motor function and loss of pain and temperature sensation on both sides distal to the injury, while showing preservation of vibratory sense and position.

dx?
(22) trauma - review from head to toe (spinal cord)

Anterior cord syndrome.
An elderly man is involved in a rear-end automobile collrsron in which he hyperextends his neck. He develops paralysis and burning pain on both upper extremities while maintaining good motor function in his legs.

dx? management?
(23) trauma - review from head to toe (spinal cord)

What is it? Central cord syndrome.
Management for cases 21- 23: Start with the precise diagnosis. X-rays and CT scans are good to look at the cervical bones. To look at the cord, magnetic resonance imaging (MRI) is better. Beyond that, I doubt that the long and complicated tnanagement of spinal cord injuries will be tested on the examination, but one item might show up: there is some suggestion that high-dose corticosteroids soon after a spinal cord injury may help minimize the permanent damage. Pick that answer, if it's offered, for the acute management.
A 75-year-old man slips and falls at home, hitting his right chest wall against the kitchen counter. He has an area of exquisite pain to direct palpation over the seventh rib, at the level of the anterror axillary line. Chest x-ray confirms the presence of a rib fracture, with no other abnormal findings.

dx/ management?
(1) trauma review head to toe (chest)

A plain rib fracture is the most common chest injury. It is bothersome but trivial in most peo- ple, but it can be hazardous in the elderly (splinting and hypoventilation leading to pneumo- nia). The key to treatment is local pain relief, best achieved by nerve block. Beware of the wrong answers that call for strapping or binding.
A 25-year-old man is stabbed in the right chest. He is moderately short of breath, has stable vital signs. No breath sounds on the right. RESONANT to percussion.
Dx?
Diagnostic Test?
Tx (specific)?
(2) trauma review head to toe (chest)

Dx: Plain pneumothorax

Diagnostic Test: There is time to get a chest X-Ray if the option if offered

Treatment: Chest tube to underwater seal and suction, high in the pleural cavity
A 25-year-old man is stabbed in the right chest. He is moderately short of breath, has stale vital signs. No breath sounds on at the base on the right chest, faint distant breath sounds at the apex. DULL to percussion.
Dx?
Diagnostic Test?
Tx?
(3) trauma review head to toe (chest)

Dx: Hemothorax

Diagnostic Test: Chest X-Ray

Treatment: Chest tube on the right, at the base of the pleural cavity
A 25-year-old man is stabbed in the right chest. He is moderately short of breath, has stable vital signs. No breath sounds on at the base on the right chest, faint distant breath sounds at the apex. DULL to percussion. A chest tube placed at the right pleural base recovers 120 cc of blood, drains another 20 cc in the next hour.
Dx?
Further Tx?
(4) trauma review head to toe (chest)

Dx: Hemothorax

Further treatment: The point of this one is that most hemothoraxes do not need exploratory surgery. Bleeding is from lung parenchyma (low pressure), stops by itself. Chest tube is all that is needed.

Key clue: little blood retrieved, even less afterwards
A 25-year-old man is stabbed in the right chest. He is moderately short of breath, has blood pressure is 95 over 70, pulse rate of 100. No breath sounds on at the base on the right chest, faint distant breath sounds at the apex. DULL to percussion. A chest tube placed at the right pleural base recovers 1250 cc of blood…(or it could be only 450 cc at the outset, but followed by another 420 cc in the next hour and so on).
Dx?
Further Tx?
(5-6) trauma review head to toe (chest)

Dx: Hemothorax

Further treatment: The rare exception who is bleeding from a systemic vessel (almost invariably intercostal) will need Thoracotomy to ligate the vessel
A 25-year-old man is stabbed in the right chest. He is moderately short of breath, has stable vital signs. No breath sounds on the right. Resonant to percussion at the apex of the right chest, dull at the base. Chest X-Ray shows one single, large air-fluid level.
Dx?
Tx?
(7) trauma review head to toe (chest)

Dx: Hemo-pneumothorax

Tx: Chest tube, surgery only if bleeding a lot
A worker has been injured at an explosion in a factory. He has multiple cuts and lacerations from flying debris, and he is obviously short of breath. The paramedics at the scene of the accident ascertain that he has a large, flaplike wound in the chest wall, about 5 cm in diameter, and he sucks air through it with every inspiratory effort.
Dx?
Tx?
(8) trauma review head to toe (chest)

The classic sucking chest wound. It needs to be covered to prevent further air intake (Vaseline gauze is ideal), but it must be allowed to let air out. Taping the dressing on three sides, creating a one-way flap (this time in the proper direction) is an option. Once in the hospital, he will need a chest tube.
A 54-year-old lady crashes her car against a telephone pole at high speed. On arrival at the E.R. she is in moderate respiratory distress. She has multiple bruises over the chest, and multiple site of point tenderness over the ribs. X-Rays show multiple rib fractures on both sides. On closer observation it is noted that a segment of the chest wall on the left side caves in when she inhales, and bulges out when she exhales.
Dx?
Next step if she is going to OR?
Next step if not doing well?
(9) trauma review head to toe (chest)

Dx: Flail Chest
(paradoxical breathing)

to OR:
prophylactic Bilateral Chest Tubes
(because she is at high risk to develop tension pneumothorax when under the positive pressure breathing of the anesthetic)

not well:
Intubate and give Positive Pressure ventilation
(Flail chest is usually assoc w/ pulmonary contusion, leading to inadequate respiration from pain)
A 54-year-old lady crashes her car against a telephone pole at high speed. On arrival at the E.R. she is breathing well. She has multiple bruises over the chest and multiple sites of point tenderness over the ribs. X-Rays show multiple rib fractures on both sides, but the lung parenchyma is clear and both lungs are expanded. Two days later her lungs “white out” on X-Rays and she is in respiratory distress.
Dx?
Management? (2 together)
(10) trauma review head to toe (chest)

Dx: Pulmonary contusion.

It does not always show up right away, may become evident one or two days after the trauma.

Management:
1. Fluid restriction (using colloids) and diuretics,
2. Respiratory support:
(intubation, mechanical ventilation and PEEP if needed)
A 33-year-old lady is involved in a high-speed automobile collision. She arrives at the E.R. gasping for breath, cyanotic at the lips, with flaring nostrils. There are bruises over both sides of the chest, and tenderness suggestive of multiple fractured ribs. Blood pressure is 60 over 45. Pulse rate 160, feeble. She has distended neck and forehead veins, is diaphoretic. Left hemithorax has no breath sounds, is tympanic to percussion.
Dx?
Where is the trauma?
Management?
(11) trauma review head to toe (chest)

Dx: Tension Pneumothorax

Where is the penetrating trauma? The fractured ribs can act as a penetrating weapon.

Management: Chest Tube to the left immediately!
A 54-year-old lady crashes her car against a telephone pole at high speed. On arrival at the E.R. she is breathing well. She has multiple bruises over the chest, and is exquisitely tender over the sternum at a point where there is a crunching feeling of crepitation elicited by palpation.
Dx?
Further Tests?
(12) trauma review from head to toe (chest)

Dx: Sternal fracture
(but the point is that she is at high risk for myocardial contusion and for traumatic rupture of the aorta)

Further tests:
Most important:
1. CT scan
2. Transesophageal echo
(or arteriogram looking for aortic rupture)

Also work-up for MI:
1. EKG
2. Cardiac enzymes
A 53-year-old man is involved in a high-speed automobile collision. He has moderate respiratory distress. Physical exam shows no breath sounds over the entire left chest. Percussion is unremarkable. Chest X-Ray shows air fluid levels in the left chest.
Dx?
Management?
(13) trauma review from head to toe (chest)

Dx: Diaphragmatic rupture
(It is always on the left)

Management: Surgical repair
A motorcycle daredevil attempts to jump over the 12 fountains in front of Caesar’s Palace Hotel in Las Vegas. As he leaves the ramp at very high speed his motorcycle turns sideways and he hits the retaining wall at the other end, literally like a rag doll. At the ER he is found to be remarkably stable, although he has multiple extremity fractures. A chest X-Ray shows fracture of the left first rib and widened mediastinum.
Dx?
Diagnostic Test?
Tx?
(14) trauma review from head to toe (chest)

Dx: traumatic rupture of the aorta

(King size trauma, fracture of a hard-to-break bone...it could be first rib, scapula or sternum...and the tell-tale hint of widened mediastinum)

Diagnostic Test: Arteriogram (aortogram)

Treatment: Emergency surgical repair
A 34-year-old lady suffers severe blunt trauma in a car accident. She has multiple injuries to her extremities, has head trauma and has a pneumothorax on the left. Shortly after initial examination it is noted that she is developing progressive subcutaneous emphysema all over her upper chest and lower neck.
Dx?
Test for additional findings?
Diagnostic test?
Tx?
(15) trauma review from head to toe (chest)

Dx: Traumatic rupture of the trachea or major bronchus

Additional findings: Chest X-Ray would confirm the presence of air in the tissues

Diagnostic test: Fiberoptic bronchoscopy
(to confirm diagnosis and level of injury and to secure an airway)

Tx: Surgical repair
A patient who had received a chest tube for a traumatic pneumothorax is noted to be putting out a very large amount of air through the tube (a large arr leak), and his collapsed lung is not expanding.

dx?
(16) trauma review from head to toe (chest)

Another presentation for a major bronchial injury.
A patient who sustained a penetrating injury of the chest has been intubated and placed on a respirator, and a chest tube has been placed in the appropriate pleural cavity. The patient had been hemodynamically stable throughout, but then suddenly goes into cardiac arrest.

dx/management?
(17) trauma review from head to toe (chest)

A typical scenario for air embolism, from an injured bronchus to a nearby injured pulmonary vein, and from there to the left ventricle. Immediate management includes cardiac massage and Trendelenburg position, followed by thoracotomy.
During the performance of a supraclavicular node biopsy under local anesthesia, suddenly a hissing sound is heard, and the patient drops dead.

dx?
(18) trauma review from head to toe (chest)

air embolism.

[Other thoracic calamities, like tension pneumothoraxor continued bleeding, will produce severe deterioration of vital signs -but there will be a sequence from being okay to becoming terribly ill. When vignettes give you sudden death, think of air embolism.]
A patient who is receiving total parenteral nutrition through a central venous line becomes frustrated because the nurses are not answering his call button, so he gets up and out of bed, and disconnects his central line from the IV tubing. With the open catheter dangling, he takes two steps in the direction of the nurses station, and drops dead.

dx?
(19) trauma review from head to toe (chest)

air embolism.

[Other thoracic calamities, like tension pneumothoraxor continued bleeding, will produce severe deterioration of vital signs -but there will be a sequence from being okay to becoming terribly ill. When vignettes give you sudden death, think of air embolism.]
A patient who sustained severe blunt trauma, including multiple fractures of long bones, becomes disoriented about 12 hours after admission. Shortly thereafter he develops petechial rashes in the axillae and neck, fever, and tachycardia. Afew hours later he has a full-blown picture of respiratory distress with hypoxemia. Chest x-ray shows bilateral patchy infiltrates, and his platelet count is low.

dx/management?
(20) trauma review from head to toe (chest)

dx: fat embolism syndrome

This is not a chest injury, but it is included here because its main problem is respiratory distress. You probably recognized already the fat embolism syndrome. It is not clear how specific is the laboratory finding of fat droplets in the urine, but it does not matter: the mainstay of therapy is respirator support- which would be needed regardless of the etiology of the respiratory distress-
Heparin, steroids, alcohol, and low-molecular-weight dextran have all been used, but are of questionable value.
A 19 year old gang member is shot in the abdomen with a 38 caliber revolver. The entry wound is in the epigastrium, to the left of the midline. The bullet is lodged in the psoas muscle on the right. He is hemodynamically stable, the abdomen is moderately tender.
Management (specific)?
(21) Abdominal Trauma

Management:
A penetrating wound of the abdomen gets exploratory laparotomy every time.

preparations prior to surgery:
an indwelling bladder catheter, a big bore venous line for fluid administration and a dose of broad spectrum antibiotics.
A 19 year old gang member is shot in the abdomen with a 38 caliber revolver. Examination shows clean, punched-out entrance and exit wounds in the transverse colon.

whats the management?
(22) Abdominal Trauma

Technical details of what to do at surgery are not going to be asked in the USMLE, but if a prevailing view dominates surgical thinking - and that view is a recent departure from older, longheld dogma-- there may be a temptation to ask. In this case, it used to be thought that injuries of the colon ALWAYS needed a colostomy, but now everybody agrees that primary repair is okay.
A 19 year old gang member is shot once with a 38 caliber revolver. The entry wound is in the left mid-clavicular line, two inches below the nipple. The bullet is lodged in the left paraspinal muscles. He is hemodynamically stable, but he is drunk and combative and physical exam is difficult to do.
Management?
(23) Abdominal Trauma

Management:
The point here is to remind you of the boundaries of the abdomen. Although this sounds like a chest wound, it is also abdominal. The belly begins at the nipple line. The chest does not end at the nipple line, though. Belly and chest are not stacked up like pancakes, they are separated by a dome. This fellow needs all the stuff for a penetrating chest wound (chest X-Ray, chest tube if needed), plus the exploratory lap
A 42-year-old man is stabbed in the belly by a jealous lover. The wound is lateral to the umbilicus, on the left, and omentum can be seen protruding through it.

whats the management?
(24) Abdominal Trauma

The general rule is that penetrating abdominal wounds get a laparotomy. That is true for gun- shot wounds, but it is also true for stab wounds if it is clear that peritoneal penetration took place.
Inthecourseofadomesticfight,a38-year-oldobesewomanisattackedwith a 4-inch-long switchblade. In addition to several superficial lacerations, she was stabbed in the abdomen. She is hemodynamically stable, and does not have any signs of peritoneal irritation.
whats the management?
(25) Abdominal Trauma

This is probably the only exception to the rule that penetrating abdominal wounds have to be surgically explored - and that is because this in fact may not be penetrating at all! (The blade was short, the woman is well padded.) Digital exploration of the wound tract in the ER may show that no abdominal surgery is needed.
A 31 year old lady smashes her car against a wall. She has multiple injuries including upper and lower extremity fractures. Her blood pressure is 75 over 55, with a pulse rate of 110. On physical exam she has a tender abdomen, with guarding and rebound on all quadrants.
Dx?
Management?
(26) Abdominal Trauma

Dx: Blood (and possible feces) in the belly

Management: Exploratory lap
A 31 year old lady smashes her car against a wall. Her abdomen is tender with guarding and rebound tenderness present in all quadrants
Dx?
Management?
(27) Abdominal Trauma

Dx: Ruptured bowel

Management: Exploratory lap, and repair of the injuries
A 26-year-old lady has been involved in a car wreck. She has fractures in upper extremities, facial lacerations and no other obvious injuries. Chest X-Ray is normal. Shortly thereafter she develops hypotension, tachycardia and dropping hematocrit. Her CVP is low.
Dx?
Diagnostic Test if stable?
Unstable? (2 possible)
Tx?
(28) Abdominal Trauma

Dx: Abdominal bleed

Diagnostic test:
Patient is stable: CT scan
Unstable:
1. Diagnostic Peritoneal Lavage
or
2. Ultrasound in ER

Tx: Exploratory Lap
A 27 year old intoxicated man smashes his car against a tree. He is tender over the left lower chest wall. Chest X-Ray shows fractures of the 8th, 9th and 10th ribs on the left. He has a blood pressure of 85 over 68 and a pulse rate of 128.
Dx?
Diagnostic test if stable?
Diagnostic test if crashing? (2)
Tx?
(29) Abdominal Trauma

Dx: Ruptured spleen

Management if Stable: CT Scan
(if he responds promptly to fluid administration, and does not require blood; further management in that case may well be continued observation with serial CT scans)

Management if “crashing”: Peritoneal Lavage or Sonogram followed by (Tx)Exploratory Laparotomy
A 27 year old intoxicated man smashes his car against a tree. He is tender over the left lower chest wall. Chest X-Ray shows fractures of the 8th, 9th and 10th ribs on the left. He has a blood pressure of 85 over 68 and a pulse rate of 128, which do not respond satisfactorily to fluid and blood administration. He has a positive peritoneal lavage and an exploratory laparotomy where a ruptured spleen is found and it is not salvagable.
Further Management?
(30) Abdominal Trauma

Further Management:

administration of Pneumovax and some would also Immunize for Hemophilus Influenza B and Meningococcus
A multiple trauma patlent is receiving massive blood transfusions as the surgeons are attempting to repair many intraabdominal injuries. It is then noted that blood is oozing from all dissected raw surfaces, as well as from his IV line sites. His core temperature is normal.

management?
(31) Abdominal trauma

Signs of coagulopathy in this setting require a shotgun approach to treatment. Empiric admin- istration of both fresh-frozen plasma and platelet packs is recommended.
During the course of a laparotomy for multiple trauma the patient develops a significant coagulopathy, a core temperature below 34°C, and refractory acidosis.

management?
(32) Abdominal trauma

This combination of hypothermia, coagulopathy, and acidosis requires that the abdomen be closed immediately and that no further operating be done (not even a formal abdominal clo- sure). The standard approach is to pack all bleeding surfaces and close the abdomen temporar- ily with towel clips
An exploratory laparotomy for multiple intraabdominal injuries has lasted 3.5 hours, during which time multiple blood transfusions have been given, and several liters of Ringer lactate have been infused. When the surgeons are ready to close the abdomen they find that the abdominal wall edges cannot be pulled together without undue tension. Both the belly wall and the abdominal contents seem to be swollen.

management?
(33) Abdominal trauma

This is the so-called abdominal compartment syndrome. All the fluid that has been infused has kept the patient alive, but at the expense of creating a lot of edema in the operative area. Forced closure would produce all kinds of problems. The bowel cannot be left exposed to the outside either, so the standard approach is to close the wound with an absorbable mesh over which for- mal closure can be done later, or with a nonabsorbable plastic cover that will be removed later.
In the first postoperative day, a trauma patient develops a very tense and distended abdomen, and the retention sutures are cutting through the abdominal wall. He also develops hypoxia and renal failure.

management?
(34) Abdominal trauma

This is also the abdominal compartment syndrome that was not obvious at the end of the oper- ation, but has developed thereafter. The abdomen will have to be decompressed by opening the incision and using a temporary cover as described above.
In a rollover car accident, a 42-year-old woman is thrown out of the car, and subsequently the car lands on her and crushes her. At evaluation in the ER it is determined that she has a pelvic fracture. She arrived hypotensive, but responded promptly to fluid administration. CT scan shows no intraabdominal bleeding, and a pelvic hematoma.

management?
(35) Pelvic Fractures

Nonexpanding pelvic hematomas in a patient who has become hemodynamically stable are left alone. Depending on the type of fracture, the orthopedic surgeons may eventually do some- thing to stabilize the pelvis, but at this time the main issue is to rule out the potential associat- ed pelvic injuries: rectum, bladder, and vagina. Physical examination and a Foley catheter will do it
In a rollover car accident, a 42-year-old woman is thrown out of the car, and subsequently the car lands on her and crushes her. At evaluation in the ER it is determined by physical examination that she has a pelvic fracture. She arrived hypotensive and did not respond to fluid resuscitation. Hemodynamic parameters have continued to deteriorate. Sonogram performed at the ER shows no intraabdominal bleeding.

management?
(36) Pelvic Fractures
A tough situation. People can bleed to death from pelvic fractures, and thus it seems that we ought to do something about it. But that is easier said than done. Surgical exploration is not the answer; these injuries are typically not in the surgical field afforded by a laparotomy. Several steps have been proposed, but there is no universal agreement as to their effectiveness.External fixation is considered the right answer by many, whereas others think that arteriographic embolization is the way to go (it is effective for arterial bleeding, but not for venous hemorrhage).
A young man is shot point blank in the lower abdomen, just above the pubis. He has blood in the urine, and no evidence of rectal injury.

dx/management?
(37) Urologic Injuries

The hallmark of urologic injuries is blood in the urine after trauma. These two are clear-cut. The therapy is also clear. Penetrating urologic injuries are like most penetrating injuries else- where: they need surgical repair.
A woman is shot in the flank, and when a Foley catheter was inserted in ER, the urine was found to be grossly bloody.

dx/management?
(38) Urologic Injuries

The hallmark of urologic injuries is blood in the urine after trauma. These two are clear-cut. The therapy is also clear. Penetrating urologic injuries are like most penetrating injuries else- where: they need surgical repair.
A patient involved in a high speed automobile collision has multiple injuries, including a pelvic fracture. On physical exam there is blood in the meatus.
Dx? (2 possible)
Diagnostic test?
(39) Urologic Injuries

Dx: Bladder or Urethral injury
(pelvic fracture plus blood in the meatus)

Diagnostic test: Retrograde Urethrogram
(because urethral injury would be compounded by insertion of a Foley catheter)
A 19 year old male is involved in a severe automobile accident. Among many other injuries he has a pelvic fracture. He has blood in the meatus, scrotal hematoma and the sensation that he wants to urinate but can not do it. Rectal exam shows a “high riding prostate”.
Dx?
Diagnostic Test?
Management?
(40) Urologic Injuries

Dx: Posterior Urethral injury.

Diagnostic test: Retrograde Urethrogram

Management:
Suprapubic catheter
(and the repair is delayed 6 months)
A 19 year old male is involved in a motorcycle accident. Among many other injuries he has a pelvic fracture. He has blood in the meatus and scrotal hematoma. Retrograde urethrogram shows an anterior urethral injury.
Management?
(41) Urologic Injuries

Management: Anterior urethral injuries are repaired right away
A 22-year-old man involved in a high-speed
automobde collision has multiple injuries, including a pelvic fracture. At the initial physical examination, blood
is seen at the meatus. insertion of a Foley catheter is attempted, but resistance is met.
(42) Urologic Injuries

Back out! Although the blood at the meatus or the perineal hematoma were not there to warn you, this is also a urethral injury. Do the retrograde urethrogram
A patient involved in a high speed automobile collision has multiple injuries, including a pelvic fracture. Insertion of a Foley catheter shows that there is gross hematuria.
Dx?
Diagnostic test?
(43) Urologic Injuries

Dx: Bladder injury
(Presumably there was no blood in the meatus to warn against the insertion of an indwelling catheter, and since the latter was accomplished without problem, the urethra must be intact)

Diagnostic test: Retrograde Cystogram
A patient involved in a high speed automobile collision has multiple injuries, including rib fractures and abdominal contusions. Insertion of a Foley catheter shows that there is gross hematuria, and retrograde cystogram is normal.
Dx?
Diagnostic Test?
(44) Urologic Injuries

Dx: Kidney injury
(Lower injuries have been ruled out)

Diagnostic test: CT scan

(They will not ask you for fine-judgment surgical decisions, but the rule is that traumatic hematuria does not need surgery even if the kidney is smashed.
They operate only if the renal pedicle is avulsed or the patient is exsanguinating)
A patient involved in a high-speed automobile collision has multiple injuries, including rib fractures and abdominal contusions. Insertion of a Foley catheter shows that there is hematuria, and retrograde cystogram is normal. CT scan shows renal injuries that do not require surgery. Six weeks later the patient develops acute shortness of breath and a flank bruit.

what happened?
(45) Urethral injuries

What is it? This is a weird one, but so fascinating that some medical school professor may not be able to resist the temptation to include it. The patient developed a traumatic arteriovenous fistula at the renal pedicle, and subsequent heart failure.
A 35 year old male is about to be discharged from the hospital where he was under observation for multiple blunt trauma sustained in a car wreck. It is then discovered that he has microscopic hematuria.
Management?
(46) Urologic Injuries

Management: Gross traumatic hematuria in the adult always has to be investigated
A 4 year old falls from his tricycle. In the ensuing evaluation he is found to have microscopic hematuria.
Management?
(47) Urologic Injuries

Management: Microhematuria in kids needs to be investigated, as it often signifies congenital anomalies…particularly if the magnitude of the trauma does not justify the bleeding.
Start with a Sonogram
A 14 year old boy slides down a banister, not realizing that there is a big knob at the end of it. He smashes the scrotum and comes in to the E.R. with a scrotal hematoma the size of a grapefruit.
What should be the physician's concern?
Diagnostic test?
Management?
(48) Urethral injuries

Concern: The issue in scrotal hematomas is whether the testicle is ruptured or not.

Diagnostic test: Sonogram

Management: If ruptured, surgery will be needed. If intact, only symptomatic treatment
A 41 year old male presents to the E.R. reporting that he slipped in the shower and injured his penis. Exam reveals a large penile shaft hematoma with normal appearing glans.
Dx?
Tx?
(49) Urethral injuries

Dx: Fracture of the tunica albuginea
(including the usual cover story given by the patient. These always happen during sexual intercourse with woman on top)

Tx: this is one of the few urological emergencies.
Surgical repair is needed
A 25-year-old man is shot with a 22-caliber revolver. The entrance wound is in the anteriolateral aspect of his thigh, and the bullet is seen by x-rays to be embedded in the muscles, posterolateral to the femur.

management?
(1) trauma to extremities

Apart from the obvious need to fix a bone that might have been shattered by a bullet, the issue in low-velocity gunshot wounds (or stab wounds) of the extremities is the possibility of injury to major vessels. In the first vignette, the anatomy precludes that possibility. Thus that patient only needs cleaning of the wound and tetanus prophylaxis. The bullet can be left where it is.
A 25-year-old man is shot with a 22-caliber revolver. The entrance wound is in the anteromedial aspect of his upper thigh, and the exit wound is in the posterolateral aspect of the thigh. He has normal pulses in the leg, and no hematoma at the entrance site. X-rays show the femur to be intact.

management?
(2) trauma to extremities

the anatomy of the area makes vascular injury very likely, and lack of symptoms does not exclude that possibility. At one time, all of these would have been surgically explored. Nowadays arteriogram is preferred.
A 25-year-old man is shot with a 22-caliber revolver. The entrance wound is in the anteromedral aspect of his upper thigh, and the exit wound is in the posterolateral aspect of the thigh. He has a large, expanding hematoma in the upper, inner thigh. The bone is intact.
management?
(3) trauma to extremities

it is clinically obvious that there is a vascular injury. Surgical exploration is in order. Arteriogram preceding surgical exploration is done only in parts of the body where the very specific site of the vascular injury dictates the use of a particular incision versus another (for instance at the base of the neck and thoracic outlet).
A young man is shot through the arm with a .38-caliber revolver. The path of the bullet goes right across the extremity, from medial to lateral sides. He has a large hematoma in the inner aspect of the arm, no distal pulses, radial nerve palsy, and a shattered humerus.

management?
(4) trauma to extremities

That he will need surgery is clear, but the issue here is what to do first. A very delicate vascular repair, and an even more fragile nerve reanastomosis, would be at risk of disruption when the orthopedic surgeons start manipulating, hammering, and screwing the bone. Thus the usual sequence begins with fracture stabilization, then vascular repair (both artery and vein if possible), and last nerve repair. The unavoidable delay in restoring circulation will make a fasciotomy mandatory.
n a hunting accident, a young man is shot in the leg with a high-powered, big game hunting rifle. He has an entrance wound in the upper outer thigh that is 1 cm in diameter, and an exit wound in the posteromedial aspect of the thigh that is 8 cm in diameter. The femur is shattered.

management?
(5) trauma to extremities

Even though the major vessels are not in the path of this bullet, this young man will need to go to the operating room to have extensive debridement of the injured tissues. High-velocity bullets (military weapons and big-game hunting rifles) produce a cone of destruction.
A 6-year-old girl has her hand, forearm, and lower part of the arm caught in the wringers of an old-fashioned washing machine. The entire upper extremity looks bruised and battered, although pulses are normal and the bones are not broken.

management?
(6) trauma to extremities

Crushing injuries lead to two concerns; the myoglobinemia- myoglobinuria- acute renal failure issue, and the delayed swelling that may lead to a compartment syndrome. For the first, plenty of fluids, osmotic diuretics (mannitol), and alkalinization of the urine help protect the kidney. For the latter, fasciotomy is the answer.
You get a phone call from a frantic mother. Her 7 year old girl spilled Drano all over her arms and legs. You can hear the girl screaming in pain in the background.
Management?
(7) burns

Management:
The point of this question is that chemical injuries – particularly alkalis-need copious, immediate, profuse irrigation.

Instruct the mother to do so right at home with tap water, for at least 30 minutes before rushing the girl to the E.R
While trying to hook up illegally to cable TV, an unfortunate man comes in contact with a high tension electrical power line. He has an entrance burn wound in the upper outer thigh and an exit burn lower down on the same side.
Tx?
What can occur from this event?
Management of this? (3)
(8) burns

Management: Extensive surgical Debridement
(there is deep tissue destruction)

What can occur? Myoglobinemia
(leading to myoglobinuria and to Renal Failure)

Management:
1. lots of IV fluids,
2. Osmotic Diuretics (Mannitol),
3. Alkalinization of the urine
A man is rescued by firemen from a burning building. On admission it is noted that he has burns around the mouth and nose, and the inside of his mouth and throat look like the inside of a chimney.
Dx?
Diagnostic Test?
Management?
(9) burns

Dx: Inhalation burns

Diagnostic test: Bronchoscopy

Management: Respiratory support
A patient has suffered third degree burns to both of his arms when his shirt caught on fire while lighting the back yard barbecue. The burned areas are dry, white, leatherly anesthetic, and circumferential all around arms and forearms.
What is main problem?
Management? (2)
(10) burns

Problem: Circumferential burns
(The leatherly eschar will not expand, while the are under the burn will develop massive edema, thus circulation will be cut off or in the case of circumferential burns of the chest, breathing will be compromised)

Management: Compulsive monitoring of peripheral pulses and capillary filling.
Escharotomies at the bedside at the first sign of compromised circulation
A toddler is brought to the E.R. with burns on both of his buttocks. The areas are moist, have blisters and are exquisitely painful to touch. The story is that the kid accidentally pulled a pot of boiling water over himself.
what type of burn?
What should the physician question?
Management? (2)
(11) burns

Dx: Second degree burn
(Note that in kids third degree is deep bright red, rather than white leatherly as in the adult)

Question: How did it really happen? Burns in kids always bring up the possibility of child abuse, particularly if they have the distribution that you would expect if you grabbed the kid by arms and legs and dunked him in a pot of boiling water.

Management: Silvadene cream.
Possibly reporting to authorities for child abuse
An adult male who weight “X” Kgs. Sustains second and third degree burns over ---whatever--- The burns will be depicted in a drawing, indicating what is second degree (moist, blisters, painful) and what is third degree (white, leatherly, anesthetic).
What is the equation for proper fluid resuscitation management?
What fluid and how much in first 8 hours?
(12-14) burns

Management:
4cc per Kg. of body weight per percentage of burned area
(up to 50%)

(if pt is 70kg and 18% burned, then 70x4x18)

Fluid: Ringers Lactate

(half of the calculated dose goes in during first 8 hours)
After suitable calculations have been made, a 70-kg adult with extensive third- degree burns is receiving Ringer lactate at the calculated rate. In the first 3 hours his urinary output is 15, 22, and 18 ml.

Are IVF at an adequate rate?
(15) burns

Most experts aim for an hourly urinary output of at least 0.5 rnllkg, or preferably 1 rnllkg body weight per hour. For patients with electrical burns the flow should be even higher (1 to 2 rnllkg per hour); thus by any criteria this fellow needs more fluid.
After suitable calculations have been made, a 70-kg adult with extensive third- degree burns is receiving Ringer lactate at the calculated rate. In the first 3 hours his urinary output is 325, 240, and 270 ml.

Are IVF at an adequate rate?
(16) burns

The opposite of the previous vignette. Somebody is trying to drown this poor guy. The calculation was too generous; the rate of administration has to be scaled back.
During the first 48 hours after a major burn, a 70-kg patient received vigorous fluid resuscrtation and maintained a urinary output between 45 and 110 ml/h. On the third postburn day- after IV flutds have been discontinued- his urinary output reaches 270 to 350 ml/h.

Are IVF at an adequate rate?
(17) burns

This is the expected. Fluid is coming back from the burn area into the circulation. He does not need more IV fluids to replace these losses
An 8-month-old baby who weighs x kilograms is burned over...areas (depicted in a front-and-back drawing). Second-degree burn will look the same as in the adult; third-degree burns will look deep bright red.

how do child burn calculations differ from adult?
(18) burns

In babies the head is bigger and the legs are smaller, thus the head has two 9%s, whereas both legs add up to three (rather than four) 9%

Proportionally fluid needs are greater in children than in adults; thus, if asked for the rate in the first hour, it should be 20 rnllkg, and if asked for 24-hour calculations, the formula calls for 4 to 6 ml/kg/%.
A patient with second- and third-degree burns over 65% of his body surface is undergoing proper fluid resuscitation. The question asks about management for the burned areas, and other supportive care?
(19) burns

First of all, tetanus prophylaxis. Then suitable cleaning, and use of topical agents. The standard one is silver sulfadiazine. If deep penetration is desired (thick eschar, cartilage), mafenide acetate is the choice (do not use everywhere; it hurts and can produce acidosis). Burns near the eyes are covered with triple antibiotic ointment. Pain medication is given IV. After about 2 or 3 weeks, grafts will be done to the areas that did not regenerate. After an initial day or two of NG suction, intensive nutritional support is needed (via the gut, high calorielhigh nitrogen). Rehabilitation starts on day one.
A 42 year old lady drops her hot iron on her lap while doing the laundry. She comes in with the shape of the iron clearly delineated on her upper thigh. The area is white, dry, leatherly, anesthetic.
Tx?
(20) burns

Tx: Early excision and skin grafting
(in very small third degree burns)
A 6-year-old child tries to pet a domestic dog while the dog is eating, and the child is bitten by the dog

management?
(21) Bites & Stings

This is considered a provoked attack, and as far as rabies is concerned only observation of the pet is required (for development of signs of rabies). Tetanus prophylaxis and standard wound care is all that is needed for the child.
During a hunting trip, a young man is bitten by a coyote. The animal is captured and brought to the authorities alive.

management?
(22) Bites & Stings

Observation of a wild animal for behavioral signs of rabies is impractical. But having the ani- mal mailable will allow it to be killed and the brain examined for signs of rabies, thus hopeful- ly sparing the hunter the necessity of getting vaccinated.
While exploring caves in the Texas hill country, a young man is bitten by bats (that promptly fly away).

management?
(23) Bites & Stings

Now we do not have the animal to examine. Rabies prophylaxis is mandatory (immunoglobulin + vaccine).
Duringahuntingtr~pahunterisbitteninthelegbyasnake.Hiscompanion, who is an expert outdoorsman, reports that the snake had elliptical eyes, pits behind the nostrils, big fangs, and rattlers in the tail. The patient arrives at the hospital 1 hour after the bite took place. Physical examination shows two fang marks about 2 cm apart, and there is no local pain, swelling, or discoloration.

management?
(24) Bites & Stings

The description of the snake is indeed that of a poisonous rattlesnake, but even when bitten by a poisonous snake, up to 30% of patients are not envenomated. The most reliable signs of envenomation are excruciating local pain, swelling, and discoloration (usually fully developed
-
within half an hour) all that is needed, plus the standard wound care (including tetanus prophylaxis).
During a hunting trip, a hunter is bitten in the leg by a snake. His companion, who is an expert outdoorsman, reports that the snake had elliptical eyes, pits behind the nostrils, big fangs, and rattlers in the tail. The patient arrives at the hospital 1 hour after the bite took place. Physical examination shows two fang marks about 2 cm apart, as well as local edema and ecchymotic discoloration. The area is very painful and tender to palpation.

management?
(25) Bites & Stings

This fellow is envenomated. Blood should be drawn for typing and crossmatch, coagulation studies, and renal and liver function. The mainstay of therapy is antivenin, of which several vials have to be given (figure at least 5, but if the description of the local site and systemic symptoms are alarming, pick 10, or even 20). Surgical excision of the bite site and fasciotomy are only needed in extremely severe cases.
While playing in the backyard of her south Texas home, a 6-year-old girl is bitten by a rattlesnake. At the time of hospital admission she has severe signs of envenomation.

management?
(26) Bites & Stings

The point of this vignette is to remind you that snake antivenin is one of the very few medicines for which the dose is not calculated on the basis of the size of the patient. The dose of antivenin depends on the amount of venom injected, regardless of the size and age of the victim. This lit- tle girl may need 10 vials, or even more if she is very sick
During a picnic outing, a young girl inadvertently bumps into a beehive and is stung repeatedly by angry bees. She is seen 20 mmutes later and found to be wheezing, hypotensive, and madly scratching an urticarial rash.

management?
(27) Bites & Stings

Epinephrine is the drug of choice (0.3 to 0.5 rnl of 1:1000 solution). The stingers have to be carefully removed.
While rummaging around her attic, a lady is bitten by a spider that she describes as black, with a red hourglass mark in her belly. The patient has nausea and vomiting and severe generalized muscle cramps.

management?
(28) Bites & Stings

Black widow spider bite. The antidote is IV calcium gluconate. Muscle relaxants also help.
A patient seeks help for a very painful ulceration that he discovered in his forearm on arising this morning. Yesterday he spent several hours cleaning up the attic, and he thinks he may have been "bitten by a bug." The ulcer is 1 cm in diameter, with a necrotic center with a surrounding halo of erythema.

management?
(29) Bites & Stings

Probably a brown recluse spider bite. Dabsone will help. Local excision and skin grafting may be needed, but do not do until the full extent of the damage is obvious (up to one week)
A 22 year old gang leader comes to the E.R. with a small, 1 cm. deep sharp cut over the knuckle of the right middle finger. He says he cut himself with a screwdriver while fixing his car.
Dx?
Management?
(30) Bites & Stings

Dx: The description is classical for a human bite. No, nobody actually bit him, he did it by punching someone in the mouth...and getting cut with the teeth that were smashed by his fist. The imaginative cover story usually comes with this kind of lesion.

Management: human bites are bacteriological the dirtiest that one can get. Rabies shots will not be needed, but surgical exploration by an orthopedic surgeon will be required.
In the newborn nursery it is noted that a child has uneven gluteal folds. Physical exam of the hips reveals that one of them can be easily dislocated posteriorly with a jerk and a “click”, and returned to normal position with a “snapping”.
Dx?
Management?
(1) Ortho - hip

Dx: Developmental Dysplasia of the hip

Management: Abduction splinting

(Don’t order X-Rays in a newborn. Calcification is still incomplete and you will not see anything)
A 6 year old boy has insidious development of limping with decreased hip motion. He complains occasionally of knee pain on that side. He walks into the office with an antalgic gait. Passive motion of the hip is guarded.
Dx?
Diagnostic test?
Management?
(2) Ortho - hip

Dx: Legg-Perthes disease
(avascular necrosis of the capital femoral epiphysis)

[Remember that hip pathology can show up with knee pain]

Diagnostic test: AP and lateral X-Rays for diagnosis

Management: Contain the femoral head within the acetabulum by casting and crutches
A 13 year old boy complains of pain in the groin (it could be the knee) and is noted by the family to be limping. He sits in the office with the foot on the affected side rotated towards the other foot. Physical examination is normal for the knee, but shows limited hip motion. As the hip is flexed, the leg goes into external rotation and it can not be rotated internally
Dx?
Diagnostic test?
Tx?
(3) Ortho - hip

Dx: Slipped Capital Femoral Epiphysis
(Forget the details: a bad hip in this age group is slipped capital femoral epiphysis, an orthopedic emergency)

Diagnostic test: AP and lateral X-Rays

Tx: The orthopedic surgeons will pin the femoral head in place
A little toddler has had the flu for several days, but he was walking around fine until about two days ago. He now absolutely refuses to move one of his legs. He is in pain, holds the leg with the hip flexed, in slight abduction and external rotation and you can not examine that hip he will not let you move it. He has elevated sedimentation rate
Dx?
Management? (2 steps)
(4) Ortho - hip

Dx: Septic Hip
(orthopedic emergency)

Management:
1. Under general anesthesia the hip is aspirated to confirm the diagnosis, and

2. Open arthrotomy is done for drainage
A child with a febrile illness but no history of trauma has persistent, severe localized pain in a bone.
Dx?
Diagnostic test?
(5) Ortho - other

Dx: Acute Hematogenous Osteomyelitis

Diagnostic test: Bone Scan
(don’t fall for the X-Ray option. X-Ray will not show anything for two weeks)
A 2-year-old child is brought in by concerned parents because he is bowlegged.
Dx?
Management?
(6) Ortho - other

Genu varum (bow-leg) is normal up to the age of 3. Genu valgus (knock-knee) is normal between the ages of 4 and 8. Thus neither of these children need therapy. Should the varum deformity (bow-legs) persist beyond its normal age range (i.e., beyond age 3), Blount disease is the most common problem (a disturbance of the medial proximal tibial growth plate), and surgery can be performed for it.
A 5-year-old child is brought in by concerned parents because he is knock- kneed.
Dx?
Management?
(7) Ortho - other

Genu varum (bow-leg) is normal up to the age of 3. Genu valgus (knock-knee) is normal between the ages of 4 and 8. Thus neither of these children need therapy. Should the varum deformity (bow-legs) persist beyond its normal age range (i.e., beyond age 3), Blount disease is the most common problem (a disturbance of the medial proximal tibial growth plate), and surgery can be performed for it.
A 14-year-old boy says he injured his knee while playing football. Although there is no swelling of the knee joint, he complains of persistent pain right over the tibial tubercle, which is aggravated by contraction of the quadriceps. Physical examination shows localized tenderness right over the tibial tubercle
Dx?
Management?
(8) Ortho - other

This is another one with a fancy name: Osgood-Schlatter disease (osteochondrosis of the tibia1 tubercle). It is usually treated with immobilization of the knee in an extension or cylinder cast for 4 to 6 weeks.
A baby boy is born with both feet turned inward. Physical examination shows that there is plantar flexion of the ankle, inversion of the foot, adduction of the forefoot, and internal rotation of the tibia.
Dx?
Management?
(9) Ortho - other

This is the complex deformity known as club foot (fancy name: talipes equinovarus). The child needs serial plaster casts started in the neonatal period. The sequence of correction starts with the adducted forefoot, then the hindfoot varus, and finally the equinus. About half the patients respond completely and need no surgery. Those who require surgery are operated on after the age of 6-8 months, but before 1-2 years of age.
A 12 year old girl is referred by the school nurse because of potential scoliosis. The thoracic spine is curved toward the right, and when the girl bends forward a “hump” is noted over her right thorax. The patient has not yet started to menstruate.
Management? (3 steps)
(10) Ortho - other

Management:
1. Baseline x-rays to monitor progression
2. Bracing may be needed to arrest progression
3. Pulmonary function could be limited if there is large deformity

(The point is that scoliosis may progress until skeletal maturity is reached. At the onset of menses skeletal maturity is about 80%, so this patient still has a way to go)
A 4-year-old falls down the stairs and fractures his humerus. He is placed in a cast at the nearby "doc ~nthe box," and he is seen by his regular pediatrician two days later. At that time he seems to be doing fine, but AP and lateral x-rays show significant angulation of the broken bone.
Management?
(11) Ortho- Fractures

Nothing else is needed. Children have such tremendous ability to heal and remodel broken bones that almost any reasonable alignment and immobilization will end up with a good result.
In fact, fractures in children are no big deal - with a few exceptions that are illustrated in the next few vignettes.
An 8-year-old boy falls on his right hand with the arm extended, and he breaks his elbow by hyperextension. X-rays show a supracondylar fracture of the humerus. The distal fragment is displaced posteriorly.

Management?
(12) Ortho- Fractures

This type of fracture is common in children, but it is important because it may produce vascular or nerve injuries - or both- and end up with a Volkmann contracture. Although it can usually be treated with appropriate casting or traction (and rarely needs surgery), the answer revolves around careful monitoring of vascular and nerve integrity, and vigilance regarding development of a compartment syndrome.
A child sustains a fracture of a long bone, involving the epiphyses and growth plate. The epiphyses and growth plate are laterally displaced from the metaphyses, but they are in one piece, i.e., the fracture does not cross the epiphyses or growth plate and does not involve the joint.
Management?
(13) Ortho- Fractures

even though the dreaded growth plate is involved, it has not been divided by the fracture. Treatment by closed reduction is sufficient.
A child sustains a fracture of a long bone that extends through the joint, the epiphyses, the growth plate, and a piece of the metaphyses.
Management?
(14) Ortho- Fractures

there are two pieces of growth plate. Unless they are very precisely aligned, growth will be disturbed. Open reduction and internal fixation will be needed.
A 16 year old boy complains of low grade but constant pain in his distal femur present for several months. He has local tenderness in the area, but is otherwise asymptomatic. X-Rays show a large bone tumor, with “sunburst” pattern and periosteal “onion skinning”.
Dx? (2 possible)
Management?
(15-16) Ortho- tumors

Dx: Osteogenic Sarcoma or Ewing’s Sarcoma

Management: The point of the vignette is that you do not mess with these. Do not attempt biopsy. Referral is needed, not just to an orthopedic surgeon (they see one of these every three years), but to a specialist on bone tumors
A 66 year old lady picks up a bag of groceries and her arm snaps broken
Dx?
Diagnostic test? (3 steps)
(17) Ortho- adults

Dx: A pathologic fracture (i.e: for trivial reasons) means bone tumor, which in the vast majority of cases will be metastatic.

Diagnostic test:
1. Get X-Rays to diagnose this particular broken bone,
2. whole body Bone Scans to identify other mets,
3. start looking for the primary cancer site
(In women, breast. In men, prostate. In heavy smokers, lung...and so on)
A 60yo M c/o fatigue and pain at specific places on several bones. He is found to be anemic, and x-rays show multiple punched out lytic lesions throughout the skeleton
Dx?
(18) Ortho- adults

Multiple lytic lesions in an old anemic man suggest multiple myeloma. The only bone tumor in which x-rays are a better diagnostic tool than bone scan, and one with a whole constellation of other diagnostic tests: Bence-Jones protein in the urine and abnormal immunoglobulins in the blood. The latter are detectable by serum electrophoresis and better yet by immunoelectrophoresis.
A 58 year old lady has a soft tissue tumor in her thigh. It has been growing steadily for six months, it is located deep into the thigh, is firm, fixed to surrounding structures and measures about 8cm in diameter
Dx?
Diagnostic test?
(19) Ortho- adults

Dx: Soft tissue sarcoma is the concern

Diagnositic test: MRI
(Leave biopsy and further management to the experts)
A man who fell from a second floor window has clinical evidence of fracture of his femur. The vignette gives you a choice of X-Rays to order.
What are the rules for ordering x-rays? (3)
(20) Ortho- general

Here are the rules:

1. Always get X-Rays at 90 degrees to each other (for instance, AP and lateral)

2. Always include the joints above and below

3. if appropriate (this case is) check the other bones that might be in the same line of force (here the lumbar spine)
While playing football, a college student fractures his clavicle. The point of tenderness is at the junction of the middle and distal thirds of the clavicle.
tx?
(21) General Ortho

Treat it with a figure-of-eight device for 4 to 6 weeks.
A 55 year old lady falls in the shower and hurts her right shoulder. She shows up in the ER with her arm held close to her body, but rotated outwards as if she were going to shake hands. She is in pain and will not move the arm from that position. There is numbness in a small area of her shoulder, over the deltoid muscle.
Dx?
Diagnostic test?
Tx?
(22) General Ortho

Dx: Anterior Dislocation of the Shoulder, with Axillary nerve damage

Diagnostic test: Get AP and lateral X-Rays

Tx: Reduce
After a grand mal seizure, a 32 year old epileptic notices pain in her right shoulder and she can not move it. She goes to the near-by “Doc in a Box”, where she has X-Rays and is diagnosed as having a sprain and given pain medication. The next day she still has the same pain and inability to move the arm. She comes to the ER with the arm held close to her body, in a “normal” (i.e., not externally rotated, but internally rotated) position
Dx?
Diagnostic test? (specific)
(23) General Ortho

Dx: Posterior Dislocation of the Shoulder
(Very easy to miss on regular X-Rays)

Diagnostic test: Get X-Rays again but order Axillary view or Scapular Lateral
An elderly woman with osteoporosis falls on her outstretched hand. She comes in with a deformed and painful wrist that looks like a "dinner fork." X- rays show a dorsally displaced, dorsally angulated fracture of the distal radius and small, nondisplaced fracture of the ulnar stylus.
tx?
(24) General Ortho

The famous Colles fracture. It is treated with close reduction and long arm cast.
During a rowdy demonstration and police crackdown, a young man is hit with a nightstick on his outer forearm that he had raised to protect himself. He is found to have a diaphyseal fracture of the proximal ulna, with anterior dislocation of the radial head
tx?
(25) General Ortho

Another classic with a fancy name: Monteggia fracture. He needs close reduction of the radial head, and open reduction and internal fixation of the ulnar fracture.
Another victim of the same melee has a fracture of the distal third of the radius and dorsal dislocation of the distal radioulnar joint.
tx?
(26) General Ortho

This one is Galeazzi fracture, which is quite similar to the previous one in terms of the resultant instability. This one is treated with open reduction and fixation of the radius, and casting of the forearm in supination to reduce the dislocated joint.
A young adult falls on an outstretchedhand and comes in complaining of wrist pain. On physical examination, he is distinctly tender to palpation over the anatomic snuff-box. AP and lateral x-rays are read as negative.
tx?
(27) General Ortho

Another classic (blissfullydevoid of eponym). This is a fracture of the scaphoid bone (carpal navicular). They are notorious because x-rays will not show them for 2 or 3 weeks, and they are also infamous because of a high rate of nonunion. The history and physical findings (the ten- derness in the snuff-box) are sufficient to indicate the use of a thumb spica cast, with repeat x- rays 3 weeks later.
Ayoungadultfallsonanoutstretchedhandandcomesincomplainingofwrist pain. On physical examination, he is distinctly tender to palpation over the anatomic snuff-box. AP, lateral, and oblique x-rays show a displaced and angulated fracture of the scaphoid.
tx?
(28) General Ortho

Displaced and angulated; will need open reduction and internal fixation.
During a bar room fight, a young man throws a punch at somebody, but misses and ends up hitting the wall. He comes in with a swollen and tender right hand. X-rays show fracture of the fourth and fifth metacarpal necks.
tx?
(29) General Ortho

Metacarpal necks, typically the fourth or the fifth (or both), take the brunt of your anger when you try to hit somebody but miss. Treatment depends on the degree of angulation, displace- ment, or rotary malalignment. Closed reduction and ulnar gutter splint for the mild ones, Kirschner-wireor plate fixation for the bad ones.
A 77 year old man falls in the nursing home and hurts his hip. X-Rays show that he has a displaced femoral neck fracture
Dx?
Tx?
(30) General Ortho

Dx: Hip fracture

Tx: Metal prosthetic surgery
(The point of this vignette is that blood supply to the femoral head is compromised in this setting and the patient is better off with a metal prosthesis put in, rather than an attempt at fixing the bone. With intertrochanteric fractures on the other hand, the broken bones can be pinned together and expected to heal)
A 77-year-old man falls in the nursing home and hurts his hip. He shows up with the affected leg shortened and externally rotated. X-rays show that he has an intertrochanteric fracture.
tx?
(31) General Ortho

These can be fixed with less concern about avascular necrosis. Open reduction and pinning are usually performed. Immobilization in these old people often leads to deep venous thrombosis and pulmonary embolus; thus an additional choice for postoperative anticoagulation may be offered in the question.
The unrestrained front-seat passenger in a car that crashes sustains a closed fracture of the femoral shaft.
tx?
(32) General Ortho

Intramedullary rod fixation is the standard treatment for these.
The unrestrained front-seat passenger in a car that crashes sustains closed comminuted fractures of both femoral shafts. Shortly after admission, he develops a blood pressure of 80 over 50, a pulse rate of 110, and a venous pressure of 0.The rest of his physical examination and x-ray survey (chest, pelvis) are unremarkable, and sonogram of the abdomen done in the ER was likewise negative.
management?
(33) General Ortho
A throwback to the trauma vignettes to remind you that femur fractures may bleed into the tis- sues sufficiently to cause hypovolemic shock. Fixation will diminish the blood loss, and fluid resuscitation and blood transfusions will take care of the shock.
The unrestrained front-seat passenger in a car that crashes sustains closed comminuted fractures of both femoral shafts. Twelve hours after admission, he develops disorientation, fever, and scleral petechia. Dyspnea is evident shortly thereafter, at which time blood gases show a Po, of 60.
dx? management?
(34) General Ortho

Another repeated topic: fat embolism. Respiratory support is the centerpiece of the treatment.
A college student is tackled while playing football, and he develops severe knee pain. When examined shortly thereafter, the knee is swollen, and he has pain on direct palpation over the medial aspect of the knee. With the knee flexed at 30•‹, passive abduction elicits pain in the same area, and the leg can be abducted further out than the normal, contralateral leg (valgus stress test).
dx? management?
(35) General Ortho
The medial collateral ligament is injured. A hinged cast is the usual treatment for either isolated injury. When several ligaments are torn, surgical repair is preferred.
A college student is tackled while playing football, and he develops severe knee pain. When examined shortly thereafter, the knee is swollen, and he has pain on direct palpation over the lateral aspect of the knee. With the knee flexed at 30•‹, passive adduction elicits pain in the same area, and the leg can be adducted further out than the normal, contralateral leg (varus stress test).
dx? management?
(36) General Ortho
Dx: an injury to the lateral collateral ligament.

tx: A hinged cast is the usual treatment for either isolated injury. When several ligaments are torn, surgical repair is preferred.
A college student is tackled while playing football, and he develops severe knee swelling and pain. On physical examination with the knee flexed at 90°, the leg can be pulled anteriorly, like a drawer being opened. A similar finding can be elicited with the knee fixed at 20" by grasping the thigh with one hand, and pulling the leg with the other.
dx? management?
(37) General Ortho
This is a lesion of the anterior cruciate ligament, shown by the anterior drawer test and the Lachman test. Further definition of the extent of internal knee injuries can be done with MRI. Sedentary patients may be treated just with immobilization and rehabilitation, but athletes require arthroscopic reconstruction.
A college athlete injured his knee while playing basketball. He has been to several physicians who have prescribed pain medication and a variety of splints and bandages, but he still has a swollen knee and knee pain. He describes catching and locking that limit his knee motion, and he swears that when his knee is forcefully extended there is a "click" in the joint. He has been told that his x-rays are normal.
dx? management?
(38) General Ortho

Meniscal tears may be difficult to diagnose clinically, but MRI will show them beautifully. Arthroscopic repair is done, trying to save as much of the meniscus as possible. If complete meniscectomy is done, late degenerative arthritis will ensue.
A young recruit complains of localized pain in his tibia after a forced march at boot camp. He is tender to palpation over a very specific point on the bone, but X-Rays are normal
Dx?
Management? (2 steps)
(39) General Ortho

Dx: Stress Fracture
(The lesson here is that stress fractures will not show up radiologically until 2 weeks later)

Management:
1. Treat the guy as if he had a fracture (cast)
2. Repeat the X-Ray in 2 weeks
A pedestrian is hit by a car. Physical examination shows the leg to be angulated midway between the knee and the ankle. X-rays confirm fractures of the shaft of the tibia and fibula.
dx? management?
(40) General Ortho

Casting takes care of the ones that can be easily reduced. Intramedullary nailing is needed for the ones that cannot be aligned.
A pedestrian is hit by a car. Physical examination shows the leg to be angulated midway between the knee and the ankle. X-rays confirm fractures of the shaft of the tibia and fibula. Satisfactory alignment is achieved, and a long leg cast applied. In the ensuing 8 hours the patient complains of increasing pain. When the cast is removed, the pain persists, the muscle compartments feel tight, and there is excruciating pain with passive extension of the toes.
(41) General Ortho

Compartment syndrome is a distinct hazard after fractures of the leg (the forearm and the lower leg are the two places with the highest incidence of compartment syndrome). Fasciotomy is needed here.
An out-of-shape, recently divorced 42-year-old man is trying to impress a young woman by challenging her to a game of tennis. In the middle of the game, a loud "pop" IS heard (like a gunshot), and the man falls to the ground clutching his ankle. He limps off the courts, with pain and swelling in the back of the lower leg, but still able to flex his foot in the plantar position. When he seeks medical help the next day, palpation of his Achilles tendon reveals an obvious defect right beneath the skin.
(42) General Ortho

A classic presentation for rupture of the Achilles tendon. Casting in equinus position will allow healing after several months, or open surgical repair may do it sooner.
While running to catch a bus, an old man twists his ankle and falls on his inverted foot. AP, lateral, and mortise X-rays show displaced fractures of both malleoli.
(43) General Ortho

A very common injury. When the foot is forcefully rotated (in either direction), the talus pushes and breaks one malleolus and pulls off the other one. Open reduction and internal fixation is needed.
A middle aged homeless man is brought to the ER because of very severe pain in his forearm. The history is that he passes out after drinking a bottle of cheap wine and he slept on a park bench for an indeterminate time, probably more than 12 hours. There are no signs of trauma, but the muscles in his forearm are very firm and tender to palpation, and passive motion of his fingers and wrist elicit excruciating pain. Pulses at the wrist are normal
Dx?
Tx?
(44) Ortho emergencies

Dx: Compartment syndrome

Tx: Emergency Fasciotomy
A patient presents to the ER complaining of moderate but persistent pain in his leg under a long leg plaster cast that was applied six hours earlier for an ankle fracture
Management?
(45) Ortho emergencies

Management: Remove the cast

(The point of this vignette is that you never give pain medication and do nothing else for pain under a cast. The cast has to come off right away. It may be too tight, it may be compromising blood supply, it may have rubbed off a piece of skin)
A young man involved in a motorcycle accident has an obvious open fracture of his right thigh. The femur is sticking out through a jagged skin laceration
Management?
(46) Ortho emergencies

Management: Reduction in the OR within 6 hours

(The point of this one is that open fractures are orthopedic emergencies. This fellow may need to have other problems treated first...abdominal bleeding, intracranial hematomas, chest tubes, etc, but the open fracture should be in the OR getting cleaned and reduced within six hours of the injury)
A front seat passenger in a car that had a head-on collision relates that he hit the dashboard with his knees, and complains of pain in the right hip. He lies in the stretcher in the ER with the right extremity shortened, adducted, and internally rotated.
Dx?
Diagnostic test?
Tx?
(47) Ortho emergencies

Dx: Posterior Dislocation of the Hip.
(Emergency: The blood supply of the femoral head is tenuous, and delay in reduction could lead to avascular necrosis)

Diagnostic test: X-Rays

Tx: Emergency reduction
A healthy 24 year old man steps on a rusty nail at the stables where he works as a horse breeder. Three days later he is brought to the ER moribund, with a swollen, dusky foot, in which one can feel gas crepitation.
Dx?
Management? (3 steps: 1 med, 1 surgery, 1 other)
(48) Ortho emergencies

Dx: Gas gangrene

Management:
1. Tons of IV penicillin
2. Immediate surgical debridement of dead tissue
3. followed by a trip to the nearest hyperbaric chamber for hyperbaric O2 treatment
A 48-year-old man breaks his arm when he falls down the stairs. X-rays demonstrate an oblique fracture of the middle to distal thirds of the humerus. Physical examination shows that he cannot dorsiflex (extend) his wrist.
(49) Ortho emergencies

Fractures of the humeral shaft can injure the radial nerve, which courses in a spiral groove right around the posterior aspect of that bone. However, surgical exploration is not usually needed. Hanging arm cast or coaptation splint are used, and the nerve function returns eventually. However, if the nerve was okay when the patient came in, and becomes paralyzed after closed reduction of the bone, the nerve is entrapped and surgery has to be performed.
A football player is hit straight on his right leg and he suffers a posterior dislocation of his knee.
Management? (3 steps)
(50) Ortho emergencies

Management:
1. Check pulses
2. Arteriogram
3. Reduction

(The point here is that posterior dislocation of the knee can nail the popliteal artery. Attention to integrity of pulses, arteriogram and prompt reduction are the key issues)
A window cleaner falls from a third-story scaffold and lands on his feet. Physical examination and x-rays show comminuted fractures of both calcanei.

what other injuries need to be considered following this accident?
(51) Ortho injury patterns

Compression fractures of the thoracic or lumbar spine are the associated, hidden injuries that have to be looked for in this case.
In a head-on automobile collision, the unrestrained front-seat passenger strikes the dashboard and windshield. He comes In with facial lacerations, upper extremity fractures, and blunt trauma to his chest and abdomen.

what other injuries need to be considered following this accident?
(52) Ortho injury patterns

In the confusion of dealing with multiple trauma, less-obvious injuries may be missed. In this particular scenario, as the knees strike the dashboard, the femoral heads may drive backward into the pelvis, or out of the acetabulum.
The unrestrained front-seat passenger in a car that crashes at hlgh speed is brought into the ER with multiple facial fractures and a closed head injury.

what other injuries need to be considered following this accident?
(53) Ortho injury patterns

We have dealt with this one before, but it is worth repeating it. The ultimate hidden injury (because of the devastating complications if missed) is the fracture of the cervical spine. This scenario demands that appropriate x-rays or CT scans be done to rule it out.
A 43-year-old female secretary who does a lot of typing complains about numbness and tingling in her hand, particularly at night. On physical examination, when asked to hang her hand limply in front of her, numbness and tingling are reproduced over the distribution of the median nerve (the radial side 3 112 fingers). The same happens when her median nerve is pressed over the carpal tunnel, or when it is percussed.

dx? management?
(54) Ortho - Hand
Carpal tunnel syndrome is diagnosed clinically, and this vignette is typical. The American Academy of Orthopedic Surgery recommends that wrist x-rays (including carpal tunnel view) be done, primarily to rule out other things. Initial treatment is splints and antiinflammatories. If surgery is needed, electromyographyshould precede it.
A 58-year-old woman describes that she wakes up at night with her right middle finger acutely flexed, and she is unable to extend it. She can do it only by pulling on it with her other hand, at which time she feels a painful "snap.
dx? management?
(55) Ortho - Hand

Trigger finger. Steroid injections are tried first. Surgery is performed if needed.
Ayoungmothercomplainsofpainalongtheradialsideofthewristandthe first dorsal compartment. She relates that the pain is often caused by the position of wrist flexion and simultaneous thumb extension that she assumes to carry the head of her baby. On physical examination the pain is reproduced by asking her to hold her thumb inside her closed fist, and then forcing the wrist into ulnar deviation.

dx? management?
(56) Ortho - Hand

De Quervain tenosynovitis. Splints and antiinflammatories can help, but steroid injection is best. Surgery is rarely needed.
A 72-year-old man of Norwegian ancestry has a contracted hand that can no longer be extended and be placed flat on a table. Palmar fascia1 nodules can be felt.
dx? management?
(57) Ortho - Hand

Dupuytren contracture. Surgery is the only effective treatment.
A 33-year-old carpenter accidentally drives a small nail into the pulp of his index finger, but he pays no attention to the injury at the time. Two days later he shows up in the ER, with throbbing pulp pain, fever, and all the signs of an abscess within the pulp of the affected finger.
dx? management?
(58) Ortho - Hand

This kind of abscess is called a felon, and like all abscesses it has to be drained. But there is a cer- tain urgency to do it, because the pulp is a closed space and the process is equivalent to a com- partment syndrome.
A young man falls while skiing, and as he does so he jams his thumb into the snow. Physical examination shows collateral laxity at the thumb metacarpophalangeal joint.
dx? management?
(59) Ortho - Hand

This one is "gamekeeper's thumb." The injury was to the ulnar collateral ligament of the thumb. If not treated it can be dysfunctional and painful, and can lead to arthritis. Casting is usually done.
Two hoodlums grab a woman's purse and run away with it. She tries to grab one of the offenders by his jersey, but he pulls away, hurting the woman's hand in the process. When she makes a fist now, the distal phalanx of her ring finger does not flex with the others.
dx? management?
(60) Ortho - Hand

classic tendon injury - jersey finger (to the flexor) - Splinting is usually the first line of treatment.
While playing volleyball, a young lady injures her middle finger. She cannot extend the distal phalanx.

dx? management?
(61) Ortho- hand
classic tendon injury - mallet finger (injury to the extensor).Splinting is usually the first line of treatment.
While working at a bookbinding shop, a young man suffers a traumatic amputation of his index finger. The finger was cleanly severed at its base.
dx? management?
(62) Ortho - hand

Replantation of severed digits is no longer "miracle surgery." It is commonly done at specialized centers, and regular physicians should know how to handle the amputated part. The answer is to clean it with sterile saline, wrap it in a saline-moistened gauze, place it in a plastic bag, and place the bag on a bed of ice. The digit should not be placed in antiseptic solutions or alcohol, put in dry ice, or allowed to freeze.
A 45-year-old man gives a history of aching back pain for several months. He has been told that he had muscle spasms, and was given analgesics and muscle relaxants. He comes in now because of the sudden onset of very severe back pain that came on when he tried to lift a heavy object. The pain is like an electrical shock that shoots down his leg, it is aggravated by sneezing, coughing, or straining, and it prevents him from ambulating. He keeps the affected leg flexed. Straight leg-raising gives excruciating pain.
dx? management?
(63) Ortho - back pain

What is it? Lumbar disk herniation. The peak age incidence is 45, and virtually all of these are at either L4-L5 or L5-S1. If the "lightning" exits the foot by the big toe, it is L4-L5, if it exits by the little toe, it is L5-S1.
Management. MRI for diagnosis. Bed rest will take care of most of these. Neurosurgical inter- vention only if there is progressive weakness or sphincteric deficits.
A 46-year-old man has sudden onset of very severe back pain that came on when he tried to lift a heavy object. The pain is like an electrical shock that shoots down his leg, and it prevents him from ambulating. He keeps the affected leg flexed. Straight leg-raising test gives excruciating pain. He has a distended bladder, flaccid rectal sphincter, and perineal saddle area anesthesia.
dx? management?
(64) Ortho - back pain

The cauda equina syndrome is a surgical emergency.
A young man began to have chronic back pain at age 34. Pain and stiffness have been progressive. He describes morning stiffness, and pain that is worse at rest, but improves with activity.
dx? management?
(65) Ortho - back pain

Think ankylosing spondylitis. X-rays will eventually show "bamboo spine." Antiinflammatory agents and physical therapy are used.
A72-year-oldmanhashada20-poundweightloss,andhecomplainsoflow back pain. The pain 1s worse at night and is unrelieved by rest or positional changes.
dx? management?
(66) Ortho - back pain

Suggestive of metastatic malignancy. If advanced, xrays will show it. If not, bone scan is more sensitive (but less specific).
A 67-year-old diabetic has an indolent, unhealing ulcer at the heel of the foot.
dx? management?
(67) Ortho - leg ulcers

What is it? Ulcer at a pressure point in a diabetic is caused by neuropathy, but once it has hap- pened it is unlikely to heal because the microcirculation is poor also.
Management. Control the diabetes, keep the ulcer clean, keep the leg elevated- and be resigned to the thought that you may end up amputating the foot.
A 67-year-old smoker with high cholesterol and coronary disease has an indolent, unhealing ulcer at the tip of his toe. The toe is blue, and he has no peripheral pulses in that extremity.
dx? management?
(68) Ortho - leg ulcers

What is it? Ischemic ulcers are at the farthest away point from where the blood comes.
Management. Doppler studies looking for pressure gradient, arteriogram. Revascularization may be possible, and then the ulcer may heal.
A44-year-oldobesewomanhasanindolent,unhealingulceraboveherright medial malleolus. The skin around it is thick and hyperpigmented. She has frequent episodes of cellulitis, and has varicose veins.
dx? management?
(69) Ortho - leg ulcers

What is it?Venous stasis ulcer. Management. Unna boot, support stockings. Varicose vein surgery may ultimately be needed.
A40-year-oldmanhashadachronicdrainingsinusinhislowerlegsincehe had an episode of osteomyelitis at age 12. In the last few months he has developed an indolent, dirty-looking ulcer at the site, with "heaped up" tissue growth at the edges.
dx? management?
(70) Ortho - leg ulcers

Both of these are classic vignettes for the development of squamous cell carcinoma at long- standing, chronic irritation sites. The name Marjolin ulcer has been applied to these tumors. Obviously biopsy is the first diagnostic step, and wide local excision (with subsequent skin grafting) is the appropriate therapy.
Ever since she had an untreated third-degree burn to her lower leg at the age of 14, a 38-year-old immigrant from Latin America has had shallow ulcerations at the scar site that heal and break down all the time. In the last few months she has developed an indolent, dirtylooking ulcer at the site, with "heaped up" tissue growth around the edges, which is steadily growing and shows no sign of healing.
dx? management?
(71) Ortho - leg ulcers

Both of these are classic vignettes for the development of squamous cell carcinoma at long- standing, chronic irritation sites. The name Marjolin ulcer has been applied to these tumors. Obviously biopsy is the first diagnostic step, and wide local excision (with subsequent skin grafting) is the appropriate therapy.
An older, overweight man complalns of dlsabllng, sharp heel paln every trme his foot strlkes the ground. The pain is worse in the mornings, preventing hlm from putting any weight on the heel. X-rays show a bony spur matching the location of his pain, and physical exam shows exquisite tenderness right over that heel spur.
dx? management?
(72) Ortho - foot pain

Although all the signs point to that bony spur as the culprit, this is in fact plantar fasciitis- a very common but poorly understood problem that needs symptomatic treatment until it resolves spontaneously within 12 to 18 months.
A woman who usually wears high-heeled, pointed shoes complains of pain in the forefoot after prolonged standing or walking. Physical examination shows a very tender spot in the third interspace, between the third and fourth toes.
dx? management?
(73) Ortho - foot pain

'This one is a Morton neuroma, which is an inflammation of the common digital nerve. If conservative management (more sensible shoes, among other things) does not suffice, the neuroma may be excised.
A55-year-oldobesemansuddenlydevelopsswelling,redness,andexquisite pain at the first metatarsal-phalangeal joints.
dx? management?
(74) Ortho - foot pain

Gout. The diagnosis of the acute attack is done with identification of uric acid crystals in fluid from the joint. Treatment of the acute attack relies on indomethacin and colchicine. Long-term control of serum uric acid levels is done with allopurinol or probenecid.
A 72-year-old man with a history of multiple myocardial mfarctions is scheduled to have an elective sigmoid resection for diverticular disease. A preoperative radionuclide ventriculography shows an ejection fraction of less than 0.35.
dx? management?
(1) Preop - Cardiac risk

This is a "no-go" situation in which cardiac risk in noncardiac surgery is prohibitive. With this ejection fraction, the incidence of perioperative MI is 75 to 85%, and the mortality for such an event is around 55 to 90%. Probably the only option here is not to operate, but to continue with medical therapy for the diverticular disease. Should he develop an abscess, percutaneous drainage would be the only possible intervention.
A 72-year-old chronically bedridden man is being considered for emergency cholecystectomy for acute cholecystitis that is not responding to medical management. He had a transmural MI 4 months ago, and currently has atrial fibrillation, 8 to 10 premature ventricular beats per minute, and jugular venous distention.
dx? management?
(2) Preop - Cardiac risk

This fellow is a compendium of almost all of the items that Goldman has compiled as predic- tors of operative cardiac risk. In fact he adds up to 50 points, and anything above 25 points (class IV) gives a mortality in excess of 22%. Here again the best option would be to treat the cholecystitis in a different way (percutaneous radiologic tube cholecystostomy being the obvi- ous choice).
A 72-year-old man is scheduled to have an elective sigmoid resection for diverticular disease. In the preoperative evaluation it is noted that he has venous jugular distention.
dx? management?
(3) Preop - Cardiac risk

Now we have fewer items, but congestive heart failure is the worst one on the list (the other one here is his age). The failure has to be treated first: calcium-channel blockers, beta-blockers, dig- italis, and diuretics.
A 72-year-old man is scheduled to have an elective sigmoid resection for diverticular disease. In the preoperative evaluation it is ascertained that he had a transmural MI 2 months ago.

dx? management?
(4) Preop - Cardiac risk

The next worst Goldman finding is the recent MI (<6 months). Time is the best therapy for that one. Mortality is highest within 3 months of the MI (near 40%), but is brought down consid- erably after 6 months (6%).Waiting is the obvious choice here. If our hand is forced and earli- er operation becomes mandatory, admission to the intensive care unit (ICU) the day before surgery is recommended, to "optimize" all the cardiac parameters.
A 72-year-old man who needs to have elective repair of a large abdominal aortic aneurysm has a history of severe, progressive angina.

dx? management?
(5) Preop - Cardiac risk
This one is more tractable than the others. His coronary revascularization should precede the aneurysm repair.
A 61-year-old man with a 60 pack-year smoking history and physical evidence of chronic obstructive pulmonary disease (COPD) needs elective surgical repair of an abdominal aortic aneurysm. He currently smokes one pack per day.

dx? management?
(6) Preop - Pulm risk

Smoking is by far the most common cause of increased pulmonary risk, and the main problem is compromised ventilation (high Pco, and low FEV1) rather than compromised oxygenation. Cessation of smoking for 8 weeks and intensive respiratory therapy (physical therapy, expecto- rants, incentive spirometry, humidified air) should precede surgery.
A cirrhotic is bleeding from a duodenal ulcer. Surgical intervention is being considered. His bilirubin is 3.5, his prothrombin time is 22 seconds, his serum albumin is 2.5, and he has encephalopathy.
can this patient undergo surgery?
(7) Preop - Pulm Risk

Please don't! Any one of those items alone (bilirubin above 2, albumin below 3, prothrombin above 16, and encephalopathy) predicts a mortality of over 40%. If three of them are present, the number is 85%, and with all four we are talking about 100%.
A cirrhotic with a blood ammonia concentration above 150 ngldl needs an operation.
A cirrhotic with an albumin level below 2 needs an operation.
A cirrhotic with a bilirubin above 4 needs an operation.
can these patients undergo surgery?
(8-10) Preop - hepatic risk
Another way to look at liver risk is to see if any one of the previously listed findings is deranged to an even greater degree. Any one of these three examples would carry a mortality of about 80%. A deranged prothrombin time is slightly kinder to the patient, predicting only a 40-60% mortality. Death, incidentally, occurs with high-output cardiac failure with low peripheral resis- tance.
An elderly gentleman needs palliative surgery for a cancer of the esophagus. He has lost 20% of his body weight over the past 2 months, and his serum albumin is 2.7. Further testing reveals anergy to injected skin-test antigens and a serum transferrin level of less than 200 mgldl.
can this patient undergo surgery?
(11) Preop - nutritional risk

Any one of these four findings indicates severe nutritional depletion. All four leave no doubt as to the enormous operative risk that this man represents. Surprisingly, as few as 4 or 5 days of preoperative nutritional support (preferably via the gut) can make a big difference, and 7 to 10 days would be optimal if there is no big hurry to operate.
An elderly diabetic man presents with a clinical picture of acute cholecystitis that has been present for 3 days. He is profoundly dehydrated, in coma, and has a blood sugar of 950, severe acidosis, and ketone bodies "all over the place."
can this patient undergo surgery?
(12) Preop - metabolic risk

The treatment of diabetes is not within the scope of this surgical review, but we should point out that someone in overt diabetic ketoacidotic coma is not a surgical candidate, no matter how urgent the operation might be. The metabolic problem has to be addressed first in this case (although aiming for complete correction to normal values would be unrealistic as long as that rotten gallbladder is there).
Shortly after the onset of a general anesthetic with inhaled halothane and muscle relaxation with succinylcholine, a patient develops rapid rise in body temperature, exceeding 104°F. Metabolic acidosis and hypercalcemia are also noted. A family member died under general anesthesia several years before, but no details are available.

what is the likely cause of the fever?
(13) Postop Fever

A classic case of malignant hyperthermia. The history should have been a warning, but once the problem develops, treat with IV dantrolene plus the obvious support measures: 100% oxygen, correction of the acidosis, cooling blankets, watch for myoglobinuria.
Forty-five minutes after completion of a cystoscopy, a patlent develops chills and a fever spike of 104°F.

what is the likely cause of the fever?
(14) Postop Fever

This early o n after an invasive procedure, and this high fever, means bacteremia. Take blood cultures times 3, and start empiric antibiotic therapy.
In the first postoperativeday after an abdominal procedure, apatient develops a fever of 102°F.

what is the likely cause of the fever?
(15) Postop Fever

Fever on day one means atelectasis, but all the other potential sources have to be ruled out. Management would include chest x-ray, look at the wound and IV sites, inquire about urinary tract symptoms, and improve ventilation: deep breathing and coughing, postural drainage, incentive spirometry. The ultimate therapy for major, recalcitrant atelectasis is bronchoscopy.
In the first postoperative day after an abdominal procedure, a patient develops a fever of 102°F. The patient is not compliant with therapy for atelectasis, and by the third postoperative day he still has daily fever in the same range.

what is the likely cause of the fever?
(16) Postop Fever

Now a pneumonic process has developed in the atelectatic segments. Chest x-ray,sputum cultures, and appropriate antibiotics are needed.
Patient #1 had major abdominal surgery is afebrile during the first 2 postoperative days, but on day 3 he has a fever spike to 103°F.

Patient #2 spiked a fever on day 4, Patient #3 on day 5

what is the likely cause of the fever in each pt?
(17-19) Postop Fever

Every potential source of post-op fever always has to be investigated, but the timing of the first febrile episode gives a clue as to the most likely source. The mnemonic used (sequentially) is the "four Ws": wind (for atelectasis), water (for urine), walking (for the veins in the leg), and wound. Thus urinary tract infection, thrombophlebitis, and wound infection are the likely cul- prits in each of these vignettes. Urinalysis and urinary culture, Doppler studies, and physical examination are the respective tests.
A patient who had major abdominal surgery has a normal postoperative course, with no significant episodes of fever, until the 10th day when he begins to spike temperatures up to 102 and 103°F every day.

what is the likely cause of the fever?
(20) Postop Fever

Now one or more deep abscesses (pelvic or subphrenic) are the most likely source, and CT scans have to be done until they are found (and drained percutaneously). Because of how difficult it was to find these abscesses before we had CTs, the old mnemonic of the "Ws" assigned "wonder where" to these late fevers.The other late "W" when nothing else is found, is "wonder drugs" (medication-induced fever).
On the second postoperative day after an abdominoperineal resection for rectal cancer, a 72-year-old man complains of severe retrosternal pain, radiating to the left arm. He also becomes short of breath and tachycardic.

Management?
(21) Postop chest pain

Perioperative MI happens within the first 3 days, and the biggest triggering cause is hypov- olemic shock. These two are fairly typical scenarios, although the classic chest pain picture is often obscured by other ongoing events. When thinking MI everybody does an ECG, but the most reliable diagnostic test is troponin.
During the performance of an abdominoperineal resection for rectal cancer, unexpected severe bleeding is encountered, and the patient is hypotensiveon and off for almost 1 hour. The anesthesiologist notes ST depression and T wave flattening in the ECC monitor.

Management?
(22) Postop chest pain

Perioperative MI happens within the first 3 days, and the biggest triggering cause is hypov- olemic shock. These two are fairly typical scenarios, although the classic chest pain picture is often obscured by other ongoing events. When thinking MI everybody does an ECG, but the most reliable diagnostic test is troponin.
Ontheseventhpostoperativedayafterpinningofabrokenhip,a76-year-old man suddenly develops severe pleuritic chest pain and shortness of breath. When examined, he is found to be anxious, diaphoretic, and tachycardic, and he has prominent distended veins in his neck and forehead.

Management?
(23) Postop chest pain

Chest pain this late post-op is pulmonary embolus (PE).This patient is obviously at high risk, and the findings are classic. If they give you a similar vignette in which the venous pressure is low, it virtually excludes this diagnosis. Arterial blood gases are your first test, and hypoxemia and hypocapnia are the obligatory findings (in their absence, it is not a PE either). Although pulmonary angiography is the "gold standard," it is invasive and rarely done. Ventilation-perfu- sion lung scans (+/Qscan) have been done as the usual definitive test. +/Qscan works well only in otherwise normal lungs, which is seldom the case in postsurgical patients. Spiral CT scan has become the study of choice in many clinical centers.
Therapy starts with heparinization. The very active natural fibrinolytic mechanism in the lung makes the use of clot-busters less clearly indicated, but if PEs recur during anticoagulation, a vena cava filter (Greenfield) is needed.
Anawakeintubationisbeingattemptedinadrunkandcombativemanwho has sustained gunshot wounds to the abdomen. In the ongoing struggle the patient vomits and aspirates a large amount of gastric contents with particulate matter.

Management?
(24) Postop- Pulm complications

The nightmare of every anesthesiologist. Aspiration can kill a patient right away, or produce chemical injury to the tracheobronchial tree, with subsequent pulmonary failure, and often sec- ondary pneumonia.
Prevention is best (empty stomach, antacids before induction), but once it happens, lavage and removal of particulate matter is the first step (with the help of bronchoscopy), followed by bronchodilators and respiratory support. It is too late for steroids to help.
A man with advanced pulmonary tuberculosis (TB) is undergoing a laparotomy for resection of a perforated terminal ileum, secondary to intestinal TB. Halfway through the case it become progressively difficult to "bag" him, and his blood pressure steadily declines, while the CVP steadily rises. There is no evidence of intraabdominal bleeding.

Management?
(25) Postop- Pulm complications

Intraoperative tension pneumothorax. He probably blew a pulmonary bleb. The setting is too cumbersome, draped as he is, for anyone to put in a chest tube; but the abdominal surgeon can reach up and make a life-saving hole in the diaphragm. The formal chest tube can be placed later.
18 hrs after major surgery, a patient becomes disoriented

Management?
(26) Postop- confusion/coma

Very brief vignette, but out of the very long list of things that can produce post-op disorienta- tion, the most lethal one if not promptly recognized and treated is hypoxia. So, unless it is clear from the vignette that we can blame metabolic problems (uremia, hyponatremia, hyperna- tremia, ammonium, hyperglycemia, delirium tremens [DTs], or our own medications), the safest thing to ask for first is blood gases.
In the second week of a stormy, complicated postoperative period in a young patient with multiple gunshot wounds to the abdomen, he becomes progressively disoriented and unresponsive. He has bilateral pulmonary infiltrates, and a Po, of 65 while breathing 40% oxygen. He has no evidence of congestive heart failure.

Management?
(27) Postop- confusion/coma

The reason for the mental changes are obvious: he is not getting enough oxygen in his blood, but the rest of the findings specifically identify adult respiratory distress syndrome (ARDS).The centerpiece of therapy for AKDS is PEEP, and the latest wrinkle is to do it allowing significant hypercapnia, to minimize barotrauma. Another issue is why does he have it. In an older patient we can blame preexisting lung disease, and when there has been trauma to the chest, that can be the cause - but when those are not present, we have to think of sepsis as the precipitating event
Analcoholicmanchecksintohaveanelectivecolonresectionforrecurrent diverticular bleeding. He swears to everyone that he has not touched a drop of alcohol for the past 6 months. In the third postoperative day he becomes disoriented and combative, and claims to see elephants crawling up the walls. The wife then reveals that the patient actually drank heavily up until the day of hospital admission.
Management?
(28) Postop- confusion/coma

An open-and-shut case for DTs. From the surgical viewpoint, therapy is clear-cut: IV alcohol (5% alcohol/5% dextrose) will solve the acute problem (and prevent significant mortality). Psychiatrists, however, strongly object to therapy that perpetuates the addiction, and they favor pharmacologic therapy.
12hrs after completion of an abdommal hysterectomy, a 42yo F becomes confused & letharguc c/o severe headache, has a grand mal seizure, and finally goes into a coma. Review of the chart reveals that an order for D5W, to run at 125mL/hr, was mistakenly implemented as 525mL/hr
Management?
(29) Postop- confusion/coma

A classic example of water intoxication. The laboratory finding that will confirm it will be a very low serum sodium concentration. Mortality for this iatrogenic condition is very high, and ther- apy very controversial. Very careful use of hypertonic saline is probably a reasonable answer.
8hrs after completion of a trans-sphenoidal hypophysectomy for a prolactinoma, a young woman becomes lethargic, confused, and eventually comatose. Review of the record shows that her urinary output since surgery has averaged 600 ml/h, although her IV fluids are going in at 100 ml/h.
Management?
(30) Postop- confusion/coma

The reverse of the previous vignette. Large, rapid, unreplaced water loss from surgically induced diabetes insipidus. The lab will show significant hypernatremia, and the safest therapy would use one-third or one-fourth normal saline to replace the lost fluid, although in this acute setting D5W would be acceptable.
A cirrhotic patient goes into coma after an emergency portocaval shunt for bleeding esophageal varices.

Whats the cause of the coma? management?
(31) postop confusion/coma

Brief but obvious: the culprit here will be ammonium. If there is also hypokalemic alkalosis and high cardiac output- low peripheral resistance, overt liver failure has occurred.
Six hours after undergoing a hemorrhoidectomy under spinal anesthesia, a 62- year-old man complains of suprapubic discomfort and fullness. He feels the need to void but has not been able to do so since the operation. There is a palpable suprapubic mass that is dull to percussion.

management?
(32) postop- urinary

By far the most common post-up urinary problem is inability to void, and men are the likely victims. In-and-out bladder catheterization is the answer. Some authors recommend leaving an indwelling Foley catheter if catheterization has to be repeated in 6 hours, others wait until it has been done twice before suggesting it.
Amanhashadanabdominoperinealresectionforcanceroftherectum,and an indwellmg Foley catheter was left in place after surgery. The nurses are concerned because even though his v~tasl igns have been stable, his urinary output in the last 2hours has been zero.

management?
(33) postop - urinary

In the presence of renal perfusing pressure, an output of zero invariably means a mechanical
problem. In this case the catheter is plugged or kinked. More ominous - but much more rare - possibilities include both ureters having been tied off or thrombosis of the renal vessels.
Several hours after completion of multiple surgery for blunt trauma in an average-sizeadult, the urinary output is reported in three consecutivehours as 12 ml/h, 17 ml/h, and 9 ml/h. His blood pressure has hovered around 95 to 130 systolic during that time.

management?
(34) postop - urinary

His dneys are perfusing, but he is either behind in fluid replacement, or has gone into renal failure. A fluid challenge would suggest which one (a bolus of 500 ml given in 10 or 20 minutes should produce diuresis in the dehydrated patient, but not in renal failure), but the more ele- gant way (and the answer in the exam) is to look at urinary sodium. The dehydrated patient will be retaining sodium, and the urine will have less than 10 or 20 mEq/L. In renal failure the fig- ure will be >40. An even more elegant calculation is the fractional excretion of sodium, which in renal failure exceeds 1.
Four days after exploratory laparotomy for blunt abdominal trauma with resection and reanastomosis of damaged small bowel, a patient has abdominal distention, without abdominal pain. He has no bowel sounds and has not passed flatus, and his abdominal x-rays show ddated loops of small bowel without air fluid levels.

management?
(35) postop - abd distention

Probably paralytic ileus, which can be expected under the circumstances. NPO and NG suction should be continued until peristaltic activity resumes. Should resolution not be forthcoming, mechanical obstruction must be ruled out. A small barium tag will give the answer (it will "hang" somewhere if there is mechanical obstruction). Hypokalemia should be ruled out.
An elderly gentleman with Alzheimer's disease who lived in a nursing home is operated on for a fractured femoral neck On the fifth postoperative day it is noted that his abdomen is grossly distended and tense, but not tender. He has occasional bowel sounds. X-rays show a very distended colon, and a few distended loops of small bowel.

management?
(36) postop-abd distention

In the elderly who were not very active to begin with, and are now further immobilized, mas- sive colonic dilatation (Ogilvie) is commonly seen. Colonoscopy will suck out the gas, rule out mechanical obstruction (cancer), and allow a long rectal tube to be left in place afterward. In rare cases when the cecum is about to blow up, cecostomy or colostomy may be needed.
On the fifth post-operative day after a right hemicolectomy for cancer, the dressings covering the midline abdominal incision are found to be soaked with a clear, pinkish, salmon-colored fluid.
Dx?
Management? (3 steps)
(37) postop - wound care

Dx: Wound dehiscence

Management:
1. Keep the patient in bed
2. Tape his belly together
3. Schedule surgery for re-closure of the wound if the patient can take the re-operation.
(If too sick, the development of a ventral incisional hernia may have to be accepted now and repaired later)
Following the discovery of the copious, salmon colored, pinkish clear fluid along the post-op abdominal incision, the patient gets out of bed, or sneezes forcefully, and you are confronted with a bucket-full of small bowel
Dx?
Management? (2 steps)
(38) postop wound care

Dx: Evisceration

Management:
1. keep the bowel covered and moist with sterile dressings
2. Rush the patient to the OR for re-closure
Seven days after an inguinal hernia repair, a patient returns to the clinic because of fever. The wound is red, hot and tender.
Dx?
Management? (3 steps)
(39) postop wound care

Wound infection

Management:
1. Open the wound
2. Drain the pus
3. Pack it and leave it open
Nine days after a sigmoid resection for cancer, the wound drains a brown fluid that everybody recognizes as feces. The patient is afebrile, and otherwise doing quite well.

management?
(40) postop wound care

A fecal fistula, if draining to the outside, is inconvenient but not serious. It will close eventual- ly with little or no therapy. If feces were accumulating on the inside, the patient would be febrile and sick, and would need drainage and probably a diverting colostomy.
Eight days after a difficult hemigastrectomy and gastroduodenostomy for gastric ulcer, a patient begins to leak 2-3 L of green fluid per day through the right corner of his bilateral subcostal abdominal wound.
(41) postop wound care

If he is febrile, with an acute abdomen, and sick, he needs to be explored. But if all the gastric and duodenal contents are leaking to the outside, further surgery right away is not the answer. The problem is serious, though. Massive fluid and electrolyte replacement will be needed, and nutritional support will have to be provided with elemental nutrients delivered into the upper jejunum (total parenteral nutrition [TPN] is a poor second choice), hoping for eventual heal- ing without having to operate again.
Eight hours after completion of a trans-sphenoidal hypophysectomy for a prolactinoma, a young woman becomes lethargic, confused, and eventually comatose. Review of the record shows that her urinary output since surgery has averaged 600 ml/h, although her IV fluids are going in at 100 ml/h. A serum sodium determination shows a concentration of 152 mEq/L.
(42) postop fluids/lytes

An elevated concentration of serum sodium invariably means that the patient has lost pure water (or hypotonic fluids). Every 3 mEq/L above the normal of 140 represents 1 L lost. This woman is 4 L shy, which fits her history of a diuresis of 500 mllh more than the intake she is getting. As previously noted, she could be given 4 L of D5W, but many would prefer a similar amount of 5% dextrose in half normal saline, or 5% dextrose in one-third normal saline.
A group of Mexican nationals are smuggled into the United States in a closed metal truck. When the Border Patrol chases them, the smugglers abandon their charges in the middle of the desert, in the locked truck with no water to drink. The victims are found and rescued 5 days later. One of them is brought to your hospital, awake and alert, with obvious clinical signs of dehydration and a serum sodium concentration of 155 mEq/L.
(43) postop fluids/lytes

This gentleman has also lost water, about 5 L, but has done so slowly, by pulmonary and cuta- neous evaporation over 5 days. He is hypernatremic, but his brain has adapted to the slowly changing situation. Were he to be given 5 L of D5W, the rapid correction of his hypertonicity would be dangerous. Five liters of 5% dextrose in half normal saline would be a much safer plan.
Twelve hours after completion of an abdominal hysterectomy, a 42-year-old woman becomes confused and lethargic, complains of severe headache, has a grand ma1 seizure, and finally goes into coma. Review of the chart reveals that an order for D5W to run in at 125 ml/h was mistakenly implemented as 525 ml/h. Her serum sodium concentration is 122 mEq/L.
(44) postop fluids/lytes

Hyponatremia invariably means that water (without sodium) has been retained, thus the body fluids have been diluted. In this case a lot of IV water was given, and the antidiuretic hormone (ADH) produced as part of the metabolic response to trauma has held onto it. Rapidly devel- oping hyponatremia (water intoxication) is a big problem (the brain has no time to adapt),and once it has occurred the therapy is very controversial. Most authors would recommend hyper- tonic saline (either 3% or 5%) given 100 ml at a time, and reassessing the situation (clinical and lab) before each succeeding dose.
A 62-year-old woman comes in for her scheduled chemotherapy administration for her metastatic cancer of the breast. Although she is quite asymptomatic, the lab reports that her serum sodium concentration is 122 mEq/L.
(45) postop fluids/lytes

In this setting, water has also been retained (by ADH produced by the tumor), but so slowly that the brain has kept up with the developing hypotonicity. Rapid correction would be lethal and ill advised. Water restriction, on the other hand, will slowly allow the abnormality to be reversed.
A 68-year-old woman comes in with an obvious incarcerated umbilical hernia. She has gross abdominal distension, is clinically dehydrated, and reports persistent fecaloid vomiting for the past 5 days. She is awake and alert, and her serum sodium concentration is 118 mEq/L.
(46) postop fluids/lytes

Hyponatremia means water retention, but in this case the problem began with loss of isotonic (sodium-containing) fluid from her gut. As her extracellular fluid became depleted, she has retained whatever water has come her way: tea and Coke that she still was able to drink early on, and endogenous water from catabolism. Thus she is now volume-depleted at the same time that she is hyponatremic (hypotonic). She desperately needs volume replacement, but we do not want to correct her hypotonicity too quickly. Thus lots of isotonic fluids (start with 1 or 2 L/h of normal saline or Ringer lactate, depending on her acid-base status) would be the way to go (use clinical variables to fine-tune). Once her volume is replenished, she will unload the retained water and correct her own tonicity.
A patient with severe diabetic ketoacidosis comes in with profound dehydration and a serum potassium concentration of 5.2 mEq/L. After several hours of vigorous therapy with insulin and IV fluids (saline, without potassium), his serum potassium is 2.9.
(47) postop fluids/lytes

Severe acidosis (or alkalosis, for that matter) results in the loss of potassium in the urine. While the acidosis is present, though, the serum concentration is high because potassium has come out of the cells in exchange for hydrogen ion. Once the acidosis is corrected, that potassium rushes back into the cells, and the true magnitude of the potassium loss becomes evident. He obviously needs potassium. (Under most circumstances, 10 mEq/h is a safe "speed limit." In this setting, 20 mEq/h can be justified.)
An18-year-oldwomanslipsandfallsunderabus,andherrightlegiscrushed. On arrival at the ER she is hypotensive, and she receives several units of blood. Over the next several hours she is in and out of hypovolemic shock, and she develops acidosis. Her serum potassium concentration, which was 4.8 mEq/L at the time of admission, is reported to be 6.1 a few hours later.
(48) postop fluids/lytes

Let's count the ways in which potassium has been pouring into her blood: it came out of the crushed leg, it came in with the blood transfusions, and it came from the cells when she became acidotic. With low perfusing pressure (in and out of shock), the kidneys have not been doing a great job of eliminating it. We will have to do that. In addition to improving her blood pressure, we can "push potassium into the cells" with insulin and 50% dextrose. We can help dispose of it with exchange resins, and we can neutralize it with IV calcium. Hemodialysis is the ultimate weapon.
An elderly alcoholic, diabetic man, with marginal renal function, sustains multiple trauma while driving under the influence of alcohol. In the course of his resuscitation and multiple surgeries, he is in and out of shock for prolonged periods of time. Blood gases show a pH of 7.1 and Pco, of 36. His serum electrolytes are sodium 138, chloride 98, and bicarbonate 15.
(49) postop fluids/lytes

This man has every reason to develop metabolic acidosis, and he will do so by retention of fixed acids (rather than by loss of bicarbonate). The main driving force in this case is the state of shock, with lactic acid production; but the diabetes, alcohol, and bad kidney are also con- tributing. The lab shows that indeed he has metabolic acidosis (low pH and low bicarbonate), he is trying to compensate by hyperventilating (low Pco?), and he shows the classic anion gap (the sum of his chloride and bicarbonate is 25 mEq shy of the serum sodium concentration- instead of the normal 10 to 15).
As for the therapy, the classic treatment for metabolic acidosis is either bicarbonate or bicar- bonate precursors, like lactate or acetate. But in cases like this, reliance on such therapy tends to eventually produce alkalosis once the low flow state is corrected. Thus the emphasis here should be in fluid resuscitation. However, the choice of fluid is critical: a lot of saline would not be a good idea (too much chloride). A lot of Ringer lactate would be a better choice.
A patient who has had a subtotal gastrectomy for cancer, with a Billroth 2 reconstruction, develops a "blowout" of the duodenal stump, and a subsequent duodenal fistula. For the past 10 days he has been draining 750 to 1,500 ml/d of green fluid. His serum electrolytes show a sodium of 132, a chloride of 104, and a bicarbonate of 15. The pH in his blood is 12, with a Pco, of 35.
(50)postop fluids/lytes

Again, metabolic acidosis, but now with a normal anion gap. He has been losing lots of bicar- bonate out of the fistula. The problem would not have developed if his IV fluid replacement had contained lots of bicarbonate (or lactate, or acetate), but the use of those agents is indicated now for the therapy of the existing abnormality.
A patient with severe peptic ulcer disease develops pyloric obstruction and has protracted vomiting of clear gastric contents (i.e., without bile) for several days. His serum electrolytes show sodium of 134, chloride 82, potassium 2.9, and bicarbonate 34.
(51) postop fluids/lytes

The classic hypochloremic, hyyokalemic, metabolic alkalosis secondary to loss of acid gastric juice. This man needs to be rehydrated (choose saline rather than Ringer lactate), and he needs lots of potassium chloride (10 mEq/h will give him plenty, and will be a safe rate). Very rarely is ammonium chloride (or diluted, buffered hydrochloric acid) needed.
A 62-year-old man describes epigastric and substernal pain th;it he cannc characterize well. At trmes his description sounds like gastroesophageal reflux, at times it does not. Sonogram of the gallbladder, ECC, and cardiac enzymes have been negative.

whats the workup needed?
(1) GI - esophagus

What is it? The question is, is it gastroesophageal reflux?

Diagnosis. Esophageal pH monitoring.
A 54 year old obese man gives a history of burning retrosternal pain and “heartburn” that is brought about by bending over, wearing tight clothing or lying flat in bed at night. He gets symptomatic relief from antiacids, but the disease process seems to be progressing since it started several years ago.
Dx?
Management?
(2) GI - esophagus

Dx: Gastroesophageal reflux

Management: Endoscopy and biopsies

(to assess the extent of esophagitis and potential complications before medication)
A 54 year old obese man gives a history of burning retrosternal pain and “heartburn” that is brought about by bending over, wearing tight clothing or lying flat in bed at night. He gets symptomatic relief from antiacids, but the disease process seems to be progressing since it started several years ago. Endoscopy shows severe peptic esophagitis and Barrett’s esophagus.
Management?
(3) GI - esophagus

Management: first step should be medical management, then Nissen Fundoplication if symptoms persist

(since Barrett’s is premalignant)
A 54-year-old obese man gives a history of many years of burning retrosternal pain and heartburn that is brought about by bending over, wearing tight clothing, or lying flat in bed at night. He gets brief symptomatic relief from antacids, but in spite of faithful adherence to a strict program of medical therapy, the process seems to be progressing. Endoscopy shows severe peptic esophagitis with no dysplastic changes. Surgical treatment has been recommended.
(4) GI - esophagus

A rather advanced question, but the point is that if a Nissen is to be done, the surgeon will want reassurance that the problem is indeed reflux (pH monitoring), that the motility of the esoph- agus is okay (manometry studies), that there is indeed esophagitis severe enough to warrant surgery (endoscopy and biopsies), and that gastric emptying is not a problem (isotope gastric
emptying study). A barium swallow will show the anatomy to plan the procedure
about every esophageal study is needed.
A 47 year old lady describes difficulty swallowing which she has had for many years. She says that liquids are more difficult to swallow than solids, and she has learned to sit up straight and wait for the fluids to “make it through”. Occasionally she regurgitates large amounts of undigested food.
Dx?
Diagnostic test? (3)
Tx? (3 possible)
(5) GI - esophagus

Dx: Achalasia

Diagnostic test:
1. Manometry studies (gold standard)
2. CXR w/ barium swallow
3. Endoscopy

Tx:
1. Pneumatic dilation
2. Surgical Myotomy
3. Botox injection (if pt >50 yo is first Tx)
A 54 year old black man with a hx of smoking and drinking describes progressive dysphagia that began 3 months ago with difficulty swallowing meat, progressed to soft foods and is now evident for liquids as well. He locates the place where food “sticks” at the lower end of the sternum. He has lost 30 pounds of weight.
Dx?
Diagnostic tests? (3 in order)
(6) GI - esophagus

Dx: Carcinoma of the Esophagus

Diagnostic test:
1. Barium swallow
2. Endoscopy w/ biopsies
3. CT scan
A 24 year old man spends the night cruising bars and drinking heavily. In the wee hours of the morning he is quite drunk and starts vomiting repeatedly. Eventually he has a particularly violent episode of vomiting and he feels a very severe, wrenching epigastric and low sternal pain of sudden onset. On arrival at the E.R. one hour later he still has the pain, he is diaphoretic, has fever and leukocytosis and looks quite ill.
Dx?
Diagnostic test?
Tx?
(7) GI - esophagus

Dx: Boerhave’s syndrome

Diagnostic test: Gastrographin swallow

Treatment: Emergency surgical repair

(Prognosis depends on time elapsed between perforation and treatment)
A 24 year old man spends the night cruising bars and drinking heavily. In the wee hours of the morning he is quite drunk and he starts vomiting repeatedly. He initially brings up gastric contents only, but eventually he vomits bright red blood.
Dx?
Diagnostic test?
(8) GI - esophagus
Dx: Mallory Weiss tear

Diagnostic test: Endoscopy

(Photocoagulation may be used if needed)
A 55 year old man has an upper G.I. endoscopy done as an outpatient to check on the progress of medical therapy for gastric ulcer. Six hours after the procedure, he returns complaining of severe, constant, retrosternal pain that began shortly after he went home. He looks prostrate, very ill, is diaphoretic, has a temperature of 104 and respiratory rate of 30.
Dx?
Diagnostic test?
(9) GI - esophagus

Dx: Instrumental perforation of the esophagus

Diagnostic test: Gastrographin swallow
A 72 year old man has lost 40 pounds of weight over a two or three month period. He gives a history of anorexia for several months, and of vague epigastric discomfort for the past 3 weeks.
Dx?
Diagnostic test?
(10) GI - Stomach

Dx: Cancer of the stomach

Diagnostic test: Endoscopy and biopsies
A 54 year old man has had colicky abdominal pain and protracted vomiting for several days. He has developed progressive moderate abdominal distention, and has not had a bowel movement or passed any gas for five days. He has high pitched, loud bowel sounds that coincide with colicky pain, and X-Rays that show distended loops of small bowel and air-fluid levels. Five years ago he had an exploratory laparotomy for a gunshot wound of the abdomen.
Dx?
Management?
(11) GI - small intestine/appendix

Dx: Mechanical Intestinal Obstruction, due to adhesions

Management: Nasogastric suction, I.V. fluids and careful observation
A 54 year old man has had colicky abdominal pain and protracted vomiting for several days. He has developed progressive moderate abdominal distention, and has not had a bowel movement or passed any gas for five days. He has high pitched loud bowel sounds that coincide with the colicky pain, and X-Rays that show distended loops of small bowel and air-fluid levels. Five years ago he had an exploratory laparotomy for a gunshot wound of the abdomen. Six hours after being hospitalized and placed on nasogastric suction and I.V. fluids, he develops fever, leukocytosis, abdominal tenderness and rebound tenderness
Dx?
Management?
(12) GI - small intestine/appendix
Dx: Strangulated Obstruction
(a loop of bowel is dying –or dead- from compression of the mesenteric blood supply)

Management: Emergency surgery
A 54 year old man has had colicky abdominal pain and protracted vomiting for several days. He has developed progressive moderate abdominal distention, and has not had a bowel movement or passed any gas for five days. He has high-pitched, loud bowel sounds that coincide with the colicky pain, and X-Rays that show distended loops of small bowel and air-fluid levels. On physical exam a groin mass is noted, and he explains that he used to be able to “push it back” at will, but for the past 5 days has been unable to do so.
Dx?
Management?
(13) GI - small intestine/appendix

Dx: Mechanical Intestinal Obstruction, due to an incarcerated (potentially strangulated) Hernia.

Management: After suitable fluid replacement needs urgent surgical intervention
A 55 year old lady is being evaluated for protracted diarrhea. On further questioning she gives a bizarre history of episodes of flushing of the face, with expiratory wheezing. A prominent jugular venous pulse is noted on her neck.
Dx?
Diagnostic test? (2 steps)
Tx? (3 depending on position)
(14) GI - small intestine/appendix

Dx: Carcinoid syndrome.

Diagnostic test:
1. 24 hour Serum determinations of 5-hydroxy-indoleacetic acid (5-HIAA) or 5-HTP;
2. CT scan of abdomen

Tx: Serotonin antagonists;
then...
If Appendiceal < 2cm = Appendectomy.
If Appendiceal > 2cm = Right hemicolectomy;
Small intestinal = resect tumor w/ mesenteric LN
A 22 year old man develops vague periumbilical pain that several hours later becomes sharp, severe, constant and well localized to the right lower quadrant of the abdomen. On physical examination he has abdominal tenderness, guarding and rebound to the right and below the umbilicus. He has a temperature of 99.6 and a WBC of 12,500, with neutrophilia and immature forms.
Dx?
Management?
(15) GI - small intestine/appendix


Dx: Acute Appendicitis

Management: Exploratory laparotomy and appendectomy
A 59 year old is referred for evaluation because he has been fainting at his job where he operates heavy machinery. He is pale and gaunt, but otherwise his physical exam is remarkable only 4+ occult blood in the stool. Lab studies show a hemogoblin of 5.
Dx?
Diagnostic test?
Tx?
(16) GI - colon

Dx: Cancer of the right colon

Diagnostic test: Colonoscopy and biopsies

Treatment: Blood transfusions and eventually Right Hemicolectomy
A 56 year old man has bloody bowel movements. The blood coats the outside of the stool, and has been constipated, and his stools have become of narrow caliber.
Dx?
Diagnostic test?
(17) GI - colon
Dx: Cancer of the distal, left side of the colon

Diagnostic test: Endoscopy and biopsies

(If given choices start with Flexible Sigmoidoscopy)
A patient presents with pigmented spots on his lips and a history of recurrent colicky abdominal pain.
Dx?
(18) GI - colon

Dx: Peutz-Jeghers syndrome
A 42 year old man has suffered from chronic ulcerative colitis for 20 years. He weights 90 pounds and has had at least 40 hospital admissions for exacerbations of the disease. Due to a recent relapse, he has been placed on high dose steroids and immuran. For the past 12 hours he has had severe abdominal pain, temperature of 104 and leukocytosis. He looks ill, and “toxic”. His abdomen is tender particularly in the epigastric area, and he has muscle guarding and rebound. X-Rays show a massively distended transverse colon, and there is gas within the wall of the colon.
Dx?
Management?
(19) GI - colon

Dx: Toxic megacolon

Management: Emergency surgery for the toxic megacolon and removal of the rectum

(but the case illustrates many other indications for surgery: chronic malnutrition, “intractability” and risk of developing cancer)
A 27 year man is recovering from an appendectomy for gangrenous acute appendicitis with perforation and periappendicular abscess. He has been receiving Clindamycin and tobramycin for seven days. Eight hours ago he developed watery diarrhea, crampy abdominal pain fever and leukocytosis
Dx?
Diagnostic test?
Management?
(20) GI - colon

Dx: Pseudomembranous colitis from overgrowth of Clostridium Difficile

Diagnostic test: Stool cultures (but proctosigmoidoscopy can show a typical picture before the cultures are back)

Management: Stop the clindamycin, give either Vancomycin or Metronidazole, and avoid lomotil
A 60 year old man known to have hemorrhoids reports bright red blood in the toilet paper after evacuation.
Dx?
Management?
(21) GI - anorectal

Dx: Internal hemorrhoids

Management: Proctosigmoidoscopic Examination
(It is not reassurance and hemorrhoid remedies prescribed by telephone. In all these cases, cancer of the rectum has to be ruled out)
A 60 year old man known to have hemorrhoids complains of anal itching and discomfort, particularly towards the end of the day. He has perianal pain when sitting down and finds himself sitting sideways to avoid the discomfort. He is afebrile.
Dx?
Management?
(22) GI - anorectal

Dx: External hemorrhoids

Management: Proctosigmoidoscopic Examination

(It is not reassurance and hemorrhoid remedies prescribed by telephone. In all these cases, cancer of the rectum has to be ruled out)
A 23 year old lady describes exquisite pain with defecation and blood streaks on the outside of the stools. Because of the pain she avoids having bowel movements and when she finally does, the stools are hard and even more painful. Physical examination can not be done, as she refuses to allow anyone to even “spread her cheeks” to look at the anus for fear of precipitating the pain.
Dx?
Management?
Surgical Tx?
(23) GI - anorectal

Dx: Anal Fissure

Management: Exam under Anesthesia

(Even though the clinical picture is classical, cancer still has to be ruled out)

Tx: Lateral Internal Sphincterotomy
A 28 year old male is brought to the office by his mother. Beginning four months ago he has had three operations, done elsewhere, for a perianal fistula, but after each one the area has not healed, but actually the surgical wounds have become bigger. He now has multiple unhealing ulcers, fissures all around the anus, with purulent discharge. There are no palpable masses.
Dx?
Diagnostic test?
Top 3 medical Tx?
(24) GI - anorectal

Dx: Crohn's Disease

(The perianal area has fantastic blood supply and heals beautifully even though feces bathe the wounds. When it does not, you immediately think of Crohn’s disease)

Diagnostic test: Flexible sigmoidoscopy with Biopsy
(You still have to rule out malignancy)

Top 3 medical Tx:
1. Sulfasalazine
2. Metronidazole
3. Prednisone
A 44 year old man shows up in the E.R. at 11 PM with exquisite perianal pain. He can not sit down, reports that bowel movements are very painful, and that he has been having chills and fever. Physical examination shows a hot, tender, red, fluctuant mass between the anus and the ischial tuberosity.
Dx?
Management?
(25) GI - anorectal

Dx Ischiorectal abscess

Management: Exam under Anesthesia with Incision and Drainage

(The treatment for all abscesses is drainage. This one is no exception. But as always, cancer has to be ruled out)
A 62 year old man complains of perianal discomfort, and reports that there are streaks of fecal soiling in his underwear. Four months ago he had a perirectal abscess drained surgically. Physical exam shows a perianal opening in the skin, and a cord-liked tract can be palpated going from the opening towards the inside of the anal canal. Brownish purulent discharge can be expressed from the tract.
Dx?
First step?
Tx?
(26) GI - anorectal

Dx: Anal Fistula

First:
Rule-out cancer with Proctosigmoidoscopy

Tx: elective Fistulotomy
A 55-year old, HIV positive man, has a fungating mass growing out of the anus, and rock hard, enlarged lymph nodes on both groins. He has lost a lot of weight, and looks emaciated and ill.
Dx?
Diagnostic Test?
Eventual Tx?
(27) GI - anorectal

Dx: Squamous cell carcinoma of the anus

Diagnostic test: Biopsies of the fungating mass.

Eventual treatment: Nigro protocol of pre-operative chemotherapy and radiation
A 33 year old man vomits a large amount of bright red blood.
Where can the bleeding be from?
Diagnostic test?
(28) GI bleeding

Bleeding from: Tip of the nose to the ligament of Treitz.

Diagnostic test: for all upper G.I. bleeding, start with Endoscopy
A 33 year old man has had three large bowel movements that he describes as made up entirely of dark red blood. The last one was 20 minutes ago. He is diaphoretic, pale, has a blood pressure of 90 over 70 and a pulse rate of 110.
Where is bleeding from?
Management?
(29) GI Bleeding

Bleeding from? Anywhere in GI tract

(The point of the vignette is that something needs to be done to define the area from which he is bleeding. With the available information it could be from anywhere in the G.I. tract)

Management: The first diagnostic move here is to place a Nasogastric tube
A 33 year old man has had three large bowel movements that he describes as made up entirely of dark red blood. The last one was 20 minutes ago. He is diaphoretic, pale, has a blood pressure of 90 over 70 and a pulse rate of 110. A nasogastric tube returns copious amounts of bright red blood.
Management?
(30) GI Bleeding

Management: Endoscopy

(Same as if he had been vomiting blood)
A 33 year old man has had three large bowel movements that he describes as made up entirely of dark red blood. The last one was 20 minutes ago. He is diaphoretic, pale, has a blood pressure of 90 over 70 and a pulse rate of 110. A nasogastric tube returns clear, green fluid without blood.
Diagnostic test?
(31) GI bleeding

Diagnostic test: Angiogram

(Clear fluid, without bile, would have exonerated the area down to the pylorus, and if there is bile in the aspirate, down to the ligament of Treitz...provided you are sure that the patient is bleeding now. That’s the case here. So, he is bleeding from somewhere distal to the ligament of Treitz. Further definition of the actual site is no longer within reach of upper endoscopy, and lower endoscopy is notoriously difficult and unrewarding in massive bleeding. If he is bleeding at more than 2 cc. per minute, emergency angiogram is the way to go)
A 72 year old man had three large bowel movements that he describes as made up entirely of dark red blood. The last one was two days ago. He is pale, but has normal vital signs. A nasogastric tube returns clear, green fluid without blood.
Diagnostic test? (2)
(32) GI bleeding

Diagnostic test: Upper and Lower Endoscopies

(The clear aspirate is meaningless because he is not bleeding right now. So the guilty territory can be anywhere from the tip of the nose to the anal canal. Across the board, ¾ of all GI bleeding is upper, and virtually all the causes of lower GI bleeding are diseases of the old: diverticulosis, polyps, cancer and angiodysplasias. So, is old, the overall preponderance of upper is balanced by the concentration of lower causes in old people...so it could be anywhere)
A 7 year old boy passes a large bloody bowel movement.
Dx?
Diagnostic test?
(33) GI bleeding

Dx: Meckel’s diverticulum
(in this age group)

Diagnostic test: Radioactively labeled Technetium scan
(not the one that tags reds cells, but the one that identifies gastric mucosa)
A 41 year old man has been in the intensive care unit for two weeks, being treated for idiopathic hemorrhagic pancreatitis. He has had several percutaneous drainage procedures for pancreatic abscesses, chest tubes for pleural effusions, and bronchoscopies for atelectasis. He has been in and out of septic shock and respiratory failure several times. Ten minutes ago he vomited a large amount of bright red blood, and as you approach him he vomits again what looks like another pint of blood.
Dx?
Diagnostic test?
How could it have been prevented?
Tx?
(34) GI bleeding

Tx: Stress Ulcer

Diagnostic test: Endoscopy

It should have been prevented by keeping the pH of the stomach above 4 with H2 blockers, antiacids or both

Treatment: Angiographic Embolization of the left gastric artery.
A 59 year old man arrives in the E.R. at 2 AM, accompanied by his wife who is wearing curlers on her hair and a robe over her nightgown. He has abdominal pain that began about one hour ago, and is now generalized, constant and extremely severe. He lies motionless in the stretcher, is diaphoretic and has shallow, rapid breathing. His abdomen is rigid, very tender to deep palpation, and has guarding and rebound tenderness in all quadrants.
Dx?
Management?
(35) acute abdomen

Dx: Acute Peritonitis (Acute Abdomen)

Management: Emergency Exploratory Laparotomy
A 62 year old man with cirrhosis of the liver and ascites, presents with generalized abdominal pain that started 12 hours ago. He now has moderate tenderness over the entire abdomen, with some guarding and equivocal rebound. He has mild fever and leukocytosis.
Dx?
Diagnostic test?
Tx?
(36) acute abdomen
Dx: Primary Peritonitis

(Peritonitis in the cirrhotic with ascitis, or the child with nephrosis and ascitis, could be primary peritonitis – which does not need surgery!)

Diagnostic test: Paracentesis with Cultures of the ascitic fluid will yield a single organism

Treatment: Antibiotics
A 43 year old man develops excruciating abdominal pain at 8:18 PM. When seen in the E.R. at 8:50 PM, he has a rigid abdomen, lies motionless in the examining table, has no bowel sounds and is obviously in great pain, which he describes as constant. X-Ray shows free air under the diaphragms.
Dx?
Management?
(37) acute abdomen

Dx: Acute abdomen plus perforated GI tract
(perforated duodenal ulcer in most cases)

Management: Emergency exploratory laparotomy
A 44 year old alcoholic male presents with severe epigastric pain that began shortly after a heavy bout of alcoholic intake, and reached maximum intensity over a period of two hours. The pain is constant, radiates straight through to the back and is accompanied by nausea, vomiting and retching. He had a similar episode two years ago, for which he required hospitalization.
Dx?
Diagnostic test?
If Dx is unclear?
Management? (3 together)
(38) acute abdomen

Dx: Acute pancreatitis

Diagnostic test: Serum and Urinary Amylase and Lipase

If unclear: CT scan
(or in a day or two if there is no improvement)

Management: NPO, NG suction, IV fluids.
A 43 year old obese lady, mother of six children, has severe right upper quadrant abdominal pain that began six hours ago. The pain was colicky at first, radiated to the right shoulder and around towards the back, and was accompanied by nausea and vomiting. For the past 2 hours the pain has been constant. She has tenderness to deep palpation, muscle guarding and rebound in the right upper quadrant. Her temperature is 101 and she has a WBC of 16,000. She has had similar episodes of pain in the past, brought about by ingestion of fatty food, but they all had been of brief duration and relented spontaneously or with anticholinergic medications.
Dx?
Diagnostic test?
Management?
(39) acute abdomen

Dx: Acute cholecystitis

Diagnostic test: Ultrasound

(If equivocal, an “HIDA” scan: radionuclide excretion scan)

Management: “cool down” the process

Surgery will follow
A 52 year old man has right flank colicky pain of sudden onset that radiates to the inner thigh and scrotum. There is microscopic hematuria.
Dx?
Diagnostic test? (2)
(40) acute abdomen

Dx: Ureteral colic

Diagnostic test: Urological evaluation always begins with a Plain Film of the abdomen (a “KUB”)

Ultrasound often is the next step
(but traditionally it has been intravenous pyelogram)
A 59 year old lady has a history of three prior episodes of left lower quadrant abdominal pain for which she was briefly hospitalized and treated with antibiotics. Now she has left lower quadrant pain, tenderness, and a vaguely palpable mass. She has fever and leukocytosis.
Dx?
Diagnostic test?
Management?
(41) Acute abdomen

x: Acute diverticulitis

Diagnostic test: CT scan
(Colonoscopy is not safe in acute setting)

Management: Elective Sigmoid resection
(for recurrent attacks, like this case or if she does not respond to medical Tx from initial attack or gets worse)

(Treatment is medical for the acute attack: antibiotics, NPO)
An 82 year old man develops severe abdominal distension, nausea, vomiting and colicky abdominal pain. He has not passed any gas or stool for the past 12 hours. He has a tympanitic abdomen with hyperactive bowel sounds. X-Ray shows distended loops of small and large bowel, and a very large gas shadow that is located in the right upper quadrant and tapers towards the left lower quadrant with the shape of a parrot’s beak.
Dx?
Management?
(42) acute abdomen

Dx: Volvulus of the sigmoid

Management: Proctosigmoidoscopy should relieve the obstruction

(Rectal tube is another option. Eventually surgery to prevent recurrences could be considered)
A 79 year old man with atrial fibrillation develops and acute abdomen. He has a silent abdomen, with diffuse tenderness and mild rebound. There is a trace of blood in the rectal exam. He has acidosis and looks quite sick. X-Rays show distended small bowel and distended colon up to the middle of the transverse colon.
Dx?
Tx if mild, moderate or severe?
(43) acute abdomen

Dx: Emboli of Mesenteric vessels

(Acute abdomen present in the elderly who has atrial fibrillation, brings to mind embolic occlusion of the mesenteric vessels. Acidosis frequently ensues, and blood in the stool is often seen)

Mild Tx: Observe only
Moderate Tx (fever and inc WBC only): IV Antibiotics

Severe Tx (Peritoneal signs): Exploratory Lap with Colostomy
A 53 year old man with cirrhosis of the liver develops malaise, vague right upper quadrant abdominal discomfort and 20 pound weight loss. Physical exam shows a palpable mass that seems to arise from the left lobe of the liver. Alpha feto protein is significantly elevated.
Dx?
Diagnostic test?
Tx?
(44) Liver

Dx: Liver cell carcinoma

Diagnostic test: CT scan

Tx: If confined to one lobe, Resection.
A 53 year old man develops vague right upper quadrant abdominal discomfort and a 20 pound weight loss. Physical exam shows a palpable liver with nodularity. Two years ago he had a right hemicolectomy for cancer of the ascending colon. His carcinoembryogenic antigen (CEA) had been within normal limits right after his hemicolectomy, is now ten times normal.
Dx?
Diagnostic test?
Tx?
(45) Liver

Dx: Metastasis to the liver from colon cancer

Diagnostic test: CT scan

Tx: If mets are confined to one lobe: Resection.

(Otherwise, Chemotherapy if he has not had it)
A 24 year old lady develops moderate, generalized abdominal pain of sudden onset, and shortly thereafter faints. At the time of evaluation in the ER she is pale, tachycardic, and hypotensive. The abdomen is mildly distended and tender, and she has a hemogoblin of 7. There is no history of trauma. On inquiring as to whether she might be pregnant, she denies the possibility because she has been on birth control pills since she was 14, and has never misses taking them.
Dx?
Management?
Tx?
(46) Liver

Dx: Bleeding from a ruptured Hepatic Adenoma, secondary to birth control pills.

Management:
CAT scan
(will confirm bleeding and probably show the liver adenoma as well)

Tx: Surgery
A 44 year old lady is recovering from an episode of acute ascending cholangitis secondary to choledocholithiasis. She develops fever and leukocytosis and some tenderness in the right upper quadrant. An ultrasound reveals a liver mass.
Dx?
Management?
(47) Liver

Dx: Pyogenic abscess

Management: it needs to be drained (the radiologists will do it percutaneously)
A 29 year old migrant worker from Mexico develops fever and leukocytosis, as well as tenderness over the liver when the area is percussed. He has mild jaundice and an elevated alkaline phosphatase. Ultrasound of the right upper abdominal area shows a normal biliary tree, and an abscess in the liver.
Dx?
Management?
(48) Liver

Dx: Amebic abscess
(very common in Mexico)

Management: Serology for Amebic titers and start on Metronidazole
(This one Abscess that does not have to be drained. Get serology for amebic titers, and start the patient on Metranidazole. Prompt improvement will tell you that you are on the right track...serologies in 3 weeks will confirm. Don’t fall for an option that suggests aspirating the pus and sending it for culture, you can not grow the ameba from the pus)
A 42 year old lady is jaundiced. She has a total bilirubin of 6 and the laboratory reports that the unconjugated, indirect bilirubin is 6 and the direct, conjugated bilirubin is zero. She has no bile in the urine.
Dx?
Management?
(49) GI - jaundice

Dx: Hemolytic Jaundice

Management: Try to figure out what is chewing her red cells.
A 19 year old college student returns from a trip to Cancun, and two weeks later develops malaise, weakness and anorexia. A week later he notices jaundice. When he presents for evaluation his total bilirubin is 12, with 7 indirect and 5 direct. His alkaline phosphatase is mildly elevated, while the SGOT and SGPT (transaminases) are very high.
Dx?
Management?
(50) GI - jaundice

Dx: Hepatocellular jaundice

Management: Get serologies to confirm diagnosis and type of Hepatitis
A patient with progressive jaundice which has been present for four weeks is found to have a total bilirubin of 22, with 16 direct and 6 indirect, and minimally elevated SGOT. The alkaline phosphatase was twice normal value couple of weeks ago, and now is about six times the upper limit of normal.
Dx?
Management?
(51) GI - Jaundice

Dx: Obstructive jaundice

Diagnostic test? Ultrasound

(looking for dilated intrahepatic ducts, possibly dilated extrahepatic ducts as well, and if we get lucky a finding of gallstones)
A 40 year old, obese mother of five children presents with progressive jaundice which she first noticed four weeks ago. She has a total bilirubin of 22, with 16 direct and 6 indirect, and minimally elevated SGOT. The alkaline phosphatase is about six times the upper limit of normal. She gives a history of multiple episodes of colicky right upper quadrant abdominal pain, brought about by ingestion of fatty food.
Dx?
Diagnostic test? (2)
Tx?
(52) GI - Jaundice

Dx: Obstructive jaundice

Diagnostic test: Ultrasound
(If you need more tests after that, ERCP is the next move, which could also be used to remove the stones from the common duct)

Tx: Cholecystectomy
A 66 year old man presents with progressive jaundice which he first noticed six week ago. He has a total bilirubin of 22, with 16 direct and 6 indirect, and minimally elevated SGOT. The alkaline phosphatase is about six times the upper limit of normal. He has lost 10 pounds over the past two months, but is otherwise asymptomatic. A sonogram shows dilated intrahepatic ducts, dilated extrahepatic ducts and a very distended, thin walled gallbladder.
Dx?
Management? (2)
(53) GI - Jaundice

Dx: Malignant obstructive jaundice.
(“Silent” obstructive jaundice is more likely to be due to tumor. A distended gallbladder is an ominous sign: when stones are the source of the problem, the gallbladder is thick-walled, non-pliable)

Management: CAT scan and ERCP
A 66 year old man presents with progressive jaundice which he first noticed six weeks ago. He has a total bilirubin of 22, with 16 direct and 6 indirect, and minimally elevated SGOT. The alkaline phosphatase is about six times the upper limit of normal. He is otherwise asymptomatic. A sonogram shows dilated intrahepatic ducts, dilated extrahepatic ducts and a very distended, thin walled gallbladder. Except for the dilated ducts, CT scan is unremarkable. ERCP shows a narrow area in the distal common duct, and a normal pancreatic duct.
Dx?
Next step?
Tx?
(54) GI - Jaundice

Dx: Malignant, but lucky... probably Cholangiocarcinoma at the lower end of the common duct.

Next step: get brushings of the common duct for cytological diagnosis.

Tx: He could be cured with a pancreatoduodenectomy
(Whipple operation)
A 64 year old lady presents with progressive jaundice which she first noticed two weeks ago. She has a total bilirubin of 12, with 8 direct and 4 indirect, and minimally elevated SGOT. The alkaline phosphatase is about ten times the upper limit of normal. She is otherwise asymptomatic, but is found to be slightly anemic and to have positive occult blood in the stool. A sonogram shows dilated intrahepatic ducts, dilated extrahepatic ducts and very distended, thin walled gallbladder.
Dx?
Diagnostic test?
Tx?
(55) GI - Jaundice

Dx: Malignant Obstructive jaundice
(The coincidence of slowly bleeding into the GI tract at the same time that she develops obstructive jaundice points to an Ampullary carcinoma, another malignancy that can be cured with Radical surgery)

Diagnostic test: Endoscopy
A 56 year old man presents with progressive jaundice which he first noticed six weeks ago. He has a total bilirubin of 22, with 16 direct and 6 indirect, and minimally elevated SGOT. He alkaline phosphatase is about eight times the upper limit of normal. He has lost 20 pounds over the past two months, and has a persistent, nagging mild pain deep into his epigastrium and in the upper back. His sister died at age 44 from a cancer of the pancreas. A sonogram shows dilated intrahepatic ducts, dilated extrahepatic ducts and a very distended, thin walled gallbladder.
Dx?
Diagnostic test? (2)
(56) GI - Jaundice

Dx: Cancer of the head of the pancreas
(Terrible prognosis)

Diagnostic test: CAT scan –which may show the mass in the head of the pancreas;
then ERCP –which will probably show obstruction of both common duct and pancreatic duct
A white, fat, female, aged 40 and mother of five children gives a history of repeated episodes of right upper quadrant abdominal pain brought about by the ingestion of fatty foods, and relieved by the administration of anticholinergic medications. The pain is colicky, radiates to the right shoulder and around to the back, and is accompanied by nausea and occasional vomiting. Physical exam is unremarkable.
Dx?
Diagnostic test?
(57) GI - Biliary

Dx: Gallstones, with biliary colic

Diagnostic test: Ultrasound

Tx: Elective cholecystectomy
A 43-year-old obese mother of six children has severe right upper quadrant abdominal pain that began 6 hours ago. The pain was colicky at first, radiated to the right shoulder and around toward the back, and was accompanied by nausea and vomiting. For the past 2 hours the pain has been constant. She has tenderness to deep palpation, muscle guarding, and rebound in the right upper quadrant. Her temperature is 10l°F,and she has a WBC count of 12,000. Liver function tests are normal.
Dx?
(58) GI - Biliary

What is it? If you are alert, you will recognize the picture of acute cholecystitis- in fact a similar vignette was presented in the acute abdomen section. It is repeated here to contrast it with the next one.
A 73 year old obese lady, mother of six children, has severe right upper quadrant abdominal pain that began three days ago. The pain was colicky at first, but has been constant for the past two and a half days. She has tenderness to deep palpation, muscle guarding and rebound in the right upper quadrant. She has temperature spikes to 104 and 105, with chills. Her WBC is 22,000, with a shift to the left. Her bilirubin is 5 and she has an alkaline phosphatase of 2,000 (about 20 times normal). She has had episodes of colicky pain in the past, brought about by ingestion of fatty food, but they all had been of brief duration and relented spontaneously or with anticholinergic medications.
Dx?
Further test?
Management? (2)
(59) GI - Biliary

Dx: Acute Ascending Cholangitis

Further test:
Ultrasound might confirm dilated ducts.

Management:
Emergency decompression of the biliary tract...
ERCP is the first choice, but PTC (percutaneous transhepatic cholangiogram) is another option
A white, fat, female, aged 40 and mother of five children gives a history of repeated episodes of right upper quadrant abdominal pain brought about by the ingestions of fatty foods, and relieved by the administration of anticholinergic medications. The pain is colicky, radiates to the right shoulder and around to the back, and is accompanied by nausea and occasional vomiting. This time she had a shaking chill with the colicky pain, and the pain lasted longer than usual. She has mild tenderness to palpation in the epigastrium and right upper quadrant. Laboratory determinations show a bilirubin of 3.5, an alkaline phosphatase 5 times normal and a serum amylase 3 times normal value.
Dx?
Diagnostic test?
Management/Tx if she gets better?
If she gets worse?
(60) GI - Biliary

Dx: She passed a common duct stone and had a transient episode of Cholangitis (the shaking chill, the high phosphatase) and a bit of Biliary Pancreatitis (the high amylase).

Diagnostic test: Ultrasound (It will confirm the diagnosis of gallstones)

Management: If she continues to get well, elective Cholecystectomy.
If she deteriorates, she may have the stone still impacted at the Ampulla of Vater, and may need ERCP and sphincterotomy to extract it
A 33 year old, alcoholic male, shows up in the E.R. with epigastric and mid-abdominal pain that began 12 hours ago shortly after the ingestion of a large meal. The pain is constant, very severe, and it radiates straight through to the back. He vomited twice early on, but since then has continued to have retching. He has tenderness and some muscle guarding in the upper abdomen, is afebrile and has mild tachycardia. Serum amylase is 1200, and his hematocrit is 52.
Dx?
Management? (3)
(61) Pancreas

Dx: Acute edematous pancreatitis.

Management: put the pancreas at rest...NPO, NG suction, IV fluids
A 56 year old alcoholic male is admitted with a clinical picture of acute upper abdominal pain. The pain is constant, radiates straight through to the back, and is extremely severe. He has a serum amylase of 800, WBC of 18,000 blood glucose of 150, serum calcium of 6.5 and a hematocrit of 40. He is given IV fluids and kept NPO with NG suction. By the next morning, his hematocrit has dropped to 30 the serum calcium has remained below 7 in spite of calcium administration, his BUN has gone up to 32 and he has developed metabolic acidosis and a low arterial PO2.
Dx?
Management/test?
(62) Pancreas

Dx: Hemorrhagic Pancreatitis
(In fact, he is in deep trouble, with at least eight of Ranson’s criteria predicting 80 to 100% mortality)

Management/test: Very intensive support will be needed, but the common pathway to death from complication of hemorrhagic pancreatitis frequently is by way of pancreatic abscesses that need to be drained as soon as they appear. Thus serial CT scans will be required.
A 57 year old alcoholic male is being treated for acute hemorrhagic pancreatis. He was in the intensive care unit for one week, required chest tubes for pleural effusion, and was on a respirator for several days, but eventually improved enough to be transferred to the floor. Two weeks after the onset of the disease he begins to spike fever and to demonstrate leukocytosis.
Dx?
Diagnostic test?
Tx?
(63) Pancreas

Dx: Pancreatic abscess

Diagnostic test: CT scan

Tx: Drainage
A 49 year old alcoholic male presents with ill-defined upper abdominal discomfort and early satiety. On physical exam he has a large epigastric mass that is deep within the abdomen, and actually hard to define. He was discharged from the hospital 5 weeks ago, after successful treatment for acute pancreatitis.
Dx?
Diagnostic test?
Tx?
(64) Pancreas

Dx: Pancreatic pseudocyst

Diagnostic test: You could diagnose it on the cheap with an ultrasound, but CT scan is probably the best choice.

Tx: It will need to be drained, and the radiologist will do it with CT guidance
A 55 year old lady presents with vague upper abdominal discomfort, early satiety and a large but ill-defined epigastric mass. Five weeks ago she was involved in an automobile accident where she hit the upper abdomen against the steering wheel.
Dx?
Diagnostic test?
(65) Pancreas

Dx: Pancreatic pseudocyst, secondary to trauma

Diagnostic test: CT scan
A disheveled, malnourished individual shows up in the emergency room requesting medication for pain. He smells of alcohol and complains bitterly of constant epigastric pain, radiating straight through to the back that he says he has had for several years. He has diabetes, steatorrhea and calcifications in the upper abdomen in a plain X-Ray.
Dx?
Diagnostic test?
Management? (3)
(66) Pancreas

Dx: Chronic pancreatitis

Diagnostic test: AXR visualizing calcifications

Management: Stop alcohol, replacement of pancreatic enzymes and control of the diabetes; ERCP
A 9-month-old baby girl is brought in because she has an umbilical hernia. The
defect is 1 cm in diameter, and the contents are freely reducible.
(67) Hernias

Although we routinely recommend elective surgical repair of all hernias (to prevent the ghast- ly complication of strangulation), there are some exceptions. This is one. Umbilical hernias in babies younger than the age of 2 years may still close spontaneously. Only observation is needed here.
An 18-year-old man has a routine physical examination as part of his college registration, and the examination reveals that he has a right inguinal hern~a. The external inguinal ring is about 2.5 cm in diameter, and a hernial bulge can be eas~lyseen and felt going down into his scrotum when he is asked to strain. He is completely asymptomatic and was not even aware of the presence of the hernia.
(68) Hernias

Elective surgical repair is in order. Even though he is asymptomatic, he should not be exposed to the risk of bowel strangulation. They will not ask you about specific technical details. The hernia is probably indirect, and either open or laparoscopic (preferably extraperitoneal) approach can be used.
A72-year-oldfarmerisforcedbyhisinsurancecompanytohaveaphysical examination to be issued a life insurance policy. He has been healthy all his life, and "has never been to the doctor". At the examination it is found that he has a large, left inguinal hernia that reaches down into the scrotum. Bowel sounds can be easily heard over it. The hernia is not reducible, and he says that many years ago he used to be able to "push it back," but for the last 10 or 20 years he has not been able to do so.
(69) Hernias

A hernia that cannot be pushed back in (reduced) is incarcerated, and one that has compro- mised blood supply is strangulated. The latter is an emergency. The former is also an emergen- cy if the irreducible state is of new onset, because one does not want to wait for overt signs of dead or compromised bowel before operating. But if he has been this way for 10 or 20 years, obviously the bowel is alive and well. Elective repair is still indicated, before he runs out of good luck and gets into trouble.
An 18 year old lady has a firm, rubbery mass in the left breast that moves easily with palpation.
Dx?
Diagnostic Test?
Imaging technique for young patient?
(1) Breast
Dx: Fibroadenoma

Diagnostic Test: Tissue diagnosis...(choices in order)
1. FNA; 2. Core Bx; 3. Excisional Bx

(The only safe answer, even if the presentation favors benign disease, is to get tissue diagnosis)

Sonogram is the only imaging technique suitable for the very young breast
A 14-year-old girl has a firm, movable, rubbery mass in her left breast that was first noticed 1 year ago and has since grown to be about 6 cm in diameter.
(2) Breast

What is it? Giant juvenile fibroadenoma.
Management. At age 14 chances of cancer are virtually zero. That avenue does not have to be explored. But the rapid growth requires resection to avoid cosmetic deformity.
A 27 year old immigrant from Mexico has a 12 x 10 x 7 cm. mass in her left breast. It has been present for seven years, and slowly growing to its present size. The mass is firm, rubbery, completely movable, is not attached to chest wall or to overlying skin. There are no palpable axillary nodes.
Dx?
Diagnostic Test?
(3) Breast

Dx: Cystosarcoma Phyllodes

(basically same presentation as Fibroadenoma, but >25yo)

Diagnostic test: given the size best done with core or incisional biopsy

(no need for axillary node dissection w/ phyllodes...mets is rare)
A 35 year old lady has a ten year history of tenderness in both breasts, related to menstrual cycle, with multiple lumps on both breasts that seem to “come and go” at different times in the menstrual cycle. Now has a firm, round, 2 cm. mass that has not gone away for 6 weeks.
Dx?
Diagnostic Test?
(4) Breast

Dx: Fibrocystic disease

Diagnostic test: Aspiration of the Cyst

(tissue diagnosis [i.e: biopsy] becomes impractical when there are lumps every month)

If the mass goes away and the fluid aspirated is clear, that’s all.
If the fluid is bloody it goes to cytology.
If the mass does not go away, or recurs she needs biopsy.
(Answers that offer mammogram or sonogram in addition to the aspiration would be OK, but not as the only choice)
A 34 year old lady has been having bloody discharge from the right nipple, on and off for several months. There are no palpable masses.
Dx?
Diagnostic Test?
if test is inconclusive?
(5) Breast

Dx: Intraductal papilloma

Diagnostic test: Mammogram
(the way to detect breast cancer that is not palpable)

(If negative, one may still wish to find an resect the intraductal papilloma to provide symptomatic relief. Resection can be guided by galactogram, or done as a retroareolar exploration)
A 26 year old lactating mother has cracks in the nipple and develops a fluctuating, red, hot, tender mass in the breast, along with fever and leukocytosis.
Dx?
Management?
(6) Breast

Dx: Abscess

(However, only lactating breasts are “entitled” to develop abscesses. On anybody else, a breast abscess is a cancer until proven otherwise.)

Management: Incision and Drainage

(if an option includes drainage with biopsy of the abscess wall, go for that one)
A 49 year old has a firm 2cm mass in the right breast that has been present for 3 months.
Dx?
Management?
(7) Breast

Dx: This could be anything.

(Age is the best determinant for Cancer of the breast. If she had been 72, you go for cancer. At 22, you favor benign. But they will not ask you what this is, they will ask what do you do.)

Management: You have to have tissue. Core biopsy is OK, but if negative you don’t stop there. Only excisional biopsy will rule out cancer
A 34-year-old woman in her fifth month of pregnancy reports a 3-cm firm, ill- defined mass in her right breast that has been present and growing for 3 months.
(8) Breast

The diagnosis of possible breast cancer in the pregnant patient is done the same way as if she had not been pregnant. Yes, you can do the mammogram and appropriate biopsies; no, you do not need to terminate the pregnancy.
A 69 year old lady has a 4 cm hard mass in the right breast, with ill defined borders, movable from the chest wall but not movable within the breast. The skin overlying the mass is retracted an has an “orange peel” appearance…or the nipple became retracted six months ago.
Dx?
Diagnostic Test?
(9) Breast

Dx: Cancer of the Breast

Diagnostic test: Core or Excisional Bx
A 69-year-old woman has a 4-cm hard mass in the right breast under the nipple and areola with ill-defined borders, movable from the chest wall but not movable within the breast. The nipple became retracted 6 months ago.

Dx?
Diagnostic Test?
(10) Breast

Dx: Cancer of the Breast

Diagnostic test: Core or Excisional Bx
A 72-year-old woman has a red, swollen breast. The skin over the area looks like orange peel. She is not particularly tender, and it is debatable whether the area is hot or not. She has no fever or leukocytosis.
(11) Breast

What is it?Another classic cancer of the breast. Management. Same as above. Start with mammogram, then get that tissue diagnosis (here a punch biopsy of the skin is an option; it probably is permeated with cancer).
A 62 year old lady has an eczematoid lesion in the areola. It has been present for 3 months and it looks to her like “some kind of skin condition” that has not improved or gone away with a variety of lotions and ointments
Dx?
Diagnostic Test?
(12) Breast

Dx: Paget’s disease of the breast
(which is a cancer under the areola)

Management: Full thickness punch biopsy of the skin would be OK, but core biopsy or incisional biopsy of the tissue underneath would be OK also
A 42 year old lady hits her breast with a broom handle while doing her housework. She noticed a lump in that area at the time, and one week later the lump is still there. She has a 3 cm hard mass deep inside the affected breast, and some superficial ecchymosis over the area
Dx?
(13) Breast

Dx: Cancer, until proven otherwise

(A classical trap for the unwary. Trauma often brings the area to the attention of the patient...but is not cause of the lump.)
A 58 year old lady discovers a mass in her right axilla. She has a discreet, hard, movable, 2cm mass. Examination of her breast is negative, and she has not enlarged lymph nodes elsewhere
Dx?
Diagnostic Test? (2)
(14) Breast

Dx: Cancer, until proven otherwise

(A tough one, but another potential presentation for cancer of the breast. In a younger patient you would think lymphoma. It could still be lymphoma on her.)

Diagnostic test:
1. Mammogram
(we are now looking for an occult primary)
2. Biopsy Node
A 60 year old lady has a routine, screening mammogram. The radiologist reports an irregular area of increased density, with fine microcalcifications, that was not present two year ago on a previous mammogram
Dx?
Further Management?
(15) Breast

Dx: Cancer of the Breast

Further management: Stereotactic Radiologically guided Core Biopsy

(If unsatisfactory, the next move would be needle localized excisional biopsy)
A 44 year old lady has a 2cm palpable mass in the upper outer quadrant of her right breast. A core biopsy shows infiltrating ductal carcinoma. The mass is freely movable and her breast is of normal, rather generous size. She has no palpable axillary nodes

Tx? (2 steps)
(16) Breast

Tx:
1. Segmental Resection (Lumpectomy) and axillary node dissection
2. followed by Radiation Therapy to the remaining breast

Axillary node dissection is to help determine the need for adjuvant systemic therapy
A 62 year old lady has a 4 cm hard mass under the nipple and areola of her rather smallish left breast. A core biopsy has established a diagnosis of infiltrating ductal carcinoma. There are no palpable axillary nodes.

Management?
(17) Breast

Management: Modified Radical Mastectomy

(A Lumpectomy is an option only when the tumor is small [in absolute terms and in relation to the breast] and located where most of the breast can be spared.) A modified radical mastectomy is the choice here.

Why go after the axillary nodes when they are not palpable?: Because palpation is notoriously inaccurate in determining the presence or absence of axillary metastasis.
A 44-year-old woman has a 2-cm palpable mass in the upper outer quadrant of her right breast. A core biopsy shows lobular cancer.
(18) Breast

If they tease you with breast cancers that are not the standard infiltrating ductal carcinoma, here are the rules: lobular has a higher incidence of bilaterality (but not enough to justify bilateral mastectomy),and inflan~n~atohrays terrible prognosis. All the other varidnts of invasive cancer have a little better prognosis than infiltrating ductal, and they are all treated the same way any- way.
A 44-year-old woman has a 2-cm palpable mass in the upper outer quadrant of her right breast. A core biopsy shows medullary cancer of the breast.
(19) Breast

If they tease you with breast cancers that are not the standard infiltrating ductal carcinoma, here are the rules: lobular has a higher incidence of bilaterality (but not enough to justify bilateral mastectomy),and inflan~n~atohrays terrible prognosis. All the other varidnts of invasive cancer have a little better prognosis than infiltrating ductal, and they are all treated the same way any- way.
52-year-old woman has a suspicious area on mammogram. Multiple radiologically guided core biopsies show ductal carcinoma in situ.
(20) Breast

No axillary sampling is needed. If the lesion is confined to one quadrant, lumpectomy and radi- ation should be performed. If there are multicentric lesions all over the breast, simple total mas- tectomy is needed.
A 32-year-old woman in the seventh month of pregnancy is found to have a 2-cm mass in her left breast. Mammogram shows no other lesions, and core biopsy reveals infiltrating ductal carcinoma.
(21) Breast

Again, pregnancy imposes very little limitations to our handling of breast cancer. The only no- no's are: no radiation therapy during the pregnancy, and no chemotherapy during the first trimester. Termination of the pregnancy is not needed.
A 44 year old lady shows up in the Emergency Room because she is “bleeding from the breast”. Physical exam shows a huge, fungating, ulcerated mass occupying the entire right breast, and firmly attached to the chest wall. The patient maintains that the mass has been present for only “a few weeks”, but a relative indicates that it has been there at least two years, maybe longer.
Dx?
Diagnostic Test?
Management?
(22) Breast

Dx: Advanced Cancer of the Breast

Diagnostic Test: Core or an Incisional biopsy

Management: currently inoperable, and incurable as well...but palliation can be offered.
Chemotherapy is the first line of treatment.
(In many cases the tumor will shrink enough to become operable)
A 37 year old lady has a lumpectomy and axillary dissection for a 3cm infiltrating ductal carcinoma. The pathologist reports clear surgical margins and metastatic cancer in 4 out of 17 axillary nodes.

Management?
(23) Breast
Management: Chemotherapy

(Only very small tumors with negative nodes and very favorable histological pattern are “cured” with surgery alone. More extensive tumors need adjuvant systemic therapy, and the rule is that premenopausal women get chemotherapy and postmenopausal women get hormonal therapy.)
A 66 year old lady has a modified radical mastectomy for infiltrating ductal carcinoma of the breast. The pathologist reports that tumor measures 4 cm. in diameter and that 7 out of 22 axillary node are positive for metastasis. The tumor is estrogen and progesterone receptor positive.

Management?
(24) Breast

Management: Hormonal therapy

(The agent used is Tamoxifen)
A 61-year-old woman has a lumpectomy and axillary dissection for infiltrating ductal carcinoma of the breast. Seven of 23 nodes in the axilla have metastat~c disease. She is also found to have liver and bone metastasis.
(25) Breast

A clear case that needs chemotherapy because of the presence of distant matastasis.
A 44 year old lady complains bitterly of severe headaches that have been present for several weeks and have not responded to the usual over-the-counter headache remedies. She is two years post-op. from modified radical mastectomy for T3, N2, M0 cancer of the breast, and she had several courses of post-op chemotherapy which she eventually discontinued because of the side effects.
Dx?
Diagnostic Test?
(26) Breast

Dx: Brain mets (until proven otherwise)

(Don’t get hung up on the TNM classification, if the numbers are not 1 for the tumor and zero for the nodes and mets, the tumor is bad.)

Diagnostic Test: CT scan of the brain
A 39 year old lady completed her last course of postoperative adjuvant chemotherapy for breast cancer six months ago. She comes to the clinic complaining of constant back pain for about 3 weeks. She is tender to palpation over two well circumscribed areas in the thoracic and lumbar spine.
Dx?
Diagnostic Test?
(27) Breast

Dx: Bone mets until proven otherwise

Diagnostic test: Bone Scan
(the most sensitive test for bone mets)

If positive, X-Rays are needed to rule out benign reasons for the scan to “light up”.
A 62 year old lady was drinking her morning cup of coffee at the same time she was applying her makeup, and she noticed in the mirror that there was a lump in the lower part of her neck, visible when she swallowed. She consult you for this and on physical exam you ascertain that she indeed has a dominant, 2cm mass on the left lobe of her thyroid as well as two smaller masses on the right lobe. They are all soft and she has no palpable lymph nodes in the neck.
Diagnostic test?
(1) Endocrine

Diagnostic test: FNA
A 21 year old college student is found on a routine physical examination to have a single, 2cm nodule in the thyroid gland. The young man had radiation to his head and neck when he was thirteen years old because of persistent acne. His thyroid function tests are normal.
Diagnostic test?
Tx?
(2) Endocrine

Diagnostic test: FNA

Tx: Surgical removal
(due to radiation leading to cancer)
A 44 year old lady has a palpable mass in her thyroid gland. She also describes losing weight in spite of a ravenous appetite, palpitations and heat intolerance. She is a thin lady, fidgety and constantly moving, with moist skin and a pulse rate of 105.
Dx?
Management/test? (3 steps)
Tx?
(3) Endocrine

Dx: A “hot” Adenoma

Management/test:
1. confirm hyperthyroidism by measuring Free T4
2. Confirm source of the excessive hormone with Radioactive Iodine Scan
3. give Beta-blocker

Tx: Surgery
(after Beta blocking)
A 22 year old male has a 2 cm round firm mass in the lateral aspect of his neck, which has been present for four months. Clinically this is assumed to be an enlarged jugular lymph node and it is eventually removed surgically. The pathologist reports that the tissue removed is normal thyroid tissue.
Dx?
Diagnostic test?
Tx?
(4) Endocrine

Dx: Follicular Carcinoma of the Thyroid (metastitic)
(There is no such thing as “lateral aberrant thyroid”)

Diagnostic test: Look for the primary with a Thyroid Scan.

Tx: Eventually Surgery
An automated blood chemistry panel done during the course of a routine medical examination indicates that an asymptomatic patient has a serum calcium of 12.1 in a lab where the upper limit of normal is 9.5. Repeated determinations are consistently between 10.5 and 12.6. Serum phosphorus is low.
Dx?
Diagnostic test? (2)
Tx?
(5) Endocrine

Dx: Parathyroid Adenoma

Diagnostic test: PTH determination and Sistimibi scan to localize the adenoma

Tx: Surgical excision
A 32 year old woman is admitted to the psychiatry unit because of wild mood swings. She is found to be hypertensive and diabetic and to have osteoporosis. (she had not been aware of such diagnosis beforehand). It is also ascertained that she has been amenorrheic and shaving for the past couple of years. She has gross centripetal obesity, with moon fascies and Buffalo hump, and thin, bruised extremities. A picture from 3 years ago shows a person of very different, more normal appearance
Dx?
Diagnostic test? (3 steps)
Tx?
(6) Endocrine

Dx: Cushings Dz
(The appearance is so typical, that you will probably be given a photograph on the test, with an accompanying brief vignette)

Diagnostic test:
1. AM and PM cortisol determinations
2. Dexamethasone suppression tests
3. MRI of the head looking for the pituitary microadenoma

Tx: removed by the trans-nasal, trans-sphenoidal route
A 28 year old lady has virulent peptic ulcer disease. Extensive medical management including eradication of H.Pylori fails to heal her ulcers. She has several duodenal ulcers in the first and second portions of the duodenum. She has watery diarrhea.
Dx?
Diagnostic test? (2 steps)
Tx?
(7) Endocrine

Dx: Gastrinoma (Zollinger-Ellison)

Diagnostic test:
1. Serum gastrin
2. CT scans (or MRI) of the pancreas looking for the tumor

Tx: Surgical excision
A second year medical student is hospitalized for a neurological work-up for a seizure disorder of recent onset. During one of his convulsions it is determined that his blood sugar is extremely low. Further work-up shows that he has high levels of insulin in the blood with low levels of C-peptide.
Dx?
Management?
(8) Endocrine

Dx: Exogenous administration of insulin
(If the C-peptide had been high along with the insulin level, the diagnosis would have been insulinoma)

Management: Psychiatric evaluation and counseling
(He is faking the disease. If it had been insulinoma, CT scan or MRI looking for the tumor in the pancreas, to be subsequently removed surgically)
A 48 year old lady has had severe, migratory necrolytic dermatitis for several years, unresponsive to all kinds of “herbs and unguents”. She is thin, has mild stomatitis and mild diabetes mellitus.
Dx?
Diagnostic test? (2)
Tx?
If this Tx is not possible, what can be done? (2)
(9) Endocrine

Dx: Glucagonoma

Diagnostic test:
1. Determine Glucagon levels
2. CT scan or MRI looking for the tumor in the pancreas.

Tx: Surgery will follow

If inoperable:
1. Somatostatin can help symptomatically
2. Streptozocin is the indicated chemotherapeutic agent
A 45 year old lady comes to your office for a “regular checkup”. On repeated determinations you confirm the fact that she is hypertensive. When she was in your office three years ago, her blood pressure was normal. Laboratory studies at this time show a serum sodium of 144 mEq/L, a serum bicarbonate of 28 mEq/L, and a serum potassium concentration of 2.1 mEq/L. The lady is taking no medications of any kind.
Dx? (2 possible)
Diagnostic test? (2 steps)
Tx for each?
(1) Surgical HTN

Dx: Hyperaldosteronism or Adrenal Adenoma

Diagnostic test:
1. Aldosterone and renin levels.
2. If confirmatory (aldo high, renin low) proceed with determinations lying down and sitting up, to differentiate Hyperplasia (not surgical) from Adenoma (surgical).

Hyperplasia Tx: Aldactone

Adenoma Tx: Imaging studies (CT scan or MRI) and Surgery
A thin, hyperactive 38 year old lady is frustrated by the inability of her physicians to help her. She has episodes of severe pounding headache, with palpitations, profuse perspiration and pallor, but by the time she gets to her doctor’s office she checks out normal in every respect.
Dx?
Diagnostic test? (2 steps)
Medication before surgery?
(2) Surgical HTN

Dx: Pheochromocytoma

Diagnostic test:
1. 24hr urinary determination of metanephrine and VMA (Vanillylmandelic acid)
2. CT scan of adrenal glands

Meds before surgery: Alpha-blockers
A 17 year old man is found to have a blood pressure of 190/115. This is checked repeatedly in both arms and it is always found to be elevated, but when checked in the legs it is found to be normal.
Dx?
Diagnostic test? (2 steps)
Tx?
(3) Surgical HTN

Dx: Coarctation of the Aorta

Diagnostic test:
1. Chest X-Ray, looking for scalloping of the ribs
2. Aortogram

Tx: Surgery
A 23 year old lady has had severe hypertension for two years, and she does not respond well to the usual medical treatment for that condition. A bruit can be faintly heard over her upper abdomen.
Dx?
Diagnostic test?
Tx? (2 possible)
(4) Surgical HTN

Dx: Renovascular Hypertension due to Fibromuscular Dysplasia

Diagnostic test: Arteriogram will precede (Tx) Surgical correction or Balloon dilatation
A 72 year old man with multiple manifestations of arteriosclerotic occlusive disease has hypertension of relatively recent onset, and is refractory to the usual medical therapy. He has a faint bruit over the upper abdomen.
Dx?
(5) Surgical HTN

Dx: Renovascular Hypertension due to arteriosclerotic plaque at the origin of the Renal Artery…or arteries
(this is usually bilateral)
Within eight hours after birth, it is noted that a baby has excessive salivation. A small, soft nasogastric tube is inserted and the baby is taken to X-Ray to have a “babygram” done. The film shows the tube coiled back upon itself in the upper chest. There is air in the gastrointestinal tract.
Dx?
Management?
Tx?
(1) Pediatric Surgery

Dx: Tracheo-esophageal fistula
(the most common type with proximal blind esophageal pouch and distal TE fistula)

Management:
1. Rule-out the associated anomalies (“VACTER”: vertebral, anal, cardiac, TE and renal/radial). The vertebral and radial will be seen in the same X-ray you already took, you need Echo for the heart, Sonogram for the kidneys and Physical Exam for the anus.

Tx: Surgical repair
A newborn baby is found on physical exam to have an imperforate anus.

Management? (2 steps)
(2) Pediatric Surgery

Management:
1. This is part of the “VACTER” (vertebral, anal, cardiac, TE and renal/radial) group, so look for the others as mentioned.
2. For the imperforate anus, look for a fistula nearby (to the vagina in little girls, to the perineum in little boys), which will help determine the level of the blind pouch and the timing and type of surgery (primary repair versus colostomy and repair later).
A newborn baby is noted to be tachypneic, cyanotic and grunting. The abdomen is scaphoid and there are bowel sounds heard over the left chest. An X-Ray confirms that there is bowel in the left thorax. Shortly thereafter, the baby develops significant hypoxia and acidosis
Dx?
Management? (4 together)
Tx?
(3) Pediatric Surgery

Dx: Congenital Diaphragmatic Hernia

Management:
1. keep the kid alive with endotracheal intubation
2. Hyperventilation (careful not to blow up the other lung)
3. Sedation
4. NG suction
(Tx: The main problem is the hypoplastic lung. It is better to wait 36 to 48 hours to do Surgery to allow transition from fetal circulation to newborn circulation)
At the time of birth it is noted that a child has a large abdominal wall defect to the right of the umbilicus. There is a normal cord, but protruding from the defect there is a matted mass of angry looking, edematous bowel loops.
Dx?
Tx?
(4) Pediatric Surgery

Dx: Gastroschisis

Tx: Pediatric Surgeon must get the bowel back into the belly; they may need to use a silicon “silo” to gradually close the abdominal wall defect.
A newborn baby is noted to have a shiny, thin, membranous sac at the base of the umbilical cord. Inside the sac one can see part of the liver, and loops of normal looking bowel.
Dx?
Management?
Tx?
(5) Pediatric Surgery

Dx: Omphalocele

Management: Look for other congenital defects. These kids can have a host of other congenital defects
Tx: Repair is performed by a Pediatric surgeon
A newborn is noted to have a moist medallion of mucosae occupying the lower abdominal wall, above the pubis and below the umbilicus. It is clear that urine is constantly bathing this congential anomaly.
Dx?
what is important regarding this repair?
(6) Pediatric Surgery

Dx: Exstrophy of the urinary bladder

Important: Repair must be done within the first 48 hours, or it will not have a good chance to succeed. It takes time to arrange for transfer of a newborn baby to a distant city that specializes in this repair. If a day or two are wasted before arrangements are made, it will be too late
Half an hour after the first feed, a baby vomits greenish fluid. The mother had polyhydramnios and the baby has Down’s syndrome. X-Ray shows a “double bubble sign”: a large air fluid level in the stomach, and smaller one in the first portion of the duodenum. There is no gas in the rest of the bowel.
Dx? (2 possible)
Management?
Tx?
(7) Pediatric Surgery

Dx: Duodenal Atresia or Annular Pancreas
(innocent vomit is clear-whitish. Green vomiting in the newborn is bad news. It means something serious)

Management: Look for other congenital anomalies first

Tx: Emergency Surgery
Half an hour after the first feed, a baby vomits greenish fluid. X-Ray shows a "double bubble sign”: a large air fluid level in the stomach, and a smaller one in the first portion of the duodenum. There is air in the distal bowel, beyond the duodenum, in loops that are not distended.
Dx? (3 possibilities)
Diagnostic test?
(8) Pediatric Surgery

Dx:
1. Incomplete obstruction from duodenal stenosis,
2. Annular Pancreas,
3. Malrotation of bowel

Diagnostic test: Contrast enema
(and if not diagnostic order a water-soluble gastrographin Upper GI study)
A newborn baby has repeated green vomiting during the first day of life, and does not pass any meconium. Except for abdominal distention, the baby is otherwise normal. X-Ray shows multiple air fluid levels and distended loops of bowel.
Dx?
Cause?
(9) Pediatric Surgery

Dx: Intestinal atresia

Cause: Vascular accident in utero
(thus there are no other congenital anomalies to look for, but there may be multiple points of atresia)
A very premature baby develops feeding intolerance, abdominal distention and a rapidly dropping platelet count. The baby is four days old, and was treated with indomethacin for a patent ductus.
Dx?
Management? (3 together)
Reasons for surgical Tx? (3)
(10) Pediatric Surgery

Dx: Necrotizing Enterocolitis

Management:
1. Stop all feedings
2. Broad spectrum antibiotics
3. IV fluids/nutrition

Tx: Surgical intervention if they develop abdominal wall erythema, air in the biliary tree or pneumoperitoneum
A three day old, full term baby is brought in because of feeding intolerance and bilious vomiting. X-Ray shows multiple dilated loops of small bowel and a “ground glass” appearance in the lower abdomen. The mother has cystic fibrosis.
Dx?
Management? (3 steps)
(11) Pediatric Surgery

Dx: Meconium Ileus

Management:
1. Gastrografin enema may be both diagnostic and therapeutic, so it is the obvious first choice.
2. If unsuccessful, surgery may be needed.
3. The kid has cystic fibrosis, and management of the other manifestations of the disease will also be needed
A three week old baby has had “trouble feeding” and it is not quite growing well. He now has bilious vomiting and is brought in for evaluation. X-Ray shows a classical “double bubble”, along with normal looking gas pattern in the rest of the bowel.
Dx?
Diagnostic test?
Tx?
(12) Pediatric Surgery

Dx: Malrotation of the bowel
(not all will show up on day one)

Diagnostic test: Contrast enema to verify the malrotation

Tx: Emergency surgery
A 3 week old first-born, full term baby boy began to vomit three days ago. The vomiting is projectile, has no bile in it, follows each feeding and the baby is hungry and eager to eat again after he vomits. He looks somewhat dehydrated and has visible gastric peristaltic waves and a palpable “olive size” mass in the right upper quadrant.
Dx?
Management? (2 steps)
Tx?
(13) Pediatric Surgery

Dx: Hypertrophic Pyloric Stenosis

Management:
1. Check electrolytes: hypokalemic, hypochloremic metabolic alkalosis may have developed (correct it).
2. Rehydrate

Tx: Ramsted Pyloromyotomy
An 8 week old baby is brought in because of persistent, progressively increasing jaundice. The bilirubin is significantly elevated and about two thirds of it is conjugated, direct bilirubin. Ultrasound rules out extrahepatic masses, serology is negative for hepatitis and sweat test is normal.
Dx?
Diagnostic test? (2)
Tx?
(14) Pediatric Surgery

Dx: Biliary Atresia

Diagnostic test:
1. HIDA scan
2. Percutaneous Liver Biopsy

Tx: Exploratory laparotomy
A two month old baby boy is brought in because of chronic constipation. The kid has abdominal distention, and plain X-Rays show gas in dilated loops of bowel throughout the abdomen. Rectal exam is followed by expulsion of stool and flatus, with remarkable improvement of the distention.
Dx?
Diagnostic test? (2)
Tx?
(15) Pediatric Surgery

Dx: Hirschsprungs’ disease (aganglionic megacolon)

Diagnostic test:
1. Barium enema will define the normal-looking aganglionic distal colon and the abnormal-looking thickness
2. Biopsy of the rectal mucosa

Tx: Surgical excision of aganglionic segment
A 9 month old, chubby, healthy looking little boy has episodes of colicky abdominal pain that make him double up and squat. The pain lasts for about one minute, and the kid looks perfectly happy and normal until he gets another colick. Physical exam shows a vague mass on the right side of the abdomen, an “empty” right lower quadrant and currant jelly stools.
Dx?
Management?
Tx?
(16) Pediatric Surgery

Dx: Intussusception

Management: Barium enema is both diagnostic and therapeutic in most cases.

Tx: If reduction is not achieved radiologically, exploratory laparotomy and manual reduction will be needed
A one year old baby is referred to the University Hospital for treatment of a subdural hematoma. In the admission examination it is noted that the baby has retinal hemorrhages.
Dx?
(17) Pediatric Surgery

Child Abuse
A one year old child is brought in with second degree burns of both buttocks. The stepfather relates that the child fell into a hot tub.
Dx?
(18) Pediatric Surgery

Child Abuse
A three year old girl is brought in for treatment of a fractured humerus. The mother relates that the girl fell from her crib. X-Rays show evidence of other older fractures at various stages of healing in different bones.
Dx?
(19) Pediatric Surgery

Child Abuse
A 4 year old boy passes a large bloody bowel movement.
Dx?
Diagnostic test?
Tx?
(20) Pediatric Surgery

Dx: Meckel’s diverticulum

Diagnostic test: Radioisotope scan looking for gastric mucosa in the lower abdomen

Tx: Surgical excision
A 6-month-old baby has occasional stridor, and episodes of respiratory distress with "crowing" respiration during which he assumes a hyperextended position. The family has also noted mild difficulty in swallowing.
(1) Cardio - congenital

The combination of pressure on the esophagus and pressure on the trachea identifies a vascu- lar ring. Barium swallow will show a typical extrinsic compression from the abnormal vessel. Bronchoscopy confirms the segmental tracheal compression and rules out diffuse tracheoma- lacia. Surgical repair is done by dividing the smaller of the double aortic arches.
A patient known to have a congenital heart defect requires extensive dental work.
Management?
(2) Cardio - congenital

Management: antibiotic prophylaxis for subacute bacterial endocarditis
During a school physical exam, a 12 year old girl is found to have a heart murmur. She is referred for further evaluation. An alert cardiology fellow recognized that she indeed has a pulmonary flow systolic murmur, but he also notices that she has a fixed split second heart sound. A history of frequent colds and upper respiratory infections is elicited.
Dx?
Diagnostic test?
Tx?
(3) Cardio - congenital

Dx: Atrial septal defect

Diagnostic test: Echocardiography

Tx: Surgical closure of the defect
A three month old boy is hospitalized for ‘failure to thrive”. He has a loud, pansystolic heart murmur best heard at the left sternal border. Chest X-Ray shows increased pulmonary vascular markings.
Dx?
Diagnostic test?
Tx?
(4) Cardio - congenital

Dx: Ventricular septal defect

Diagnostic test: Echocardiography

Tx: surgical correction
Because of a heart murmur, an otherwise asymptomatic 3-month-old baby is diagnosed with a small, restrictive ventricular septal defect located low in the muscular septum.
management?
(5) Cardio - congenital

This particular variant has a good chance to close spontaneously within the first 2 or 3 years of life, and the only treatment required is suitable antibiotic prophylaxis for subacute bacterial endocarditis.
A three day old premature baby has trouble feeding and pulmonary congestion. Physical exam shows bounding peripheral pulses and a continuous, machinery-like heart murmur.
Dx?
Diagnostic test?
Tx? (2 possible)
(6) Cardio - congenital

Patent Ductus Arteriosus

Diagnostic test: Echocardiography

Tx:
1. Surgical closure
2. Indomethacin
A premature baby girl has mild pulmonary congestion, slgns of mcreased pulmonary blood flow on x-ray, a wide pulse pressure, and a precordial machinery-like murmur. She is not in congestive failure.
(7) Cardio - congenital

Same diagnosis of patent ductus, but with no urgency, and being premature, she is a clear candidate for medical treatment with indomethacin.
A 6 year old boy is brought to the U.S. by his new adoptive parents, from an orphanage in Eastern Europe. The kid is small for his age, and has a bluish hue in the lips and tips of his fingers. He has clubbing and spells of cyanosis relieved with squatting. He has a systolic ejection murmur in the left third intercostal space. Chest X-Ray shows a small heart, and diminished pulmonary vascular markings. EKG shows right ventricular hypertrophy.
Dx?
Diagnostic test?
(8) Cardio - congenital

Dx: Tetralogy of Fallot

Diagnostic test: Echocardiogram
A 72 year old man has a history of angina and exertional syncopal episodes. He has a harsh midsystolic heart murmur best heard at the second intercostal space and along the left sternal border.
Dx?
Diagnostic test?
Definitive Tx?
When is it indicated? (2)
(9) Cardio - acquired heart dz

Dx: Aortic Stenosis

Diagnostic test: Echocardiogram

Tx: Surgical Valvular replacement

Surgery indications:
1. gradient of more than 50 mm.Hg.
2. indication of CHF, angina or syncope
A 72 year old man has been known for years to have a wide pulse pressure and a blowing, high-pitched, diastolic heart murmur best heard at the second intercostal space and along the left lower sternal border with the patient in full expiration. He has had periodic echocardiograms, and in the most recent one there is evidence of beginning left ventricular dilatation.
Heart Dx?
Diagnostic test?
Next step?
(10) Cardio - acquired heart dz

Dx: Chronic Aortic Insufficiency

Diagnostic test: Echocardiogram

Next step: Aortic valve replacement
A 26 year old drug-addicted man develops congestive heart failure over a short period of a few days. He has a loud, diastolic murmur at the right, second intercostal space. A physical exam done a few weeks ago, when he had attempted to enroll in a detoxification program was completely normal.
Heart Dx?
Management/Tx? (2 together)
(11) Cardio

Dx: Acute Aortic Insufficiency due to Endocarditis

Management:
1. Emergency valve replacement
2. Antibiotics for a long time
A 35 year old lady has dyspnea on exertion, orthopnea, paroxysmal nocturnal dyspnea, cough and hemoptysis. She has had these progressive symptoms for about 5 years. She looks thin and cachectic, has atrial fibrillation and a low-pitched, rumbling diastolic apical heart murmur. At age 15 she had rheumatic fever.
Heart disorder Dx?
Diagnostic test?
Tx?
(12) CardioDx: Mitral stenosis

Diagnostic test: Echocardiogram

Tx: Eventually surgical mitral valve repair
A 55 year old lady has been known for years to have mitral valve prolapse. She now has developed exertional dyspnea, orthopnea and atrial fibrillation. She has an apical, high pitched, holosystolic heart murmur that radiates to the axilla and back.
Dx?
Diagnostic test?
Tx? (2 possible)
(13) Cardio

Dx: Mitral Regurgitation

Diagnostic test: Echocardiogram

Tx: eventually surgical repair of the valve (Annuloplasty) or possibly valve replacement
A 55 year old man has progressive, unstable, disabling angina that does not respond to medical management. His father and two older brothers died of heart attacks before the age of 50. The patient stopped smoking 20 years ago, but still has a sedentary life style, is a bit overweight, has type two diabetes mellitus and has high cholesterol.

Management?
(14) Cardio

It’s a heart attack waiting to happen...

Management: Cardiac Catheterization
(to see if he is a suitable candidate for coronary revascularization)
A 55 year old man has progressive, unstable, disabling angina that does not respond to medical management. His father and two older brothers died of heart attacks before the age of 50. The patient stopped smoking 20 years ago, but still has a sedentary life style, is a bit overweight, has type two diabetes mellitus and has high cholesterol. Cardiac catheterization demonstrates 70% occlusion of three coronary arteries, with good distal vessels. His left ventricular ejection fraction is 65%
Management?
(15) Cardio

Management: Angioplasty

(He is lucky. He has good distal vessels...smokers and diabetics often do not...and enough cardiac function left. He clearly needs coronary bypass, and with 3-vessel disease there should be no argument for angioplasty instead of surgery)
A postoperatrve patient who underwent open heart surgery is determined to have a cardiac index of 1.7 L/rnln/m2, and a left ventricular end-diastolic pressure of 3 mm Hg.
(16) Cardio

The postoperative management of open heart surgery is a little too esoteric for the USMLE, but a little bit of applied physiology is not. You should be able to recognize a dangerously low car-
diac index, without a high end-diastolic pressure
a clear indication for increased fluid intake.
Cardiac catherization shows a "square root" sign and equalizaion of pressures (right atrial, right ventricular diastolic, pulmonary artery diastolic, pulmonary capillary wedge, and left ventricular diastolic) in a patient with dyspnea on exertion, hepatomegaly, and ascites
(17) Cardio

Another example of applied physiology. This is chronic constrictive pericarditis- which needs to be surgically relieved.
On a routine pre-employment physical examination, a chest X-Ray is done on a 45 year old chronic smoker. A “coin lesion” is found in the upper lobe of the right lung.
Dx?
Next step?
(18) Lung

Dx: Cancer of the lung

Next step: Find and older chest X-Ray if one is available (from one or more years ago). If an older X-Ray has the same unchanged lesion, it is not likely cancer. No further work up is needed now, but the lesion should be followed with periodic X-Rays.
A 65-year-old man with a 40 pack-year history of smoking gets a chest x-ray because of persistent cough. A peripheral, 2-cm "coin lesion" is found in the right lung. A chest x-ray taken 2 years ago was normal.
Management?
(19) Lung

Above age 50, "coin lesions" have an 80% chance of being malignant. In this particular man it is almost certainly cancer of the lung. These vignettes typically have already had a chest x-ray done, thus the next step in management consists of noninvasive ways to establish the diagnosis and some idea of the extension of the tumor (about two thirds of patients are already beyond surgical stage when first seen). If other findings do not dictate a different approach (we will see an example soon), start with sputum cytology and CT scan (including the upper abdomen to detect liver metastases). The next step (if needed) would be biopsy of the mass, by bron- choscopy if it is central, percutaneously if it is peripheral.
A 54 year old man with a 40 pack/year history of smoking gets a chest X-Ray because of persistent cough. A peripheral, 2cm “coin lesion” is found in the right lung. A chest X-Ray taken two years ago had been normal. CT scan shows no calcifications in the mass and no enlarged peribronchial or peritracheal lymph nodes. The man has good pulmonary function and is otherwise in good health.
Dx?
Diagnostic test?
If first Dx test does not work, what are 2 others (in order)?
(20) Lung

Dx: Cancer of the lung

Diagnostic test:
1. Start with Bronchoscopy and washings,
2. if unrewarding go to Percutaneous Needle Biopsy
3. if still unsuccessful go to Open Biopsy
(Thoracotomy and Wedge Resection)
A 72 year old chronic smoker with severe COPD is found to have a central, hilar mass on chest X-Ray. Bronchoscopy and biopsy establish a diagnosis of squamous cell carcinoma of the lung. His FEV1 is 1100, and a ventilation/perfusion scan shows that 60% of his pulmonary functions comes from the affected lung.
Management/Tx?
(21) Lung

Management: It takes an FEV1 of at least 800 to survive surgery and not be a pulmonary cripple afterwards. If this fellow got a pneumonectomy (which he would need for a central tumor) he would be left with an FEV1 of 440. No way... Don’t do any more tests. He is not a surgical candidate.

Tx: pursue Chemotherapy and Radiation
A 62 year old chronic smoker has an episode of hemoptysis. Chest X-ray shows a central hilar mass. Bronchoscopy and biopsy establish a diagnosis of squamous cell carcinoma of the lung. His FEV1 is 2200, and a ventilation/perfusion scan shows that 30% of his pulmonary function comes from the affected lung.
Diagnostic test?
Tx?
(22) Lung

Diagnostic test: CT scan and Mediastinoscopy
(to ascertain if surgery has a decent chance to cure him)

Tx: Pneumonectomy
(can tolerate it due to high FEV1)
A 33 year old lady is undergoing a diagnostic work-up because she appears to have Cushing’s syndrome. Chest X-Ray shows a central, 3cm round mass on the right lung. Bronchoscopy and biopsy confirm a diagnosis of small cell carcinoma of the lung.
Management for cancer?
(23) Lung

Management: Radiation and chemotherapy.

(Small cell lung cancer is not treated with surgery, and thus we have no need to determine FEV1 or nodal status)
A 54 year old right handed laborer notices coldness and tingling in his left hand as well as pain in the forearm when he does strenuous work. What really concerned him, though, is that in the last few episodes he also experienced transitory vertigo, blurred vision and difficulty articulating his speech. Angiogram demonstrates retrograde flow through the vertebral artery.
Dx?
Management/Tx?
(24) Vascular

Dx: Subclavian Steal syndrome

(A combination of “claudication of the arm” with posterior brain neurological symptoms is classical for this)

Management: Angiographic study (If you had been given the vignette without it), then Vascular surgery
A 62 year old man is found on physical exam to have a 6cm pulsatile mass deep in the abdomen, between the xiphoid and the umbilicus
Dx?
Tx?
(25) Vascular

Dx: Abdominal Aortic Aneurysm

Tx: Elective surgical repair
A 62 year old man has vague, poorly described epigastric and upper back discomfort. He has been found on physical exam to have a 6cm pulsatile mass deep in the abdomen, between the xiphoid and the umbilicus. The mass is tender to palpation.
Dx?
Management?
Tx?
(26) Vascular

Dx: Abdominal Aortic Aneurysm that is beginning to leak.

Management: Get a consultation with the vascular surgeons today
A 68 year old man is brought to the ER with excruciating back pain that began suddenly 45 minutes ago. He is diaphoretic and has a systolic blood pressure of 90. There is an 8cm pulsatile mass palpable deep in his abdomen, between the xiphoid and the umbilicus.
Dx?
Tx?
(27) Vascular

Dx: Abdominal Aortic Aneurysm, rupturing right now.

Tx: Emergency surgery
A retired businessman has claudication when walking more than 15 blocks.

Management?
(28) Vascular

Management: If he is smoking he should quit; otherwise he needs nothing

(Vascular surgery, or angioplasty and stenting are palliative procedures. They do not cure arteriosclerotic occlusive disease. Claudication has an unpredictable course, thus there is no advantage to an “early operation”)
A 56 year old postman describes severe pain in his right calf when he walks two or three blocks. The pain is relieved by resting 10 or 15 minutes, but recurs if he walks again the same distance. He can not do his job this way, and he does not qualify yet for retirement, so he is most anxious to have this problem resolved. He does not smoke.

Diagnostic test? (2 steps)
Tx?
(29) Vascular

Diagnostic test:
1. Start with Doppler studies
2. If he has significant gradient, Arteriogram comes next

Tx: Bypass surgery or stenting
A patient consults you because he “can not sleep”. On questioning it turns out that he has pain in the right calf, which keeps him from falling asleep. He relates that the pain goes away if he sits by the side of the bed and dangles the leg. His wife adds that she has watched him do that, and she has noticed that the leg which was very pale when he was lying down becomes deep purple several minutes after he is sitting up. On physical exam the skin of that leg is shiny, there is no hair and there are no palpable peripheral pulses.
Dx?
Diagnostic test? (2 steps)
Tx?
(30) Vascular

Dx: Claudication

Dx test:
1. Start with Doppler studies
2. If he has significant gradient, Arteriogram comes next

Tx: Bypass surgery or stenting
A 45 year old man shows up in the ER with a pale, cold, pulseless, paresthetic, painful and paralytic lower extremity. The process began suddenly two hours ago. Physical exam shows no pulses anywhere in that lower extremity. Pulse at the wrist is 95 per minute, grossly irregular.
Dx?
Tx?
(31) Vascular

Dx: Embolization by the broken-off tail of a clot from the left atrium

Tx: Emergency surgery with use of Fogarty catheters to retrieve the clot
A 74 year old man has sudden onset of extremely severe, tearing chest pain that radiates to the back and migrates down shortly after it’s onset. His blood pressure is 220/100, he has unequal pulses in the upper extremities and he has a wide mediastinum on chest X-Ray. Electrocardiogram and cardiac enzymes show that he does not have a myocardial infarction
Dx?
Management w/ high BP? (2)
Normal BP?
Tx? (depends on area; 2 possible)
(32) Vascular

Dx: Dissecting aneurysm of the thoracic Aorta

Management:
1. if high BP, beta-blockers or IV nitrates to lower BP
(b/c forces that dissected the vessel plus the force of the dye injection could further shear the aorta)
2. Arteriogram
(first if BP is normal)

Tx:
Ascending Aorta = emergency surgery
Descending Aorta = intensive therapy (in the ICU) for the hypertension will be the preferable option.
A 65 year old West Texas farmer of Swedish ancestry has an indolent, raised, waxy, 1.2 cm skin mass over the bridge of the nose that has been slowly growing over the past three years. There are no enlarged lymph nodes in the head and neck.
Dx?
Diagnostic Test?
Tx?
(1) Skin

Dx: Basal cell carcinoma

Diagnostic test: Full thickness biopsy at the edge of the lesion (punch or knife)

Treatment: Surgical excision with clear margins, but conservative width
A 71 year old West Texas farmer of Irish ancestry has a non-healing, indolent, punched out, clean looking 2 cm ulcer over the left temple, that has been slowly becoming larger over the past three years. There are no enlarged lymph nodes in the head and neck.
Dx?
Diagnostic Test?
Tx?
(2) Skin

Dx: Basal cell carcinoma

Diagnostic Test: Full thickness biopsy at the edge of the lesion (punch or knife)

Tx: Surgical excision with clear margins, but conservative width
A blond, blue eyed, 69 year old sailor has a non-healing, indolent 1.5 cm. ulcer on the lower lip, that has been present, and slowly enlarging for the past 8 months. He is a pipe smoker, and he has no other lesions or physical findings.
Dx?
Diagnostic Test?
Tx? (2 possible)
(3) Skin

Dx: Squamous cell carcinoma

Diagnostic test: Biopsy

Treatment: Surgical resection with wider (about 1 cm) clear margins.
Local radiation therapy is another option
A red headed 23 year old lady who worships the sun, and who happens to be full of freckles, consults you for a skin lesion on her shoulder that concerns her. She has a pigmented lesion that is asymmetrical, with irregular borders, of different colors within the lesion, and measuring 1.8 cms
Dx?
Diagnostic Test?
(4) Skin
Dx: Melanoma or Dysplastic Nevus

Diagnostic test: full thickness biopsy at the edge of the lesion

- margin free local excision if superficial melanoma
(Clarks’ levels one or two, or under 0.75 mm)
- wide local excision with 2 or 3 cm margin if deep melanoma
A 35 year old blond, blue eyed man left his native Minnesota at age 18, and has been living the life as a crew member for a sailing yacht charter operation in the Caribbean. He has multiple nevi all over his body, but one of them has changed recently.
Dx?
Management?
(5) Skin

Dx: Melanoma

(Change in a pigmented lesion is the other tip off to melanoma...It may be growth, or bleeding, or ulceration, or change in color)

Management: Full-thickness biopsy at the edge of the lesion

- margin free local excision if superficial melanoma
(Clarks’ levels one or two, or under 0.75 mm)
- wide local excision with 2 or 3 cm margin if deep
A 44 year old man has unequivocal signs of multiple liver metastasis, but no primary tumor has been identified by multiple diagnostic studies of the abdomen and chest. The only abnormality in the physical exam is a missing toe, which he says was removed at the age of 18 for a black tumor under the toenail.
Dx?
Diagnostic Test for initial problem?
(6) Skin

Dx: Malignant Melanoma

(the alternate version has a glass eye, and history of enucleation for a tumor. No self-respecting malignant tumor would have this time interval, but melanoma will)

Diagnostic Test: full thickness biopsy at the edge of the lesion

- margin free local excision if superficial melanoma
(Clarks’ levels one or two, or under 0.75 mm)
- wide local excision with 2 or 3 cm. margin if deep melanoma
A 32 year old gentleman had a Clark’s level 5, 3.4 mm. Deep, melanoma removed from the middle of his back three years ago. He now has…(a tumor in a weird place, like his left ventricle, his duodenum, his ischiorectal area...anywhere!)
Dx?
(7) Skin

Dx: Melanoma

(The point of this vignette is that invasive melanoma...it has to be deep...metastasizes to all the usual places [lymph nodes plus liver-lung-brain-bone] but it is also the all-time-champion in going to weird places where few other tumors dare to go)
A 1-year-old child is suspected of having strabismus. You verify that indeed the corneal reflection from a bright light in your examining room comes from different places from each of his eyes.
(1) Opthalmology

What is the point of these vignettes? To remind you that the brain "learns" to see what the eyes see during early infancy (up to about age 7). If one eye cannot see (any kind of obstruction) or the brain does not like what they see (double vision), the brain will refuse to process the image and that cortical "blindness" will be permanent (the concept of amblyopia).
Management. The problem has to be surgically corrected as early as possible.
A 2-year-old child is diagnosed with a congenital cataract obstructing his vision in the right eye.
(2) Opthalmology

What is the point of these vignettes? To remind you that the brain "learns" to see what the eyes see during early infancy (up to about age 7). If one eye cannot see (any kind of obstruction) or the brain does not like what they see (double vision), the brain will refuse to process the image and that cortical "blindness" will be permanent (the concept of amblyopia).
Management. The problem has to be surgically corrected as early as possible.
A young mother is visiting your office for routine medical care. She happens to have her 18 month old baby with her, and you happen to notice that one of the pupils of the baby is white, while the other one is black.
Dx Differential? (2)
(3) Opthalmology

Dx Diff: Retinoblastoma or Cataracts

(An ophthalmological and potentially life-and-death emergency. A white pupil (leukocoria) at this age can be retinoblastoma. This kid needs to see the ophthalmologist not next week, but today or tomorrow. If it turns out to be something more innocent, like a cataract, the kid still needs it corrected to avoid amblyopia.)
A 53 year old lady is in the ER complaining of extremely severe frontal headache. The pain started about one hour ago, shortly after she left the movies where she watched a double feature. On further questioning, she reports seeing halos around the lights in the parking lot when leaving the theater. On physical exam the pupils are mid-dilated, do not react to light, the corneas are cloudy and with a greenish hue, and the eyes feel “hard as a rock”.
Dx?
Management?
Medicine Tx? (3 possible)
(4) Opthalmology

Dx: Acute glaucoma

(most are asymptomatic)

Management: An ophthalmologist is needed stat

Tx:
1. Diamox
2. Pilocarpine drops
3. Mannitol
A 32 year old lady presents in the E.R. with swollen, red, hot, tender eyelids on the left eye. She has fever and leukocytosis. When prying the eyelids open, you can ascertain that her pupil is dilated and fixed and that she has very limited motion of that left eye.
Dx?
Management?
Tx?
(5) Opthalmology

Dx: Orbital Cellulitis

Management: CT scan

(Ophthalmological emergency that requires immediate consultation)

Tx: Surgical drainage
A frantic mother reaches you on the phone, reporting that her 10 year old boy accidentally splashed Drano on his face and is screaming in pain complaining that his right eye hurts terribly.
Management?
(6) Opthalmology

Management: The key is immediate irrigation. Instruct the mother to pry the eye open under the cold water tap at home, and irrigate for about ½ hour before she brings the kid to the hospital.
A 59 year old, myopic gentleman reports “seeing flashes of light” at night, when his eyes are closed. Further questioning reveals that he also sees “floaters” during the day, that they number ten or twenty, and that he also sees a cloud at the top of his visual field.
Dx?
Management and Tx?
(7) Opthalmology

Dx: Retinal Detachment

(that “cloud” at the top of the visual field is hemorrhage settling at the bottom of the eye)

Management: Another Ophthalmological emergency. The retina specialist will use Laser treatment to “spot weld” the retina back in place
A 77 year old man suddenly loses sight from the right eye. He calls you on the phone 10 minutes after the onset of the problem. He reports no other neurological symptoms.
Dx?
Management?
(8) Opthalmology

Dx: Embolic occlusion of the retinal artery

Management: Another ophthalmological emergency...although little can be done for the problem.
He has to get the ER instantly and it might help for him to breathe into a paper bag on route, and have someone press hard on his eye and release repeatedly
A 55 year old man is diagnosed with type two diabetes mellitus. On questioning about eye symptoms he reports that sometimes after a heavy dinner the television becomes blurry and he has to squint to see it clearly.
Dx?
Management?
(9) Opthalmology

Dx: Simply DM-related changes in eye

(no big deal: the lens swells and shrinks in response to swings in blood sugar)

Management: regular ophthalmological follow up for retinal complications
A 15 year old girl has a round, 1cm cystic mass in the midline of her neck at the level of the hyoid bone. When the mass is palpated at the same time that the tongue is pulled, there seems to be a connection between the two. The mass has been present for at least 10 years, but only recently bothered the patient because it got infected and drained some pus.
Dx?
Tx?
(1) ENT

Dx: Thyroglossal Duct Cyst

Tx: Sistrunk operation
(removal of the mass and the track to the base of the tongue, along with the medial segment of the hyoid bone).
An 18 year old woman has a 4cm fluctuant round mass on the side of her neck, just beneath and in front of the sternomastoid. She reports that is has been there at least 10 years, although she thinks that it has become somewhat larger in the last year or two. A CT scan shows the mass to be cystic.
Dx?
Tx?
(2) ENT

Dx: Branchial Cleft Cyst

Tx: Elective surgical removal
A 6 year old child has a mushy, fluid filled mass at the base of the neck, that has been noted for several years. The mass is about 6 cm. in diameter, occupies most of the supraclavicular area and seems by physical exam to go deeper into the neck and chest.
Dx?
Diagnostic test?
Tx?
(3) ENT

Dx: Cystic hygroma

Diagnostic test: CT scan to see how deep this thing goes.
(They can extend down into the chest and mediastinum)

Tx: Surgical removal will eventually be done
A 22 year old lady notices an enlarged lymph node in her neck. The node is in the jugular chain, measures about 1.5cm, is not tender, and was discovered by the patient yesterday. The rest of the history and physical exam are unremarkable.
Management?
(4) ENT

Management: Reschedule an appointment for 3 weeks to see its progress

(If the node has gone away by then, it was inflammatory and nothing further is needed. If it’s still there, it could be neoplastic and something needs to be done)
A 22 year old lady seeks help regarding an enlarged lymph node in her neck. The node is in the jugular chain, measures about 2cm, is firm, not tender, and was discovered by the patient six weeks ago. There is a history of low grade fever and night sweats for the past three weeks. Physical examination reveals enlarged lymph nodes in both axillas and in the left groin.
Dx?
Diagnostic test?
(5) ENT

Dx: Lymphoma (most likely)

Disgnostic test: Tissue diagnosis will be needed. You can start with FNA of the available nodes, but eventual node biopsy will be needed to establish not only the diagnosis but also the type of lymphoma
A 72 year old man has 4cm hard mass in the left supraclavicular area. The mass is movable, non tender and has been present for three months. The patient has had a 20 pound weight loss in the past two months, but is otherwise asymptomatic.
Dx?
Management? (2)
(6) ENT

Dx: Malignant mets to a supraclavicular node from a primary tumor below the neck.

Management:
1. Look for the obvious primary tumors: lung, stomach, colon, pancreas, and kidney
2. The node itself will eventually be Biopsied
A 69 year old man who smokes and drinks and has rotten teeth has a hard, fixed, 4cm mass in his neck. The mass is just medial and in front of the sternomastoid muscle, at the level of the upper notch of the Thyroid cartilage. It has been there for at least six months, and it is growing.
Dx?
Diagnostic test?
(7) ENT

Dx: Metastatic squamous cell carcinoma to a jugular chain node, from a primary in the mucosa of the head and neck
(oro-pharyngeal-laryngeal territory)

Diagnostic test: Triple Endoscopy
(examination under anesthesia of the mouth, pharynx, larynx, esophagus and tracheobronchial tree)

(Don’t biopsy the node! FNA is OK if Triple endoscopy not available)
A 69 year old man who smokes and drinks and has rotten teeth has hoarseness that has persisted for six weeks in spite of antibiotic therapy
Dx?
Diagnostic test?
(8) ENT

Dx: Squamous cell carcinoma of the mucosa of the head and neck

Diagnostic test: Triple endoscopy to find and biopsy the primary tumor
A 69 year old man who smokes and drinks and has rotten teeth has a painless ulcer in the floor of the mouth that has been present for 6 weeks and has not healed.
Dx?
Diagnostic test?
(9) ENT

Squamous cell carcinoma of the mucosa of the head and neck

Diagnostic test: Triple endoscopy to find and biopsy the primary tumor
A 23-year-old man with AIDS has a painless ulcer in the floor of the mouth that has been present for 6 weeks and has not healed. He does not smoke or drink.
Dx?
Diagnostic test?
(10) ENT

Squamous cell carcinoma of the mucosa of the head and neck

Diagnostic test: Triple endoscopy to find and biopsy the primary tumor
A 69 year old man who smokes and drinks and has rotten teeth has unilateral ear ache that has not gone away in 6 weeks. Physical examination shows serious otitis media on that side, but not on the other.
Dx?
Diagnostic test?
(11) ENT

Dx: Squamous cell carcinoma of the mucosa of the head and neck

Diagnostic test: Triple endoscopy to find and biopsy the primary tumor
A 52 year old man complains of hearing loss. When tested he is found to have unilateral sensory hearing loss on one side only. He hoes not engage in any activity (such as sport shooting) that would subject that ear to noise that spares the other side.
Dx?
Diagnostic test?
(12) ENT

Dx: Acoustic Nerve Neuroma
(Unilateral versions of common ENT problems in the adult suggest malignancy. Note that if the hearing loss had been conductive, a Cerumen Plug would be the obvious first diagnosis)

Diagnostic test: MRI looking for the tumor
A 56 year old man develops slow, progressive paralysis of the facial nerve on one side. It took several weeks for the full blown paralysis to become obvious, and it has been present now for three months. It affects both the forehead as well as the lower face.
Dx?
Diagnostic test?
(13) ENT

Dx: Gradual, unilateral nerve paralysis suggests a neoplastic process

Diagnostic test: Gadolinium enhanced MRI
A 45 year old man presents with a 2cm firm mass in front of the left ear, which has been present for four months. The mass is deep to the skin and it is painless. The patient has normal function of the facial nerve.
Dx?
Management?
(14) ENT
Dx: Pleomorphic adenoma (mixed tumor) of the parotid gland

Management: Referral to a head and neck surgeon for formal superficial parotidectomy
(FNA is appropriate, but the point of the question will be to bring out the fact that parotid masses are never biopsied in the office or under local anesthesia)
A 65 year old man present with a 4cm hard mass in front of the left ear, which has been present for six months. The mass is deep to the skin and it is fixed. He has constant pain in the area, and for the past two months has had gradual progression of left facial nerve paralysis. He has rock-hard lymph nodes in the left neck.
Dx?
Management?
(15) ENT
Dx: Cancer of the parotid gland

Management: Referral to a head and neck surgeon for formal superficial parotidectomy
(Amateurs should not mess with parotid)
A two year old boy has unilateral ear ache.
Dx?
(16) ENT

Dx: Unilateral versions of common bilateral ENT conditions in toddlers suggest Foreign Body
A two year old has unilateral foul smelling purulent rhinorrhea.
Dx?
(17) ENT

Dx: Unilateral versions of common bilateral ENT conditions in toddlers suggest Foreign Body
A two year old has unilateral wheezing and the lung on that side looks darker on X-Rays (more air) than the other side.
Dx?
(18) ENT

Dx: Unilateral versions of common bilateral ENT conditions in toddlers suggest Foreign Body
A 45 year old lady with a history of a recent tooth infection shows up with a huge, hot, red, tender, fluctuant mass occupying the left lower side of her face and upper neck, including the underside of the mouth. The mass pushes up the floor of the mouth on that side. She is febrile.
Dx?
Tx? (2 together)
(19) ENT

Dx: Ludwigs’ Angina
(An abscess of the floor of the mouth)

Tx:
1. Tracheostomy
2. Incision & Drainage of the abscess
A 29 year old lady calls your office at 10 AM with the history that she woke up that morning with one side of her face paralyzed.
Dx?
Management?
(20) ENT

Dx: Bell’s palsy

Management: Immediate anti-viral medication
(the process is idiopathic and will resolve spontaneously in most cases)
A patient with multiple trauma from a car accident is being attended to in the emergency room. As multiple invasive things are done to him, he repeatedly grimaces with pain. The next day it is noted that he has a facial nerve paralysis on one side.
Dx?
(21) ENT

Dx: Paralysis from Edema

(Trauma to the temporal bone can certainly transect the facial nerve, but when that happens the nerve is paralyzed right there and then. Nothing needs to be done...it will correct itself)
Your office receives a phone call from Mrs. Rodriguez. You know this middle aged lady very well because you have repeatedly treated her in the past for episodes of sinusitis. In fact, six days ago you started her on decongestants and oral antibiotics for what you diagnosed as frontal and ethmoid sinusitis. Now she tells you over the phone that ever since she woke up this morning, she has been seeing double.
Dx? (2 possible)
Management? (3 steps)
Dx test?
(22) ENT

Dx: Cavernous Sinus Thrombosis or Orbital Cellulitis

Management: This is a real emergency (fact that is most likely questioned).
1. Immediate Hospitalization,
2. high dose IV Antibiotic treatment
3. Surgical Drainage of the paranasal sinuses or the orbit.

Dx Test: CT scan
(which will also be needed to guide the surgery)
A 10 year old girl has epistaxis. Her mother says that she picks her nose all the time.
Dx?
Tx?
(23) ENT

Dx: Bleeding from the Anterior part of the septum

Tx: Phenylephrine spray and local pressure
An 18 year old boy has epistaxis. The patient denies picking his nose. No source of anterior bleeding can be seen by physical examination.
Dx? (2 possible)
(24) ENT

Dx:
1. Septal perforation from cocaine abuse
2. Posterior juvenile Nasopharyngeal Angiofibroma
A 72, hypertensive male, on aspirin for arthritis, has a copious nosebleed. His blood pressure is 220/105 when seen in the E.R. He says he began swallowing blood before it began to come out through the front of his nose.
Dx?
Management? (2)
(25) ENT

Dx: Epistaxis secondary to hypertension

Management:
1. Lower BP with Medication
2. Involve ENT

(These are serious problems that can end up with death)
A 57-year-old man seeks help for "dizziness." On further questioning he explains that he gets light-headed and unsteady, but the room is not spinning around.

Dx?
Management?
(26) ENT

What is it? Neurologic, probably vascular occlusive- but not inner ear. Direct your management and workup in that direction.
A 57 year old man seeks help for “dizziness”. On further questioning he explains that the room spins around him
Dx?
Management?
(27) ENT

Dx: Vestibular Apparatus

Management: Symptomatic treatment (meclizine, phenergan, diazepam), or an ENT workup
A 62 year old right handed man has transient episodes of weakness in the right hand, blurred vision, and difficulty expressing himself. There is not associated headache, the episodes last about 5 or 10 minutes at the most, and they resolve spontaneously. Fundoscopic examination reveals highly refractile crystals in the left retinal artery.
Dx?
Diagnostic test?
Tx?
(1) Neurosurgery - vascular

Dx: Transient Ischemic Attacks
in the territory of the left carotid artery
(probably an ulcerated plaque at the left carotid bifurcation)

Diagnostic test: Angiogram

Treatment: Carotid endarterectomy
A 61 year old man presents with a one year history of episodes of vertigo, diplopia, blurred vision, dysarthria and instability of gait. The episodes last several minutes, have no associated headache and leave no neurological sequela.
Dx?
Diagnostic test?
Tx?
(2) Neurosurgery - vascular

Dx: Transient Ischemic Attacks
(but now the vertebrals may be involved)

Diagnostic test: Arteriogram that examines all the arteries going to the brain (i.e. an aortic arch study)

Tx: Vascular surgery will follow
A 60 year old diabetic male presents with abrupt onset of right third nerve paralysis and contralateral hemiparesis. There was no associated headache. The patient is alert, but has the neurological deficits mentioned.
Dx?
Diagnostic test?
(3) Neurosurgery - vascular

Dx: Stroke
(Neurological catastrophes that begin suddenly and have no associated headache are vascular occlusive)

Diagnostic test: CT scan

(Vascular surgery in the neck is designed to prevent strokes, not to treat them once they happen)
A 64 year old black man complains of a very severe headache of sudden onset and then lapses into a coma. Past medical history reveals untreated hypertension and examination reveals a stuporous man with profound weakness in the left extremities.
Dx?
Diagnostic test?
Tx?
(4) Neurosurgery - vascular

Dx: Vascular Hemorrhagic stroke
(Neurological catastrophes of sudden onset with severe headache)

Diagnostic test: CT scan

Tx: Supportive with eventual rehabilitation efforts if he survives.
A 39 year old lady presents to the ER with a history of a severe headache of sudden onset that she says is different and worse than any headache she has ever had before. She is given pain medication and sent home. She improves over the next few days, but ten days after the initial visit she again gets a sudden, severe and singular diffuse headache and she returns to the ER. This time she has some nuchal rigidity on physical exam.
Dx?
Diagnostic test? (2 steps)
Tx?
(5) Neurosurgery - vascular

Dx: Subarachnoid bleeding from an intracranial aneurysm.
(the nuchal rigidity betrays the presence of blood in the subarachnoid space)

Diagnostic test:
1. CT scan to find bleeder
2. Angiograms will eventually follow, in preparation for (Tx) Surgery to clip the aneurysm
A 31 year old nursing student developed persistent headaches that began approximately 4 months ago, have been gradually increasing in intensity and are worse in the mornings. For the past three weeks, she has been having projectile vomiting. Thinking that she may need new glasses, she seeks help from her optometrist, who discovers that she has bilateral papilledema.
Dx?
Diagnostic test?
Management until surgery? (3)
(6) Neurosurgery - tumors

Dx: Brain Tumor

(Neurological processes that develop over a period of a few months and lead to increased intracranial pressure, spell out tumor)

Diagnostic test: MRI
(If not offered, settle for CT scan)

Management: Measures to decrease intracranial pressure include Mannitol, Hyperventilation, and high dose Steroids (decadron).
A 42 year old right handed man has a history of progressive speech difficulties and right hemiparesis for five months. He has had progressively severe headaches for the last two months. At the time of admission he is confused, vomiting, has blurred vision, papilledema and diplopia. Shortly thereafter his blood pressure goes up to 190 over 110, and he develops bradychardia.
Dx?
Management? (3 together)
Tx?
(7) Neurosurgery - tumors

Dx: Brain tumor
(but now with two added features...there are localizing signs: left hemisphere, parietal and temporal area...and he manifests the Cushing’s reflex of extremely high intracranial pressure)

Management: Emergent Decrease ICP w/ Mannitol, Hyperventilation and Steroids

Tx: Surgery
A 42 year old man has been fired from his job because of inappropriate behavior. For the past two months he has gradually developed very severe, “explosive” headaches that are located on the right side, above the eye. Neurologic exam shows optic nerve atrophy on the right, papilledema on the left and anosmia.
Specific Dx?
Diagnostic test?
Tx?
(8) Neurosurgery - tumors

Dx: Brain tumor in the right frontal lobe
(Foster-Kennedy syndrome)

(A little knowledge of neuroanatomy can help localize tumors. The frontal lobe has to do with behavior and social graces, and is near the optic nerve and the olfactory nerve)

Diagnostic test: MRI

Tx: Neurosurgery
A 12 year old boy is short for his age, has bitemporal hemianopsia and has a calcified lesion above the sella in X-Rays of the head.
Dx?
Diagnostic test?
Tx?
(9) Neurosurgery - tumors

Dx: Craniopharyngioma

Diagnostic test: MRI

Tx: Pituitary surgery
A 23 year old nun presents with a history of amenorrhea and galactorrhea of six months duration. She is very concerned that other may think that she is pregnant, and she vehemently denies such a possibility.
Dx?
Diagnostic test? (2 steps)
Tx?
If Tx is not possible, what medication?
(10) Neurosurgery - tumors

Dx: Prolactinoma

Diagnostic test:
1. Measure Prolactin level
(Every time you suspect a functioning tumor of an endocrine gland, you measure the appropriate hormone)
2. MRI to see tumor for surgery

Tx: Trans-nasal, trans-sphenoidal

If inoperable: Bromocriptine
A 44 year old man is referred for treatment of hypertension. His physical appearance is impressive: he has big, fat, sweaty hands; large jaw and thick lips, large tongue and huge feet. He is also found to have a touch of diabetes. In further questioning he admits to headaches and he produces pictures of himself taken several years ago, where he looks strikingly different.
Dx?
Diagnostic test? (2 steps)
Tx?
(11) Neurosurgery - tumors

Dx: Acromegaly

Diagnostic test:
1. Growth hormone levels
2. MRI for surgery

Tx: Pituitary surgery
A 15 year old girl has gained weight and become “ugly”. She shows a picture of herself a year ago, where she was a lovely young lady. Now she has a hairy, red, round face full of pimples; her neck has a posterior hump and her supraclavicular areas are round and convex. She has a fat trunk and thin extremities. She has mild diabetes and hypertension.
Dx?
Diagnostic test? (3 steps)
Tx? (3 possible)
(12) Neurosurgery - tumors

Dx: Cushing’s syndrome

Diagnostic test:
1. AM and PM cortisol levels
2. Dexamethasone suppression test
3. MRI of the sella

Tx:
1. Cushings Dz: Trans-sphenoidal pituitary surgery
2. Adrenal CA: Adrenalectomy
3. Ectopic ACTH: remove Primary tumor
A 55 year old lady is involved in a minor traffic accident where her car was hit sideways by another car that she “did not see” at an intersection. When she is tested further it is recognized that she has bitemporal hemianopsia. Ten years ago she had bilateral adrenalectomies for Cushing’s disease.
Dx?
Diagnostic test?
Tx?
(13) Neurosurgery - tumors

Dx: Nelson’s syndrome
(Years ago, before imaging studies could identify pituitary microadenomas, patients with Cushing’s were treated with bilateral adrenalectomy instead of pituitary surgery. In some of those patients the pituitary microadenoma kept on growing and eventually gave pressure symptoms)

Diagnostic test: MRI will show the tumor

Tx: Trans-nasal, trans-sphenoidal surgery will remove it
A27-year-oldwomandevelopsasevereheadacheofsuddenonset,making her stuporous. She is taken to the hospital, where she is found at admission to have a blood pressure of 75 over 45. Funduscopic examination reveals bilateral pallor of the optic nerves. Relatives indicate that for the past six months, she has been complaining of morning headaches, loss of peripheral vision, and amenorrhea. After she developed the severe headache, and just before she went into a deep stupor, she told her relatives that her peripheral vision had suddenly deteriorated even more than before.
(14) Neurosurgery - tumors

What is it?Pituitary apoplexy. (She has bled into a pituitary tumor.) Management.Steroid replacement is urgently needed. Other hormones will need to be replaced
eventually. MRI or CT scan will determine extent of the problem.
A 32 year old man complains of progressive, severe generalized headaches that began three months ago are worse in the mornings and lately have been accompanied by projectile vomiting. He has lost his upper gaze and he exhibits the physical finding known as “sunset eyes”.
Specific Dx?
Diagnostic test?
Tx?
(15) Neurosurgery - tumors

Dx: Tumor is in the pineal gland (Parinaud’s syndrome)

Diagnostic test: MRI

Tx: Neurosurgery
A six year old boy has been stumbling around the house and complaining of severe morning headaches for the past several months. While waiting in the office to be seen, he assumes the knee-chest position as he holds his head. Neurological exam demonstrates truncal ataxia.
Specific Dx?
Diagnostic test?
Tx?
(16) Neurosurgery - tumors

Dx: Tumor of the Posterior Fossa.
(Most brain tumors in children are located there, and cerebellar function is affected)

Diagnostic test: MRI

Tx: Neurosurgery
A 23 year old man develops severe headache, seizures and projectile vomiting over a period of two weeks. He has low grade fever, and was recently treated for acute otitis media and mastoiditis.
Dx?
Diagnostic test?
Tx?
(17) Neurosurgery - tumors

Dx: Brain abscess
(Signs and symptoms suggestive of brain tumor that develop in a couple of weeks with fever and an obvious source on infection, spell out abscess)

Diagnostic test: These are seen in CT as well as they would on MRI, and the CT is cheaper and easier to get...so pick CT if offered.

Tx: Resected by the neurosurgeons
A 52 year old lady has constant, severe back pain for two weeks. While working on her yard, she suddenly falls and can not get up again. When brought to the hospital she is paralyzed below the waist. Two years ago she had a mastectomy for cancer of the breast.
Dx?
Diagnostic test?
Tx?
(18) Spinal Cord

Dx: Canacer metastasis causing Spinal fracture
(Most tumors affecting the spinal cord are metastatic, extradural; the sudden onset of the paralysis suggests a fracture with cord compression or transection)

Diagnostic test: MRI is the best imaging modality for the spinal cord.

Tx: Neurosurgeons may be able to help if the cord is compressed rather than transected
A 45 year old male gives a history of aching back pain for several months. He has been told that he had muscle spasms, and was given analgesics and muscle relaxants. He comes in now because of the sudden onset of very severe back pain that came on when he tried to lift a heavy object. The pain is “like an electrical shock that shoots down his leg”, and it prevents him from ambulating. He keeps the affected leg flexed. Straight leg raising test gives excruciating pain.
Dx?
Diagnostic test?
Management? (2 possible)
(19) Spinal Cord

Dx: Lumbar disk Herniation
(The peak age incidence is 45, and virtually all of these are either L4-L5 or L5-S1)

Diagnostic test: MRI

Management:
1. Bed rest will take care of most of these
2. Neurosurgical intervention only if there is progressive weakness or sphincteric deficits
A 79 year old man complains of leg pain brought about by walking and relieved by rest. On further questioning it is ascertained that he has to sit down or bend over for the pain to go away. Standing at rest will not do it. Furthermore, he can exercise for long periods of time if he is “hunched over”, such as riding a bike or pushing a shopping cart. He has normal pulses in his legs.
Dx?
Diagnostic test?
Tx?
(20) Spinal Cord

Dx: Neurogenic Claudication

Diagnostic test: MRI

Tx: Eventually surgical decompression of this cauda equina
AbusinessexecutivewhohasbeenaT6paraplegicformanyyearsisheldat a business meeting for several hours beyond the time when he would normally have done his in-and-out self catheterization of the urinary bladder. He develops a pounding headache, profuse perspiration, and bradycardia. His blood pressure is 220 over 120.
(21) Spinal Cord

The classic picture of autonomic dysreflexia. Obviously his bladder needs to be emptied, but he also needs alpha-adrenergic blocking agents and may benefit from calcium-channel blockers (such as nifedipine).
A 60 year old man complains of extremely severe, sharp, shooting, “like a bolt of lighting”, pain in his face which is brought about by touching a specific area, and which lasts about 60 seconds. His neurological exam is normal, but it is noted that part of his face is unshaven, because he fears to touch that area.
Dx?
Diagnostic test?
Tx?
(22) Pain syndromes

Dx: Tic Doloreaux (Trigeminal neuralgia)

Diagnostic test: Rule out organic lesions with MRI

Tx: Anticonvulsants
Several months after sustaining a crushing injury of his arm, a patient complains bitterly about constant, burning, agonizing pain that does not respond to the usual analgesic medications. The pain is aggravated by the slightest stimulation of the area. The arm is cold, cyanotic and moist.
Dx?
Management? (2 possible)
(23) Pain syndromes

Dx: Causalgia (reflex sympathetic distrophy)

Management:
1. Sympathetic block is diagnostic
2. Surgical sympathectomy will be curative
A 14 year old boy presents in the Emergency Room with very severe pain of sudden onset in his right testicle. There is no fever, pyuria or history of recent mumps. The testis is swollen, exquisitely painful, “high riding”, and with a “horizontal lie”. The cord is not tender.
Dx?
Tx?
(1) Urology

Dx: Testicular Torsion
(urological emergency)

Tx: Emergency surgery to save the testicle
A 24 year old man presents in the emergency room with very severe pain of recent onset in his right scrotal contents. There is fever of 103 and pyuria. The testis is in the normal position, and it appears to be swollen and exquisitely painful. The cord is also very tender.
Dx?
Diagnostic test?
Tx?
(2) Urology

Dx: Acute Epididimitis

Diagnostic test: Ultrasound (to rule-out torsion)

Tx: Antibiotics

(The differential diagnosis is with testicular torsion. Torsion is a surgical emergency. Epididimitis is not. Don’t rush this guy to the OR. If the vignette is not clear-cut, i.e: and adolescent that looks like epidimitis, but could be torsion, pick a sonogram to rule out torsion before you choose the non-surgical option)
A 72 year old man is being observed with a ureteral stone that is expected to pass spontaneously. He develops chills, a temperature spike to 104 and flank pain.
What should be given to him?
What is initial Tx? (2)
(3) Urology

Give: Massive IV Antibiotics

Tx:
Decompression by:
1. Ureteral stent, or
2. Percutaneous Nephrostomy

(Obstruction and Infection of the urinary tract: a true urological emergency. In a septic patient stone extraction would be hazardous)
An adult female relates that five days ago she began to notice frequent, painful urination, with small volumes of cloudy and malodorous urine. For the first three days she had no fever, but for the past two days she has been having chills, high fever, nausea and vomiting. Also in the past two days she has had pain in the right flank. She has had no treatment whatsoever up to this time
Dx?
Management? (3 steps)
(4) Urology

Dx: Pyelonephritis

Management:
1. Hospitalization
2. IV antibiotics
3. Sonogram to make sure that there is no concomitant obstruction

(UTI should not happen in men or in children, and thus they should trigger looking for a cause. Women of reproductive age on the other hand, get cystitis all the time and they are treated with appropriate antibiotics without great fuss)
A 62 year old male presents with chills, fever, dysuria, urinary frequency, diffuse low back pain and an exquisitely tender prostate on rectal exam
Dx?
Management? (2 steps)
(5) Urology

Dx; Acute Bacterial Prostatitis

Management:
1. I.V. antibiotics
2. what should not be done is any more rectal exams or any vigorous prostatic massage...doing so could lead to septic shock
A 33 year old man has urgency, frequency, and burning pain with urination. The urine is cloudy and malodorous. He has mild fever. On physical exam the prostate is not warm, boggy or tender
Dx?
Management? (3 together)
(6) Urology

Dx: Urinary Tract Infections

Management:
1. start Urinary cultures
2. start Antibiotics
3. either IVP or Sonogram
You are called to the nursery to see an otherwise healthy looking newborn boy because he has not urinated in the first 24 hours of life. Physical exam shows a big distended urinary bladder.
Dx? (2 possible)
First step?
Diagnostic test?
Tx?
(7) Urology

Dx: Urinary Obstruction secondary to
1. Meatal Stenosis
2. Posterior Urethral valves

First step: Drain the bladder with a catheter
(it will pass through the valves)

Diagnostic test: Voiding cystourethrogram

Tx: Endoscopic Fulguration or Resection
A bunch of newborn boys are lined up in the nursery for you to do circumcisions. You notice that one of them has the urethral opening in the ventral side of his penis, about mid-way down the shaft.
Dx?
Next step?
(8) Urology

Dx: Hypospadias

Next step: The point of the vignette is that you don’t do the circumcision. The foreskin may be needed later for reconstruction when the hypospadias is surgically corrected
A 7 year old child falls off a jungle gym and has minor abrasions and contusions. When checked by his pediatrician, a urinalysis shows microhematuria
Dx?
Diagnostic test?
(9) Urology

Dx: Congenital Anomaly
(Hematuria from the trivial trauma in kids means congenital anomaly of some sort)

Diagnostic test: start with Sonogram (IVP may be needed later)
A 9 year old boy gives a history of three days of burning on urination, with frequency, low abdominal and perineal pain, left flank pain and fever and chills for the past two days
Dx?
Management? (2 steps)
(10) Urology

Dx: UTI
(Little boys are not supposed to get urinary tact infections. There is more than meets the eye here. A congenital anomaly has to be ruled out)

Management:
1. treat the infection
2. Sonogram right away to begin the work up
A mother brings her 6-year-old girl to you because “ she has failed miserably to get proper toilet training”. On questioning you find out that the little girl perceives normally the sensation of having to void, voids normally and at appropriate intervals, but also happens to be wet with urine all the time
Dx?
Management? (2 steps)
Tx?
(11) Urology

Dx: (classic vignette) Low implantation of one ureter
(In little boys there would be no symptoms, because low implantation in boys is still above the sphincter, but in little girls the low ureter empties into the vagina and has no sphincter. The other ureter is normally implanted and accounts for her normal voiding pattern)
Management:
1. PE might show the abnormal ureteral opening
2. IVP

Tx: Surgical repair
A 16 year old boy sneaks out with his older brother’s friends, and goes on a beer-drinking binge for the first time in his life. He shortly thereafter develops colicky flank pain
Dx?
Diagnostic test?
Tx?
(12) Urology

Dx: (classic) Ureteropelvic Junction Obstruction

Diagnostic test: Ultrasound

Tx: Surgical Repair will follow
A 62 year old male known to have normal renal function reports an episode of gross, painless hematuria. Further questioning determines that the patient has total hematuria rather than initial or terminal hematuria
Dx? (2 possible)
Diagnostic test?
If test is normal what is next step?
(13) Urology

Dx: Either Infection or Tumor can produce hematuria.
(The blood is coming anywhere from the kidneys to the bladder, rather than the prostate or the urethra. In older patients without signs of infection cancer is the main concern)

Diagnostic test: IVP
(“gold standard-first study” in urology, except in postential obstruction, then Ultrasound)
If normal the next step: Cystoscopy
A 70 year old man is referred for evaluation because of a triad hematuria, flank pain and a flank mass. He also has hypercalcemia, erythrocytosis and elevated liver enzymes
Dx?
Diagnostic test? (2 steps)
(14) Urology

Dx: Renal cell carcinoma (also known as clear cell carcinoma, or hypernephroma)

Diagnostic test:
1. IVP first
2. CT scan next would be the standard sequence.
(In real life, if a urologist saw a patient with a palpable flank mass, he or she might go straight for the CT scan)
A 61 year old man presents with a history of hematuria. Intravenous pyelogram shows a renal mass, and sonogram shows it to be solid rather than cystic. CT scan shows a heterogenic, solid tumor.
Dx?
(15) Urology

Dx: Renal cell carcinoma
A 55 year old, chronic smoker, reports three instances in the past two weeks when he has had painless, gross, total hematuria. In the past two months he has been treated twice for irritative voiding symptoms, but has not been febrile and urinary cultures have been negative
Dx?
Diagnostic test? (2 steps)
(16) Urology

Dx: Bladder Cancer

Diagnostic test:
1. IVP
2. Cystogram
(With this very complete presentation some urologist would go for the cystoscopy first, but the standard sequence of IVP first and cystoscopy next is the only correct answer for an exam. An option both IVP and cystoscopy would be OK)
A 59 year old black man has a rock-hard, discrete, 1.5cm nodule felt in his prostate during a routine physical examination
Dx?
Diagnostic test?
Tx?
(17) Urology

Dx: Cancer of the Prostate

Diagnostic test: Trans-rectal needle biopsy

Tx: Surgical resection after the extent of the disease has been established
A59-year-oldblackmanistoldbyhisprimarycarephysicianthathisprostatic specific antigen (PSA) has gone up significantly since his last visit. He has no palpable abnormalities in his prostate by rectal exam.
Dx?
Diagnostic test?
Tx?
(18) Urology

Dx: Cancer of the Prostate

Diagnostic test: Trans-rectal needle biopsy

Tx: Surgical resection after the extent of the disease has been established
A62-year-oldmanhadaradicalprostatectomyforcanceroftheprostate3 years ago. He now presents with widespread bony pain. Bone scans show metastases throughout the entire skeleton, including several that are very large and very impressive.
(19) Urology

Management. The point of the vignette is that significant, often dramatic palliation can be obtained with orchiectomy, although it will not be long-lasting (1 or 2 years only). An expen- sive alternative is luteinizing hormone-releasing hormone agonists, and another option is antiandrogens (flutamide).
A 78-year-old man comes in for a routine medical checkup. He is asymptomatic. When a physician had seen him 5 years earlier, a PSA had been ordered, but he notices as he leaves the office this time that the study has not been requested. He asks if he should get it.
(20) Urology

Management. He should not get the test. After a certain age, most men get prostatic cancer, but die of something else. As a rule, asymptomatic prostatic cancer is not treated after the age of 75. Therefore, there is no point in looking for it.
A 25 year old man presents with a painless, hard testicular mass.
Dx?
Diagnostic test? (2)
(21) Urology

Dx: Testicular cancer

Diagnostic test:
1. Pre-op Alpha-fetoprotein and Beta-HCG
2. Diagnosis is made by performing a radical orchiectomy by the inguinal route.
(That irreversible, drastic step is justified because testicular tumors are almost never benign. Beware of the option to do a trans-scrotal biopsy: that is a definitive no-no)
A 25 year old man is found on a pre-employment chest X-Ray to have what appears to be a pulmonary metastasis from an unknown primary tumor. Subsequent physical examination discloses a hard testicular mass, and the patient indicates that for the past six months he has been losing weight for no obvious reason.
Dx?
Diagnostic test?
Tx? (2 steps)
(22) Urology

Dx: Testicular Cancer with metastasis.

Diagnostic test:
pre-op Blood Test for Alpha-fetoprotein and Beta-HCG levels

Tx:
1. Removal of testicle
2. Chemotherapy
(The point of this vignette is that testicular cancer responds so well to chemotherapy, that treatment is undertaken regardless of the extent of the disease when first diagnosed)
A 60 year old man shows up in the ER because he has not been able to void for the past 12 hours. He wants to, but can not. On physical exam his bladder is palpable half way up between the pubis and the umbilicus, and he has a big, boggy prostate gland without nodules. He gives a history that for several years now, he has been getting up four or five times a night to urinate. Because of a cold, two days ago he began taking anthihystaminics, using “nasal drops”, and drinking plenty of fluids.
Dx?
Management?
Tx? (2 possible)
(23) Urology

Dx: Acute urinary retention, with underlying BPH

Management: Indwelling bladder catheter, to be left in for at least 3 days

Tx: long-term Alpha-blockers for symptomatic relief, or some form of Prostatic Resection
On the second post-operative day after surgery for repair of bilateral inguinal hernias, the patient reports that he “can not hold his urine”. Further questioning reveals that every few minutes he urinates a few cc’s of urine. On physical examination there is a large palpable mass arising from the pelvis and reaching almost to the umbilicus.
Dx?
Management?
(24) Urology

Dx: Acute Urinary Retention with Overflow Incontinence

Management: Indwelling bladder catheter
A 42 year old lady consults you for urinary incontinence. She is the mother of five children and ever since the birth of the last one, seven years ago, she leaks a small amount of urine whenever she sneezes, laughs, gets out of a chair or lifts any heavy objects. She relates that she can hold her urine all through the night without any leaking whatsoever
Dx?
Tx?
(25) Urology

Dx: Stress Incontinence

Tx: Surgical repair of the pelvic floor.
A 72 year old man who in previous years has passed a total of three urinary stones is now again having symptoms of ureteral colic. He has relatively mild pain that began six hours ago, and does not have much in the way of nausea and vomiting. X-Rays show a 3mm Ureteral stone just proximal to the ureterovesical junction
Management? (3 together)
(26) Urology

Management:
1. Watch him (time)
2. Pain medication
3. Plenty of Fluids

(there is still a role for watching and waiting. This man is a good example: small stone, almost at the bladder. Give him time, medication for pain, and plenty of fluids, and he will probably pass it)
A 54 year old lady has a severe ureteral colic. IVP shows a 7mm Ureteral stone at the ureteropelvic junction
Tx?
(27) Urology

Tx: Shock-wave Lithotripsy

(whereas a 3mm stone has a 70% chance of passing, a 7mm stone only has a 5% probability of doing so. This one will have to be smashed and retrieved)
A 72 year old man consults you with a history for that for the past several days he has noticed that bubbles of air come out along with the urine when he urinates. He also gives symptoms suggestive of mild cystitis
Dx? (2 possible)
Diagnostic test?
Tx?
(28) Urology

Dx: Pneumaturia due to a Fistula between the bowel and the bladder.
(Most commonly from sigmoid colon to dome of the bladder, due to diverticulitis)
or Sigmoid Cancer

Diagnostic test: CT scan
(Intuitively you would think that either cystoscopy, sigmoidoscopy or contrast studies would verify the diagnosis, but they seldom show anything in this case)

Tx: Surgery will be needed
A 32 year old man has sudden onset of impotence. One month ago he was unexpectedly unable to perform with his wife after an evening of heavy eating and heavier drinking. Ever since then he has not been able to achieve an erection when attempting to have intercourse with his wife, but he still gets nocturnal erections and can masturbate normally
Dx?
Management?
(29) Urology

Dx: Classical Psychogenic Impotence
(young man, sudden onset, partner-specific. Organic impotence is typically older, of gradual onset and universal)

Management: Curable with psychotherapy if promptly done
(It will become irreversible after two years)
Ever since he had a motorcycle accident where he crushed his perineum, a young man has been impotent
(30) Urology

Organic impotence has sudden onset only when it is related to trauma. Vascular injury explains the first of these two, and vascular reconstruction may help. Nerve injury accounts for the second, and only prosthetic devices can help there.
Ever since he had an abdominoperineal resection for cancer of the rectum, a 52-year-old man has been impotent.
(31) Urology

Organic impotence has sudden onset only when it is related to trauma. Vascular injury explains the first of these two, and vascular reconstruction may help. Nerve injury accounts for the second, and only prosthetic devices can help there.
A66-year-olddiabeticmanwithgeneralizedarter~osclerot~occlusivedisease notices gradual loss of erectile function. At first he could get erections, but they did not last long; later the quality of the erection was poor; and eventually he developed complete impotence. He does not get nocturnal erections.
(32) Urology

This is the classic pattern of organic impotence (not related to trauma). A wide range of thera- peutic options exists, but probably the first choice now is Viagra (sildenafil).
A 62-year-old man who had a motorcycle accident has been in a coma for several weeks. He is on a respirator, has had pneumonia on and off, has been on vasopressors, and shows no signs of neurologic improvement. The family inquires about brain death and possible organ donation.
(1) Transplant
At one time we were very fussy about who was accepted as an organ donor. Nowadays, with 65,000 patients on transplant waiting lists, and many dying every day for lack of organs, we take almost anybody. The rule now is that allpotential donors are referred to the local organ har- vesting organization. They will still reject some, but there are no blanket exclusions any more. Donors with specific infections (such as hepatitis) can be used for recipients with the same infection. Even donors with metastatic cancer are eligible for eye donation.
Ten days after liver transplantation, levels of y-glutamyltransferase (CCT), alkaline phosphatase, and bilirubin begin to go up. There is no ultrasound evidence of biliary obstruction or Doppler evidence of vascular thrombosis.
(2) Transplant

There are three kinds of rejection. Hyperacute rejection happens within minutes of re-estab- lishing blood supply, produces thrombosis, and is caused by preformed antibodies. ABO match- ing and lymphocytotoxic crossmatch prevents it, and thus we do not see it clinically- and you will not encounter it in the USMLE.
Acute rejection is the one we deal with all the time. It occurs after the first 5 days, and usually within the first few months. Signs of organ dysfunction (as in these vignettes) suggest it, but biopsy is what confirms it. In the case of the heart, there are no early clinical signs; thus biop- sies there are done routinely at set intervals. Once diagnosed, the first line of therapy is steroid boluses. If unsuccessful,antilymphocyte agents (OKT3) are used.
On the third week after a closely matched renal transplant, there are early clinical and laboratory signs of decreased renal function.
(3) Transplant

There are three kinds of rejection. Hyperacute rejection happens within minutes of re-estab- lishing blood supply, produces thrombosis, and is caused by preformed antibodies. ABO match- ing and lymphocytotoxic crossmatch prevents it, and thus we do not see it clinically- and you will not encounter it in the USMLE.
Acute rejection is the one we deal with all the time. It occurs after the first 5 days, and usually within the first few months. Signs of organ dysfunction (as in these vignettes) suggest it, but biopsy is what confirms it. In the case of the heart, there are no early clinical signs; thus biop- sies there are done routinely at set intervals. Once diagnosed, the first line of therapy is steroid boluses. If unsuccessful,antilymphocyte agents (OKT3) are used.
Two weeks after a lung transplant, the patient develops fever, dyspnea, hypoxemia, decreased FEV,, and interstitial infiltrate on chest x-ray.
(4) Transplant

There are three kinds of rejection. Hyperacute rejection happens within minutes of re-estab- lishing blood supply, produces thrombosis, and is caused by preformed antibodies. ABO match- ing and lymphocytotoxic crossmatch prevents it, and thus we do not see it clinically- and you will not encounter it in the USMLE.
Acute rejection is the one we deal with all the time. It occurs after the first 5 days, and usually within the first few months. Signs of organ dysfunction (as in these vignettes) suggest it, but biopsy is what confirms it. In the case of the heart, there are no early clinical signs; thus biop- sies there are done routinely at set intervals. Once diagnosed, the first line of therapy is steroid boluses. If unsuccessful,antilymphocyte agents (OKT3) are used.
Several years after a successful (renal, hepatic, cardiac, pulmonary) transplantation, there is gradual, insidious loss of organ function.
(5) Transplant

This is the third form: chronic rejection. Poorly understood, and irreversible. We have no treat- ment for it, but the correct answer for such vignette would be to do biopsy. Late acute rejection episodes could be the problem, and we can treat those.